Download as pdf or txt
Download as pdf or txt
You are on page 1of 131

Chapter 1

GİRİŞ

Bu dersin yakın amacı, MAT-2 (İntegral ve Uygulamaları) dersine hazırlık


yapmaktır. Uzak amacı ise, soyut düşünebilme yeteneǧini ve dolayısıyla da
problem çözebilme yeteneǧini geliştirmektir. (Bu yüzden, kendinize biz bu
konuları öǧretecek miyiz ki öǧreniyoruz benzeri sorular sormayınız.)
Herhangi bir matematik dersinde olduǧu gibi, bu dersi çalışırken de ezberden
kaçınınız; öǧrenmek için çok çalışınız. Bunun için şunlara dikkat etmelisiniz:

1. Derslere mutlaka devam ediniz. Matematik konuları birbirine önkoşul


ilişkisiyle baǧlı olduǧundan, kaçırdıǧınız bir dersin eksikliǧini girdiǧiniz
sonraki derste mutlaka hissedersiniz. 4 haftalık devamsızlık hakkı ka-
nunen tanınan bir haktır, sadece olaǧanüstü durumlarda kullanınız.

2. Dersi ders anında anlamaya çalışınız, olmadı günü gününe çalışınız,


daha olmadı haftasına çalışınız, ama, nasıl olsa sınavlar yaklaştıǧında
çalışırım düşüncesiyle hareket etmeyiniz. Bu durum çalışmayı zevkli
yapmak yerine işkence yapar. Dahası, böyle durumlarda genellikle çok
geç kalınır.

3. Her derste mutlaka soru sorunuz. Bu, hem merakınızı giderip dersten
zevk almanızı saǧlar, hem derse katılgınlıǧınızı arttırır, hem de dersin
öǧretim elemanına öǧrenme düzeyiniz hakkında dönüt (geri bildirim)
verir.

4. Matematik roman okur gibi çalışılmaz. Çalışırken mutlaka elinizin


altında kaǧıt ve elinizde kalem bulundurunuz. Unutmayın ki, en meşhur

1
2 CHAPTER 1. GİRİŞ

matematikçiler en basit problem(!) için bile bunu yaparlar. Çünkü,


yapılan iş soyuttur.

5. Bir problemi çözemediǧinizde, bir konu veya kavramı anlayamadıǧı-


nızda, başınıza en olaǧan şey gelmiş demektir, panik yapmayınız. Arka-
daşlarınızla tartışınız veya konuyu bir arkadaşınıza anlatarak anlamaya
çalışınız. En sonunda dersin öǧretim elemanına danışınız. (Bu konuda
şu tavsiyede bulunanlar da vardır: Bir konuyu anlayamadıǧınızda o
konuda bir ders anlatın, yine anlayamazsanız o kunuda bir kitap yazın,
yine anlayamazsanız yazdıǧınız kitabı dikkatlice çalışın.)

6. Derste gördüǧünüz bir konuyu tekrar çalışırken, konuyu kavramış ol-


mak için kendi cümlelerinizle tekrar yazabilmelisiniz.

7. ”Sabahlara kadar ders çalıştım ama başaramadım” diyen öǧrenci ol-


mayınız. Çünkü, uykusuz yapılacak bir etkinlik deǧildir matematik.
Dinç ve zinde olmalısınız; hele hele sınavlara uykusuz girmemelisiniz.

8. Son olarak, Matematikçi olmak demek bir matematik bölümü bitirmek


demek deǧildir. Yukarıdakilere ek olarak, soru sormayı ve sorgulamayı
seviyorsanız, çalıştıǧınız konu veya çözdüǧünüz problemler size yeni
problemlerin, soruların kapısını aralıyorsa ve bu soruları ifade edebili-
yorsanız, siz iyi bir matematikçisiniz.

İzleyen Temel Bilgiler bölümünde küme, sayı, fonksiyon kavramları ders


için yetecek kadar irdelenecektir.
Türev bölümünde söz konusu kavram tanımlanacak ve sık kullanacaǧımız
fonksiyonların türevleri hesaplanacaktır. Uygulamalar kısmı türevin mate-
matik ve çevremizdeki ilginç bazı uygulamalarını içerecektir.
Belirsiz integral bölümünde uygulamalarda karşımıza çıkan bazı fonksi-
yonların integralleri ile integral alma teknikleri incelenecektir.
Son bölüm olan Belirli integral bölümünde ise integral kavramının çıkış
noktasına gidecek, Newton ile Leibniz’in izlediǧi yolu keşfederek alanların
kolaylıkla ve kesin olarak nasıl hesaplanabileceǧini göreceǧiz.
Chapter 2

TEMEL BİLGİLER

Matematik nedir? Hepimizde beliren ortak düşünce, genellikle sayılar ile


yapılan bir zihinsel etkinlik oluşudur. Sayının tanımı ise birbirine denk olan
kümelerin ortak özelliǧi olunca, küme kavramı ön plana çıkmaktadır. Bu
yüzden, hemen her temel matematik dersinde olduǧu gibi biz de kümeler
konusu ile başlayacaǧız.

2.1 KÜME KAVRAMI


Nasıl ki nokta kavramı tanımlanamıyorsa, ancak sezgi yoluyla anlatılmaya
çalışılıyorsa, küme için de bu yol izlenir. Küme de tanımsız kabul edilen bir
kavramdır, tanımlama çabasına girişilmemelidir. Küme, varlıklar topluluǧu,
bazı özelliklere sahip nesneler topluluǧu gibi cümlelerle anlatılabilir.
Kümeler A, B, C,... gibi büyük harflerle gösterilir. Kümeyi oluşturan nes-
nelere eleman denir. Küme elemanları da a, b, x, y,... gibi küçük harflerle
gösterilir. Eǧer a bir A kümesinin elemanı ise, bunu

a∈A

biçiminde gösteririz. Eǧer x bir A kümesinin elemanı deǧil ise, bunu da

x 6∈ A

biçiminde gösteririz. Bir kümenin belirli olması için, onun elemanlarının


teker teker verilmesi veya o kümenin elemanlarının belirtilmesine yarayan

3
4 CHAPTER 2. TEMEL BİLGİLER

karakteristik bir özelliǧinin verilmesi gerekir. Yani, kümenin elemanlarının


anlamlı ve belirli olması gerekir.
Buna göre, bir kümeyi gösterirken ya o kümenin elemanlarını {· · · } şeklinde
parantez arasına, ya da

{x : x’in karakteristik özelliǧi}

şeklinde yazarız. Örneǧin, elemanları a, b, c olan kümeye A dersek, bunu

A = {a, b, c}

ile veya
A = {x : x Türkçe alfabenin ç’ye kadarki harfleri}
şeklinde gösterebiliriz. Bu yazışta, kümenin genel elemanı x ile gösterilmiş,
”:” işareti de ”öyle ki” anlamına gelen bir sembol olarak kullanılmıştır.
Kolaylık olsun diye bazen küme ve elemanlarını bir bölge içinde kalan nok-
talar ile göstermek gelenek olmuştur. Bu şekillere Venn Diyagramları denir
(John Venn, 1806-1923). Böylece yukarıdaki kümeyi şöyle gösterebiliriz:

Şekil 2.1 A kümesi.

Venn diyagramından da yararlanarak kümelerle ilgili 3 durumun sözkonusu


olduǧunu söyleyebiliriz. Bunlar şöyle gösterilebilir:

Şekil 2.2 Kümelerin birbirine göre durumları.


2.1. KÜME KAVRAMI 5

Şimdi kümelerle ilgili temel kavramları vereceǧiz.


Tanım 2.1. (Alt Küme) Eǧer bir A kümesinin her bir elemanı B kümesinin
de bir elemanı ise A kümesi B kümesinin bir alt kümesidir denir ve A ⊂ B
şeklinde gösterilir. Eǧer A kümesi B’nin alt kümesi deǧilse bu durum A 6⊂ B
ile gösterilir. Kısaca

A ⊂ B ⇔ (x ∈ A ⇒ x ∈ B)

yazabiliriz.
Tanım 2.2. (Eşitlik) İki kümenin eşitliǧi

A = B ⇐⇒ [A ⊂ B ve B ⊂ A]

denkliǧi ile verilir. A’nın en az bir elemanı B’de deǧilse A ile B farklıdır
denir ve A = 6 B şeklinde gösterilir.
Not 2.1. Bu tanıma göre A ve B eşit kümelerse aynı elemanlardan oluşurlar.
Eǧer A ⊂ B fakat A 6= B ise, A’ya B’nin öz alt kümesi denir.
Tanım 2.3. (Boş küme) Hiç elemanı olmayan kümeye boş küme denir ve
∅ (Fi) ile gösterilir.

Boş küme için


∅ := {x : x 6= x}
tanımı da yapılabilir. Boş kümenin tanımından şu teorem ve zarif ispatı
verilebilir:
Teorem 2.1. ∅ (boş küme) her kümenin alt kümesidir.

İspat: A herhangi bir küme olsun. ∅ 6⊂ A ise, bu ∅ kümesinin A’da olmayan


en az bir elemanı olduǧu anlamına gelir. Bu ise ∅’nin tanımına aykırıdır.2
Tanım 2.4. (Evrensel küme) Bir problemde göz önüne aldıǧımız tüm
kümeleri içeren bir küme (veya kümeler) düşünebiliriz. Bu kümeye evrensel
küme deyip E ile göstereceǧiz.

Örneǧin, sınıfımızdaki öǧrencileri bir küme olarak düşünürsek, bu küme için


evrensel küme Fen Bilgisi öǧretmenliǧi öǧrencileri, okulumuz öǧrencileri veya
üniversitemiz öǧrencileri olabilir.
6 CHAPTER 2. TEMEL BİLGİLER

Tanım 2.5. (İki kümenin birleşim kümesi) A ve B kümelerinin eleman-


larından oluşan ümeye A ile B’nin birleşimi denir ve A ∪ B ile gösterilir.

A ∪ B kümesi aşaǧıdaki şekilde de tanımlanır.

A ∪ B := {x : x ∈ A veya x ∈ B}

Buna göre, kümelerin birbirine göre şu durumları söz konusudur:

Şekil 2.3 Kümelerin birleşimi.

Görülüyor ki A ⊂ A ∪ B ve B ⊂ A ∪ B’dir.

Tanım 2.6. (İki kümenin kesişim kümesi) A ile B kümelerinin ortak ele-
manlarından oluşan kümeye A ile B’nin kesişimi denir ve A ∩ B ile gösterilir.

A ∩ B kümesi şu şekilde de tanımlanır:

A ∩ B := {x : x ∈ A ve x ∈ B}.

Buna göre, kümelerin birbirine göre şu durumları söz konusudur:

Şekil 2.4 Kümelerin kesişimi.

O halde, A ∩ B ⊂ A ve A ∩ B ⊂ B’dir.

Tanım 2.7. (Ayrık kümeler) A ∩ B = ∅ ise A ile B kümelerine ayrık


kümeler diyeceǧiz.
2.1. KÜME KAVRAMI 7

Tanım 2.8. (İki kümenin fark kümesi) A kümesinde olup da B’de bulun-
mayan elemanların kümesine A’nın B’den farkı denir ve A \ B ile gösterilir.

Aynı tanım şöyle de yapılabilir:

A \ B := {x : x ∈ A ve x 6∈ B}.

Tanım 2.9. (İki kümenin simetrik fark kümesi) İki kümenin kesişimi
dışındaki elemanlarının kümesine, bu iki kümenin simetrik fark kümesi denir
ve A 4 B ile gösterilir.

Buna göre A 4 B = (A \ B) ∪ (B \ A) yazılabilir.


Tanım 2.10. (Bir kümenin tümleyeni) Evrensel kümenin A kümesinden
farkına A kümesinin tümleyeni denir ve A0 ile gösterilir.

Buna göre, A0 := {x : x 6∈ A} yazılabilir.

Şekil 2.5 A kümesinin tümleyeni.


Sonuç 2.1. 8 ve 10 tanımlarından görülüyor ki, aşaǧıdakiler yazılabilir.
1. A \ B = A ∩ B 0

2. ∅0 = E ve E 0 = ∅

3. (A0 )0 = A
Teorem 2.2. A ve B herhangi iki küme ise,
1. (A ∪ B)0 = A0 ∩ B 0

2. (A ∩ B)0 = A0 ∪ B 0

dür. Bunlar De Morgan kuralları olarak bilinir.


8 CHAPTER 2. TEMEL BİLGİLER

İspat:
1. x ∈ (A ∪ B)0 ⇐⇒ x ∈ A0 ∩ B 0 olduǧunu göstermeliyiz. Gerçekten,

x ∈ (A ∪ B)0 ⇐⇒ x 6∈ A ∪ B
⇐⇒ x 6∈ A ve x 6∈ B
⇐⇒ x ∈ A0 ve x ∈ B 0
⇐⇒ x ∈ A0 ∩ B 0
bulunur. Benzer olarak 2’yi gösteriniz. 2

Tanım 2.11. (Sonlu küme) Sonlu sayıda elemanı olan kümeye sonlu küme
diyecek ve eleman sayısını s(A) ile göstereceǧiz.

Ödev 2.1. s(A ∪ B) = s(A) + s(B) − s(A ∩ B) eşitliǧini ve bundan yarar-


lanarak aşaǧıdaki eşitliǧi gösteriniz.

s(A∪B∪C) = s(A)+s(B)+s(C)−s(A∩B)−s(A∩C)−s(B∩C)+s(A∩B∩C)

Tanım 2.12. (İki kümenin denkliǧi) Aralarında birebir eşleme yapılabi-


len A ve B kümelerine birbirine denktir denir ve A ≡ B ile gösterilir.

ALIŞTIRMALAR (Kümeler)

1. A, B ve C kümeleri için aşaǧıdaki eşitliklerin doǧruluǧunu gösteriniz.

(a) A ∩ B=B ∩ A
(b) A \ B=B 0 \ A0
(c) A 4 B=B 4 A
(d) A ∩ (B ∪ C)=(A ∩ B) ∪ (A ∩ C)
(e) A \ A=∅

2. Aşaǧıdaki önermelerin doǧruluǧunu gösteriniz.

(a) A ∩ B=A ⇔ A ⊂ B
(b) A ∪ B=B ⇔ A ⊂ B
(c) A ⊂ B ⇔ A0 ⊃ B 0
(d) A ∩ B=∅ ⇔ B ⊂ A0
2.2. SAYILAR 9

2.2 SAYILAR
Birbirine denk sonlu elemanlı iki kümenin ortak özelliǧi nedir diye soracak
olursak, cevabımız onların çokluklarının aynı olmasıdır. İşte bu çokluǧa doǧal
sayı diyoruz. Buna göre, 1 sembolü ile gösterdiǧimiz sayı {∗} kümesine denk
olan bütün kümelerin ortak özelliǧidir. Benzer olarak diǧer doǧal sayılar
tanımlanır. Doǧal sayıların kuruluşundaki temel esas nesnelerin belli bir
sayıda sürekli olarak gruplanmasıdır. Bu gruplamada, bir gruptaki eleman
sayısına taban, ardışık gruplamaların her birine de basamak adı verilir. Kul-
landıǧımız taban on’luk tabandır. (Nedenini düşününüz.) Belli bir tabanda
kullanılan sembollere rakam diyeceǧiz.
Temel sayı kümeleri olarak şunları göz önüne alacaǧız:
Rakamlar: {0, 1, 2, · · · , 9}
Sayma sayıları: N+ = {1, 2, 3, · · · }
Doǧal (Naturel) sayılar: N = {0, 1, 2, · · · }
Tam (Zahlen) sayılar: Z = {0, ∓1, ∓2, · · · }
Z kümesi de günlük yaşantımıza cevap veremediǧinden, rasyonel sayıları
tanımlarız. Çünkü, bx = a (b 6= 0) denkleminin kökü her zaman tamsayı
deǧildir. Bu denklemin çözümü x = ab olduǧundan, bu biçimdeki sayıları
tanımlamaya gereksinim duyarız:
Rasyonel (Quotient) sayılar: Q = { ab : a, b ∈ Z ve b 6= 0}
Rasyonel sayılarla işlemlerin nasıl yapıldıǧını biliyorsunuz, üzerinde durmaya-
caǧız. Bu sayı kümesinin bir özelliǧi de devirli ondalık kesirlerden oluşmaları-
dır. Q kümesi ile ilgili olarak şunu da söyleyelim.
a c.a
=
b c.b
olduǧundan, Q’daki sayı tekrarını önlemek için a ile b sayılarını aralarında
asal kabul edeceǧiz.
Q kümesi de günlük yaşantıda yeterli olmadıǧından, örneǧin x2 = 5 denk-
leminin kökleri ab şeklinde (yani rasyonel) olmadıǧından (neden?), rasyonel ol-
mayan (irrasyonel) sayılar kümesi’ni tanımlarız. Aşaǧıdaki teorem (tarihte)
irrasyonel sayıların ilk çıkış noktası olması bakımından oldukça meşhurdur
ve önemlidir. Ancak, önemli bir noktayı vurgulayalım: Birçok matematik
10 CHAPTER 2. TEMEL BİLGİLER

veya geometri kitabında Pisagor teoremi diye bu teoremin sadece gerek şart
kısmı, yani aşaǧıdaki notta belirttiǧimiz 1. kısmı alınmaktadır.
Teorem 2.3. (Pisagor Teoremi) ABC öklid düzleminde bir üçgen ise,
b = 90o ⇐⇒ a2 + b2 = c2 .
m(C) (2.1)

Not 2.2. Bu teorem sırayla gerek şart ve yeter şart dediǧimiz iki bölümden
oluşur:
b açısı dik ise a2 + b2 = c2 ’dir.
1. C

2. a2 + b2 = c2 ise C
b açısı diktir.

İspat:
(1) Gerek şart: (⇒) C
b açısı dik olan ABC üçgeni aşaǧıdaki gibi verilsin.

Şekil 2.6 Özdeş iki karenin alanından Pisagor baǧıntısı.

Yukarıdaki karelerin kenarları, verilen üçgenin dik kenarlarının toplamı olup,


bu karelerin alanlarının eşitliǧinden gereklilik ispatı tamamlanır. (Bu ispat
12. yy’da yaşamış Hint matematikçi Bhashkara’ya aittir. Başka ispatlar için,
ki farklı 367 ispatı olduǧu söyleniyor, Matematik Dünyası, cilt-1/sayı-3 ile
cilt-5/sayı-4’e bakabilirsiniz. Ayrıca, WEB’de bir arama motorunda türkçe
sayfalar için pisagor teoremi ve ingilizce sayfalar için pythagorean theorem
anahtar kelimelerini girerek ilgili güzel sayfalara ulaşabilirsiniz.)
(2) Yeter şart: (⇐) Bir ABC üçgeninde

|AB|2 = |CB|2 + |CA|2


2.2. SAYILAR 11

olsun. Şimdi, |P Q| = |CB|, |P R| = |CA| ve m(Pb) = 90o olacak şekilde bir


P QR dik üçgeni alalım. Yukarıdaki (1) ispatından dolayı
|P Q|2 + |P R|2 = |QR|2
olacaktır. Burada |P Q| = |CB| ve |P R| = |CA| deǧerleri yerine yazılırsa,
|CB|2 + |CA|2 = |QR|2
= |AB|2

ve buradan |AB| = |QR| bulunur. Bu ise bize, KKK eşlik teoreminden


dolayı, P QR ile CAB üçgenlerinin eş olduklarını, yani, C
b açısının dik açı
olduǧunu gösterir. 2
Sonuç √2.2. ABC dik üçgeninde a = 1 ve b = 2 olması durumunda c2 = 5
ve c = 5 olur. Bu ise rasyonel olmayan bir sayıdır.

Gerçekten, 5 rasyonel olsa, x ile y aralarında asal olmak üzere
√ x
5=
y
şeklinde yazılabilirdi. Bu durumda ise, iki tarafın karesi alınarak,
x2
5= 2
⇒ 5y 2 = x2
y
bulunur. Bu ise, x2 ’nin 5’e bölünebildiǧini, dolayısıyla da x’in 5’e bölünebil-
diǧini gösterir. Buradan x = 5k yazılıp yerine konursa,
5y 2 = 25k 2 ⇒ y 2 = 5k 2
olduǧunu, bu ise y’nin de 5’e bölünebildiǧini √ gösterir. Bu durum x ile y’nin
aralarında asal olması ile çelişir. Bu çelişki 5’in√rasyonel olması kabulünden
kaynaklandı, o halde kabulümüz yanlıştır, yani 5 rasyonel deǧildir.
Rasyonel olmayan bu şekildeki sayılara İrrasyonel sayılar diyecek ve Q∗ ile
göstereceǧiz. İrrasyonel sayılar köklü sayılardan ibaret deǧildir. e (doǧal
logaritma tabanı), π (bir çemberin çevresinin çapına oranı), gibi sayılar √ π
da irrasyoneldir. Bu sayıların deǧerleri seriler yardımıyla hesaplanır. 2
sayısının ne anlama geldiǧini bilmesek de bu sayı da irrasyoneldir. (İrrasyonel
sayılar için bir WEB arama motorunda ingilizce sayfalar için irrational num-
bers ve türkçe sayfalar için de irrasyonel sayılar ipuçlarını girip özellikle in-
gilizce deǧerli sayfalar bulabilirsiniz. Hemen belirtelim ki e sayısının irras-
yonel olduǧunu ispatlamak bile kolay iş deǧildir.)
12 CHAPTER 2. TEMEL BİLGİLER

Tanım 2.13. (Reel sayılar) Rasyonel sayılar ile irrasyonel sayıların birle-
şim kümesine Reel sayılar diyecek ve R ile göstereceǧiz.

Bu sayı kümesindeki işlemleri biliyorsunuz. Reel sayılar ile sayı doǧrusu


üzerindeki noktalar birebir eşlenebilir. Bundan dolayı reel sayı deyince sayı
doǧrusu aklımıza gelir. Bu yüzden reel sayılara doǧrusal nokta kümeleri de
denir.
Pisagor teoremini de kullanarak, kareköklü reel sayıları sayı doǧrusu üzerin-
deki noktalarla (uzunluklarla) eşleştirmek için aşaǧıdaki güzel şekilden yarar-
lanabiliriz.

Şekil 2.7 Kareköklü sayıların türeyişi.

Tanım 2.14. (Polinom denklem) a0 , a1 , a2 , · · · , an tam sayı olmak üzere,

an xn + an−1 xn−1 + an−2 xn−2 + · · · + a1 x1 + a0 = 0 (∗)

şeklindeki denkleme polinom denklem, n’ye denklemin derecesi, an 6= 0 sayı-


sına denklemin başkatsayısı denir.

Tanım 2.15. (Cebirsel sayılar) (∗) denklemini saǧlayan sayılara cebirsel


sayı denir. Biraz daha formal olmak gerekirse, (∗)’ın kökü olup n’den daha
küçük dereceli bir denklemin kökü olmayan sayıya n-inci basamaktan cebirsel
sayı denir. Cebirsel olmayan sayılara da transendent (transcendental) veya
aşkın sayılar denir.

Örnek 2.1. 2, 3 ve 2/5 sayıları cebirsel sayılardır, çünkü, bunlar sırasıyla
x2 − 2=0, x√ − 3=0 ve 5x − 2=0 polinom denklemlerinin kökleridir. Ancak,
π, e, ln3, 2 2 gibi sayılar cebirsel deǧildir.
√ √ √ √
Örnek 2.2. 2 √ + 3 sayısının cebirsel olduǧunu gösterelim. x = √2 + √3
ise x2 = 2 + 3 + 2 6 ve buradan x4 − 10x2 +1 = 0 bulunur. O halde, 2+ 3
sayısı 4-üncü dereceden bir cebirsel sayıdır.
2.2. SAYILAR 13

Not 2.3. Reel sayılar ile ilgilenen matematik dalı Reel Analiz’dir. Bizim reel
sayılara yaklaşımımız aksiyomatik deǧil, daha çok sezgisel (yani, önceki sayı
kümelerinin doǧal bir genişlemesi) olmaktadır. Reel sayıların aksiyomatik
yapısı için www.alinesin.org sayfasına bakılabilir.
Tanım 2.16. a, b ∈ R ve a < b olsun.

{x ∈ R : a < x < b}

kümesi bir açık aralık belirtir ve (a, b) ile gösterilir. Benzer olarak

{x ∈ R : a ≤ x ≤ b}

kümesi bir kapalı aralık belirtir ve [a, b] ile gösterilir. Bunun gibi, yarı açık
aralık da tanımlanabilir.
Tanım 2.17. A ⊂ R olsun. Eǧer A kümesinin her x elemanı için x ≥ a
olacak şekilde bir a sayısı varsa A kümesine alttan sınırlı küme; a sayısına da
A kümesinin bir alt sınırı denir. Benzer olarak üst sınır kavramı tanımlanır.
Hem alttan hem de üstten sınırlı kümeye kısaca sınırlı küme denir.
Örnek 2.3. (3, 7) açık aralıǧı aynı zamanda bir sayı kümesidir. Bu küme
hem alttan hem de üstten sınırlıdır. Çünkü, her x ∈ (3, 7) için m ve M reel
sayıları vardır öyle ki m ≤ x ≤ M ’dir. Burada, m = 3 ve M = 7’dir.
Aksiyom 2.1. Üstten sınırlı bir kümenin üst sınırları içinde bir en küçüǧü,
alttan sınırlı bir kümenin alt sınırları içinde bir en büyüǧü vardır.
Tanım 2.18. R’nin üstten sınırlı alt kümelerinden biri A olsun. A’nın üst
sınırlarının en küçüǧüne A’nın en küçük üst sınırı (eküs A) veya supremumu
(sup A) denir. Benzer olarak A’nın en büyük alt sınırı (ebas A veya inf A)
tanımlanır.
Örnek 2.4. A = { n1 : n ∈ N} kümesi için sup A=1, inf A=0, sup A∈ A fakat
inf A 6∈ A’dır.
Örnek 2.5. B = { 1r : r ∈ R ve r < 0} kümesi için sup B = 0 6∈ B ve inf B
yoktur.
Tanım 2.19. Bir A ⊂ R kümesinin en küçük üst sınırı bu kümenin elemanı
ise buna en büyük elemanı veya maksimumu denir ve maks A ile gösterilir.
Benzer olarak min A tanımlanır.
14 CHAPTER 2. TEMEL BİLGİLER

Örnek 2.6. A=[0, 1] ve B=[0, 1) için sup A=1, sup B=1, inf A=0, inf B=0,
maks A=1, min A=0, min B=0 olup maks B yoktur.

Sonuç 2.3. A ⊂ R bir aralık ve i=inf A, s=sup A olsun. i ve s sayılarının


şu özellikleri vardır:

1. Her x ∈ A için i ≤ x’tir.

2. Her x ∈ A için s ≥ x’tir.

3. Her δ > 0 için i + δ > x olacak şekilde en az bir x ∈ A vardır.

4. Her δ > 0 için x > s − δ olacak şekilde en az bir x ∈ A vardır.

Gerçekten, (1) ve (2) tanımdan yazılabilirken, (3) ve (4) belki o kadar açık
deǧildir. Eǧer A’nın hiç bir elemanı için i + δ > x olmasaydı, A’nın bütün
elemanları için i + δ ≤ x olacaktı. Bu ise ise i + δ sayısının bir alt sınır
olduǧunu ifade eder. Oysa bu sayı ebas olarak kabul ettiǧimiz i sayısından
da büyüktür, ki bu bir çelişkidir. O halde (3) doǧrudur. Benzer olarak (4)’ü
gösteriniz.
Şimdi önemli bir kavramı, yıǧılma noktası kavramını tanımlayalım. Ancak,
bu kavram uzaklık kavramına dayandıǧından, öncelikle uzaklıǧı ve buna baǧlı
olarak mutlak deǧer kavramını tanımlayacaǧız.

Tanım 2.20. (Uzaklık) a ile b reel sayıları arasındaki uzaklık |a − b| ile


gösterilir ve 
a − b, a ≥ b
|a − b| =
b − a, a < b
şeklinde tanımlanır. Örneǧin -3 ile 5 arasındaki uzaklık |−3−5|=5-(-3)=8’dir.

Tanım 2.21. (Mutlak deǧer) Bir a reel sayısının mutlak deǧeri onun 0
(sıfır)’a uzaklıǧı olarak tanımlanır ve |a| ile gösterilir. Buna göre,

a, a ≥ 0;
|a| =
−a, a < 0

olur.

Buna göre, her a, b ∈ R için şunlar yazılabilir:


2.2. SAYILAR 15

1. |a| > 0’dır. (Uzaklık pozitiftir.)



2. |a| = a2 ’dir. (Karekök pozitif deǧerlidir.)

3. −|a| ≤ a ≤ |a|’dır. (Gösteriniz.)

4. |a.b| = |a|.|b|’dir. (Gösteriniz.)

5. |a| ≤ x ise −x ≤ a ≤ x’dir.

Aşaǧıdaki teorem üçgen eşitsizliǧi olarak bilinir ve oldukça önemlidir.

Teorem 2.4. Her a, b ∈ R için ||a| − |b|| ≤ |a + b| ≤ |a| + |b|’dir.

İspat: Her a, b ∈ R için


(1)
(|a| − |b|)2 = |a|2 + |b|2 − 2|a|.|b|
= a2 + b2 − 2|a.b|
≤ a2 + b2 + 2ab
= (a + b)2
ve buradan ||a| − |b|| ≤ |a + b| elde edilir.
(2)
(a + b)2 = a2 + b2 + 2ab
= |a|2 + |b|2 + 2ab
≤ |a|2 + |b|2 + 2|a|.|b|
= (|a| + |b|)2
ve buradan |a + b| ≤ ||a| + |b|| = |a| + |b| bulunur.2

Tanım 2.22. (Komşuluk) δ > 0 için tanımlanan

K = {x ∈ R : |x − a| < δ}

kümesine a’nın δ-komşuluǧu, K\{a} kümesine de a’nın delinmiş δ-komşuluǧu


denir.

Tanım 2.23. (Yıǧılma noktası) A ⊂ R ve a ∈ R olsun. a noktasının


her δ-komşuluǧunda A kümesinin a’dan farklı en az bir elemanı varsa, bu a
noktasına A kümesinin bir yıǧılma noktası denir.
16 CHAPTER 2. TEMEL BİLGİLER

Örnek 2.7. A = {x : x = n1 , n ∈ N} kümesinin yıǧılma noktası 0’dır.


Çünkü, (0 − δ, 0 + δ) = (−δ, δ) aralıǧı A kümesinin birçok elemanını içerir.
Örneǧin, δ = n10 alınırsa, n0 = 1δ olup, her n > n0 için n1 ∈ (−δ, δ) olur. Bu
yıǧılma noktasının A’nın elemanı olmadıǧı açıktır.

Örnek 2.8. B = [0, 1] kümesinin her elemanı bir yıǧılma noktasıdır. Bu


kümenin kendine ait olmayan yıǧılma noktası yoktur.

Örnek 2.9. N kümesinin hiç yıǧılma noktası yoktur.

Teorem 2.5. Bir kümenin supremumu (veya infimumu) kümeye ait deǧilse,
kümenin bir yıǧılma noktasıdır.

İspat: A bir küme ve sup A=s olsun. s 6∈ A olduǧundan, her x ∈ A için


x < s’dir. Yukarıdaki sonuç 1.3’ten dolayı her δ > 0 için A’nın en az bir x
elemanı vardır ki x > s − δ yazılabilir. O halde,

s−δ <x<s+δ

eşitsizliǧi saǧlanır. A kümesinin s’nin δ komşuluǧunda s’den farklı en az bir


x elemanı bulunduǧundan s kümenin yıǧılma noktasıdır. (İnfimum için de
benzer ispatı yapınız.)

Tanım 2.24. Bir A kümesinin en saǧda olan yıǧılma noktasına üst limit veya
limit superiyor denir ve limsup A veya lim A ile gösterilir. (Benzer biçimde
lim A’yı siz tanımlayınız.)

Tanım 2.25. (Tam deǧer) Bir a reel sayısından büyük olmayan tam sayıla-
rın en büyüǧüne a’nın tam deǧeri denir ve [|a|] ile gösterilir.

Örnek 2.10. [| 3|]=1, [| − 3, 01|]=-4, [|π|]=3’tür.

Sonuç 2.4. Görülüyor ki her a reel sayısı onun tam kısmı [|a|] ile 0≤ t< 1
koşulunu saǧlayan kesir kısmı t’nin toplamı olarak yazılabilir. Yani,

a = [|a|] + t (0 ≤ t < 1)

yazılabilir. Gerçekten 2=1+0,42...; -2,5=-3+0,5 ve −π=-4+0,85...’dir.

Şimdi, tam deǧerle ilgili problemlerin çözüm metodunu görmek açısından bir
problem çözelim.
2.2. SAYILAR 17

Problem 2.1. Her a, b ∈ R için [|a + b|] ≥ [|a|] + [|b|] olduǧunu gösteriniz.

Çözüm:
a = [|a|] + t1 (0 ≤ t1 < 1)
b = [|b|] + t2 (0 ≤ t2 < 1)
a+b = [|a|] + [|b|] + t1 + t2 (0 ≤ t1 + t2 < 2)
olduǧundan, karşımıza iki durum çıkmaktadır:
(1) 0 ≤ t1 + t2 < 1 halinde [|a + b|] = [|a|] + [|b|] olur (eşitlik hali).
(2) 1 ≤ t1 + t2 < 2 halinde [|a + b|] = [|a|] + [|b|] + 1 > [|a|] + [|b|] olur
(büyüklük hali). Dolayısıyla eşitsizlik doǧrudur.

ALIŞTIRMALAR (Sayılar)
1. n bir tam sayı olmak üzere x=2n şeklinde yazılabilen tam sayıya çift
sayıdır denir. Çift sayının bir eksiǧine de tek sayı denir. Buna göre, y
tek sayı iken y 2 − 1 sayısının 8’in katı olduǧunu gösteriniz.
2. Tek tamsayıların karesinin de tek olduǧunu gösteriniz.
3. a ve b aynı işaretli reel sayıları için ab + ab ≥ 2 olacaǧını gösteriniz. a
ve b zıt işaretli ise nasıl bir eşitsizlik yazılabilir? Yazdıǧınız eşitsizliǧi
kanıtlayınız.
a+b
4. a < b olmak üzere a, b ∈ R için a < 2
< b olduǧunu gösteriniz.
5. Herhangi iki rasyonel sayı arasında bir rasyonel sayı olduǧunu gösterin.
6. Eǧer a2 + b2 = 1 ve c2 + d2 = 1 ise ac + bd ≤ 1 olduǧunu gösteriniz.
x y z
7. x, y, z’nin hangi deǧerleri için (z−x)(x−y)
+ (x−y)(y−z) + (y−z)(z−x) =0 oldu-
ǧunu belirleyiniz.
8. 0’dan farklı her a ∈ R için |a + a1 | ≥ 2 olduǧunu gösteriniz.
√ √
9. 2 ile 3 2 sayılarının irrasyonel olduklarını gösteriniz.
10. Her x, y, z ∈ R için x2 + y 2 + z 2 ≥ xy + xz + yz olduǧunu gösteriniz.
a
11. a rasyonel ve x irrasyonel ise a+x, a.x, x
sayılarının irrasyonel olduǧunu
gösteriniz.
18 CHAPTER 2. TEMEL BİLGİLER
√ √ √ √ √
12. √5+√2 ve 2 + 3 + 5 sayılarının kaçıncı basamaktan cebirsel oldu-
5− 2
ǧunu belirleyiniz.

13. Her rasyonel sayının cebirsel olduǧunu gösteriniz. Rasyonel sayılar


kaçıncı basamaktan cebirseldir? Belirleyiniz.

14. Sadece iki farklı sayma sayısına bölünebilen sayılara asal sayı denir.
Her n ∈ N için p(n) = n2 + n + 41 sayısı asal mıdır? p(n) hangi n’ler
için asaldır?

15. |x2 − 5x + 6| > x2 − 5x + 6 eşitsizliǧini çözünüz.

16. Her x ∈ R ve m ∈ Z için [|x + m|]=[|x|] + m olduǧunu gösteriniz.

17. [|2x|]=2[|x|] denklemini çözünüz.

18. x−1
 
x
= 1 denklemini çözünüz.

2.3 TÜMEVARIMLA İSPAT YÖNTEMİ


Tam sayılar ile ilgili önermelerin doǧruluǧunu göstermeye yarayan ispat me-
totlarından biridir. Bir t tam sayısından itibaren her tam sayı için geçerli
olan genellemeleri ispatlamaya yarar. Bu metot şu basit teoreme dayanır:
Teorem 2.6. D ⊂ N ve 0 ≤ k olsun. D kümesi
1. 0 ∈ D

2. k ∈ D ⇒ (k + 1) ∈ D
koşullarını gerçeklerse D = N’dir.
Not 2.4. Gerçekte bu teorem bize, ”0’dan itibaren her sayının ardışıǧı D’nin
elemanı ise D kümesi 0’dan itibaren 1 artarak devam eden bir küme, yani
doǧal sayılar kümesi olmalıdır” der.

İspat: Varsayalım ki D 6= N’dir. Bu durumda en az bir m ∈ N için


m 6∈ D’dir. Bu durumda (m − 1) 6∈ D olmalıdır, aksi halde (2)’den dolayı
( m − 1 ) + 1 ∈ D, yani m ∈ D olurdu. Benzer şekilde m − 2, m − 3, ...,
2, 1, 0 sayıları da D’ye ait olamazlar. Oysa, (1)’den dolayı 0 ∈ D olduǧunu
biliyoruz. Bu ise çelişkidir. O halde kabulümüz yanlıştır, D = N olmalıdır.
2.3. TÜMEVARIMLA İSPAT YÖNTEMİ 19

Sonuç 2.5.

(1) P (n) doǧal sayıların her biri için yazılan bir önerme ve D kümesi de
bu önermeyi gerçekleyen doǧal sayıların bir kümesi olsun. Yani,

D = {n : P (n) doǧru, n ∈ N}

olsun. Eǧer, 0 ∈ D ve k ∈ D iken (k + 1) ∈ D ise, D = N’dir. Yani, P (n)


önermesi her n ∈ N için doǧrudur.
(2) P (n) önermesinin 0’dan başlama zorunluluǧu yoktur, herhangi bir
t ∈ Z sayısından itibaren her tam sayı için de gerçeklenebilir.
O halde, bir P (n) önermesinin bir t ∈ Z tam sayısından itibaren her tam sayı
için doǧruluǧunu göstermek için n=t için doǧruluǧunu göstermek ve n=k için
doǧru iken n=k + 1 için doǧru olduǧunu göstermek gerekiyor. Bu metoda
Tümevarım metodu adı verilir.

Örnek 2.11. Her n ∈ N için 9n+1 − 2n+1 sayısının 7 ile bölünebileceǧini


gösterelim:
n = 0 için, 90+1 − 20+1 = 7 sayısı 7 ile bölünebilir.
n = k için, 9k+1 − 2k+1 = 7m olsun, yani, 7 ile bölünebilsin. Buradan,
n = k + 1 için de önermenin doǧruluǧunu görelim:
9(k+1)+1 − 2(k+1)+1 = 9.9k+1 − 2.2k+1
= (7 + 2).9k+1 − 2.2k+1
= 7.9k+1 + 2(9k+1 − 2k+1 )
= 7.9k+1 + 2.7m
= 7p
olduǧundan, istenen görülmüş olur.
20 CHAPTER 2. TEMEL BİLGİLER

ALIŞTIRMALAR (Tümevarımla ispat yöntemi)

1. Her n ∈ N+ (veya uygun n’ler) için aşaǧıdakileri gösteriniz.

(a) 1 + 3 + 5 + · · · + (2n − 1) = n2
(b) 12 + 22 + 32 + · · · + n2 = n(n + 1)(2n + 1)/6
11 11 1 1 3n2 −n−2
(c) 13
+ 24
+ ··· + n−1 n+1
= 4n(n+1)

(d) n! > 2n
(e) 13 + 23 + · · · + n3 = (1 + 2 + · · · + n)2

2. Her n doǧal sayısı için

(a) n(2n + 1)(7n + 1) sayısının 6 ile


(b) 12n + 10 sayısının 11 ile
(c) 174n+1 + 34n+1 sayısının 5 ile
(d) 32n+1 + 2n+2 sayısının 7 ile

bölünebileceǧini gösteriniz.

3. Uygun olan her n doǧal sayısı için 3n + 4n ≤ 5n olduǧunu gösteriniz.


1−rn+1
4. Her n doǧal sayısı için 1 + r + r2 + r3 + · · · + rn = 1−r
olduǧunu
gösteriniz.

2.4 FONKSİYONLAR
Tanım 2.26. (Kartezyen çarpım) A ve B iki küme olsun.

A × B := {(a, b) : a ∈ A, b ∈ B}

kümesine A ile B’nin kartezyen çarpımı veya dik çarpımı denir.

Örnek 2.12. A={x, y} ve B={a, b, c} kümeleri için

A × B = {(x, a), (x, b), (x, c), (y, a), (y, b), (y, c)}

olup, A × B 6= B × A olduǧu görülmektedir.


2.4. FONKSİYONLAR 21

Tanım 2.27. (Baǧıntı) A × B’nin herhangi bir alt kümesine A’dan B’ye
bir baǧıntı denir.
Tanım 2.28. (Fonksiyon) A’dan B’ye bir f baǧıntısı şu özelliklere sahipse,
f ’ye A’dan B’ye bir fonksiyon denir.
1. Her x ∈ A için (x, y) ∈ f olacak şekilde bir y ∈ B vardır.

2. (x, y) ∈ f ve (x, z) ∈ f ise y = z’dir.


Not 2.5. İlk özellik A’nın her elemanının mutlaka bir karşılıǧının olacaǧını,
ikinci özellik ise bu karşılıǧın tek olacaǧını söyler. Buna göre, fonksiyon
tanımını daha kısa yapabiliriz:
Tanım 2.29. (Fonksiyon) A’dan B’ye bir f baǧıntısı, A’nın her bir ele-
manını B’nin bir ve yalnız bir elemanına eşliyorsa f ’ye fonksiyon denir ve

f : A 7→ B, f : x 7→ f (x) = y

ile gösterilir. A kümesine tanım kümesi, B kümesine deǧer kümesi ve f (A)


kümesine de A’nın görüntü kümesi denir. Bu kümeleri belirlemek bazen
oldukça zordur.
Örnek 2.13. f : R 7→ R, f : x 7→ f (x) = x2 +1 = y bir fonksiyondur, çünkü,
her reel sayıyı karesinin 1 fazlasına eşler. Yani, her reel sayının sadece bir
tane karesinin 1 fazlası vardır.

Şekil 2.8 y = x2 + 1 grafiǧi.

Tanıma göre, tanım kümesi (tanım bölgesi de denir) R, deǧer kümesi R ve


görüntü kümesi de [1, ∞) yarı açık aralıǧıdır. Hemen belirtelim ki, f (x)
ile f sıkça karıştırılan iki kavramdır. Bunlardan f (x) görüntü kümesinin
bir elemanı; f ise matematiksel bir varlık olan fonksiyondur. Ancak, biz
22 CHAPTER 2. TEMEL BİLGİLER

de geleneǧi bozmayacaǧız: Sadelik adına, bir fonksiyon yazarken ya sadece


görüntü kümesini f (x)=x2 + 1 şeklinde verecek veya y=x2 + 1 şeklinde ordi-
natını belirteceǧiz.
Tanım 2.30. (Kökler) Bir fonksiyonun kökleri diye f (x)=0 eşitliǧini saǧla-
yan x’lerin kümesine denir.
Tanım 2.31. (Eşit fonksiyonlar) Aynı tanım kümelerine sahip f ve g
fonksiyonları, tanımlı oldukları her x için f (x)=g(x) eşitliǧini saǧlıyorsa,
bunlara eşit fonksiyonlar denir ve f =g ile gösterilir.
Tanım 2.32. (Fonksiyonlarda işlemler) f ve g fonksiyonlarına ilişkin
işlemler şöyle tanımlanır:
1. (f ∓ g)(x) := f (x) ∓ g(x)

2. (f.g)(x) := f (x).g(x)

3. (f /g)(x) := f (x)/g(x)

4. (c.f )(x) := c.f (x)


Örnek 2.14. f ve g fonksiyonları f (x) = x2 + x ve g(x) = x + 1 ise,

(f + g)(x) = (x + 1)2 ve (f /g)(x) = x

olur.
Tanım 2.33. (Sabit fonksiyon) Tanım kümesinin her elemanını deǧer
kümesindeki bir tek elemana eşleyen fonksiyondur. f (x)=a (a sabit) şeklin-
dedir.
Tanım 2.34. (Örten fonksiyon) Deǧer kümesinde boşta elemanı kalmayan
fonksiyondur. f : A 7→ B örten fonksiyonu için f (A) = B’dir. Örten olmayan
fonksiyona içine fonksiyon denir.
Tanım 2.35. (Birebir fonksiyon) Deǧer kümesinden bir elemana tanım
kümesinden sadece bir elemanı eşleyen fonksiyondur. Yani, tanım kümesinin
x1 , x2 elemanları için

f (x1 ) = f (x2 ) ⇒ x1 = x2

oluyorsa, f fonksiyonu birebirdir.


2.4. FONKSİYONLAR 23

Ödev 2.2. Grafikleriyle verilen fonksiyonların örten, içine veya birebir oldu-
ǧunu nasıl belirleyebileceǧimizi düşününüz.

Tanım 2.36. (İki fonksiyonun bileşkesi) f : A 7→ B ve g : B 7→ C


fonksiyonları verilmiş olsun. Bu durumda, g fonksiyonu f (A)’daki her bir
f (x) elemanını C kümesinin bir g(f (x)) elemanına dönüştürür. Böylece,
A’nın her bir x elemanını C’nin bir z=g(f (x)) elemanına dönüştüren yeni
bir fonksiyon elde edilir. Bu fonksiyona f ve g fonksiyonlarının bileşkesi denir
ve g ◦ f ile gösterilir.

Şekil 2.9 g ◦ f fonksiyonu.

Örnek 2.15. f (x)=x2 ve g(x)=x + 2 için

(g ◦ f )(x) = g(f (x)) = g(x2 ) = x2 + 2

(f ◦ g)(x) = f (g(x)) = f (x + 2) = (x + 2)2


olup, genel olarak g ◦ f 6= f ◦ g’dir.

Tanım 2.37. (Bir fonksiyonun tersi) Birebir ve örten bir f fonksiyonu


verildiǧinde, (f ◦ g)(x) = (g ◦ f )(x) = x eşitliǧini saǧlayan g fonksiyonuna
f ’nin tersi denir ve f −1 ile gösterilir.

Şekil 2.10 g = f −1 fonksiyonu.



Örnek 2.16. f : R+ 7→ R+ , f (x) = x eşitliǧi ile verilen fonksiyonun birebir
ve örten olduǧunu gösterip f −1 tersini bulalım.
24 CHAPTER 2. TEMEL BİLGİLER

√ √
f fonksiyonu birbirdir: Gerçekten, f (x1 ) = f (x2 ) ise x1 = x2 olup, iki
tarafın karesi alınırsa x1 = x2 bulunur.

f fonksiyonu örtendir: Gerçekten, deǧer kümesinden her y ∈ R+ için y = x
olacak şekilde tanım kümesinde bir x ∈ R+ vardır.
f −1 fonksiyonu kare almadır: Gerçekten,
√ √
(f −1 ◦ f )(x) = x ⇒ f −1 (f (x)) = x ⇒ f −1 ( x) = x ⇒ f −1 ( u2 ) = u2

ve buradan f −1 (u) = u2 ; yani, f −1 (x) = x2 bulunur.


Şekil 2.11 f (x) = x ve tersi.

Görülüyor ki, y = x ve y = x2 fonksiyonları birbirinin tersidir ve y = x
doǧrusuna göre simetriktirler.

Tanım 2.38. (Ters görüntü) f : A 7→ B, C ⊂ B için,

f −1 (C) = {x ∈ A : f (x) ∈ C}

kümesine C’nin ters görüntüsü diyeceǧiz.

Örnek 2.17. f : R 7→ R, f (x) = 2x − 1 olsun. A = {−2, −1, 0, 1, 2} için


f (A) = {−5, −3, −1, 1, 3} ve f −1 (A) = {− 12 , 0, 12 , 1, 32 } olur.

Şimdi aşaǧıdaki önerme, fonksiyonlar ile kümelere ilişkin bilgilerimizi çok


güzel birleştirmektedir.

Önerme 2.1. f : X 7→ Y bir fonksiyon ve A, B⊂X olsun.

1. A ⊂ B⇒f (A) ⊂ f (B)

2. f (A ∩ B) ⊂ f (A) ∩ f (B)
2.4. FONKSİYONLAR 25

3. f (A ∪ B) = f (A) ∪ f (B)
4. f (A \ B) ⊂ f (A)

İspat:
1. A⊂B olduǧundan, x∈A⇒x∈B ve her x∈A için f (x)∈f (B) demektir.
Bu durum her x∈A için geçerli olduǧundan f (A)⊂f (B)’dir.
2. (1)’den, A∩B⊂A ve A∩B⊂B olması f (A∪B)⊂f (A) ve f (A∪B)⊂f (B)
olmasını gerektirdiǧinden f (A∩B)⊂f (A)∩f (B) bulunur. Aşaǧıdaki ör-
nek bu içermenin eşitliǧe dönüşemeyeceǧini göstermektedir.
3. İki kümenin eşit olması onların karşlıklı birbirini içermesi demek oldu-
ǧundan f (A∪B) ⊂ f (A)∪f (B) ve f (A)∪f (B) ⊂ f (A∪B) içermelerini
göstermeliyiz.
(a) y∈f (A∪B) olsun. Buradan, bir x∈A∪B vardır öyle ki y=f (x)’dir.
Bu x, ya A’nın veya B’nin elemanı olacaǧından, f (x)∈f (A) veya
f (x)∈f (B) olur. ∴ f (x)∈f (A)∪f (B), ve f (A∪B)⊂f (A)∪f (B)
bulunur.
(b) f (A)⊂f (A)∪f (B), f (B)⊂f (A)∪f (B) olup f (A)∪f (B)⊂f (A∪B)
bulunur.
Böylece, (a) ve (b)’den, f (A ∪ B) = f (A) ∪ f (B) elde edilir.
4. f (A\B)⊂f (A) içermesi ise A\B⊂ A ile (1)’in sonucudur. 2
Örnek 2.18. f : R 7→ R+ , f (x)=|x| olsun. A={−2, −1, 0}, B={2, 1, 0}
kümeleri için f (A∩B) ve f (A)∩f (B) kümelerini bulalım. f (A∩B)={0}’dir.
Öte yandan, f (A)={2, 1, 0}, f (B)={2, 1, 0} olup f (A)∩f (B)={2, 1, 0}’dır.
Görülüyor ki bu kümeler eşit deǧildir.
Tanım 2.39. (Denk kümeler) A ve B kümeleri arasında birebir ve örten
bir fonksiyon tanımlanabiliyorsa (yani birebir bir eşleme kurulabiliyorsa),
bunlara denk veya aynı kardinale sahip kümeler denir ve A ∼ B ile gösterilir.
Örnek 2.19. N doǧal sayılar kümesi ile çift doǧal sayılar Ç={0, 2, 4, · · · }
kümesi denktir. Çünkü, bu sayı kümeleri arasında tanımlanan
f : N 7→ Ç, f (x) = 2x
fonksiyonu birebir ve örtendir. Bu, bize sonsuz kümelere ilişkin ilginç bir
gerçeǧi söyler; doǧal sayılar kadar pozitif çift sayı vardır.
26 CHAPTER 2. TEMEL BİLGİLER

Tanım 2.40. (Sayılabilir küme) Sayma sayılarının tümü veya bir alt
kümesi ile birebir eşlenebilen kümeye sayılabilir küme denir.
Örnek 2.20. Z tam sayılar kümesi sayılabilirdir, çünkü, f : Z 7→ N+

−2x, x < 0
f (x) =
2x + 1, x ≥ 0

biçimindeki fonksiyon birebir ve örtendir.

Şekil 2.12 Z’nin sayılması.


Problem 2.2. Q rasyonel sayılar kümesinin sayılabilir olduǧunu görelim.
Aşaǧıdaki şekilde, rasyonel sayıları şöyle saymaktayız: Bir adım saǧa, sonra
diyagonal olarak sola aşaǧıya gidebileceǧin kadar, sonra bir adım aşaǧı, sonra
diyagonal olarak saǧa yukarıya gidebileceǧin kadar, ve böyle devam et; tekrar
eden kesirlerin üstünden atla.

Şekil 2.13 Q’nun sayılması.


Tanım 2.41. f : A 7→ R, x1 , x2 ∈ A ⊂ R verilsin. Bu durumda, x1 < x2 için
1. f (x1 ) < f (x2 ) ise f ’ye artan,
2.4. FONKSİYONLAR 27

2. f (x1 ) > f (x2 ) ise f ’ye azalan,

3. f (x1 ) ≤ f (x2 ) ise f ’ye azalmayan,

4. f (x1 ) ≥ f (x2 ) ise f ’ye artmayan

fonksiyon denir.

Şekil 2.14 Artan ve artmayan fonksiyonlar.


Örnek 2.21. y=x2 + 1 fonksiyonu x≤ 0 için azalan, x≤ 0 için artandır.
Gerçekten, x1 , x2 ≤ 0 olsun.

x1 < x2 ⇒ x21 > x22 ve x21 + 1 > x22 + 1 ⇒ f (x1 ) > f (x2 )
olup, bu ise yukarıdaki tanıma göre, f ’nin azalan olduǧu anlamına gelir.
Benzer olarak, x ≥ 0 için f ’nin artan olduǧu gösterilebilir.

Şekil 2.15 y = x2 + 1’in artan ve azalan olduǧu bölgeler.


Tanım 2.42. (Simetrik küme) Her x∈A için −x∈A ise A’ya simetriktir
denir.
Tanım 2.43. (Tek ve çift fonksiyonlar) Simetrik bir A kümesi üzerinde
tanımlanan bir f fonksiyonu için f (−x)=f (x) ise, f ’ye çift; f (−x)=−f (x)
ise, f ’ye tek fonksiyon denir.
28 CHAPTER 2. TEMEL BİLGİLER

Örnek 2.22. f (x)=x2 , g(x)=x3 , h(x)=x + 1, k(x)=5, l(x)=0 fonksiyonları


sırasıyla çift, tek, ne çift ne tek, çift, hem çift hem tektir.
Tanım 2.44. (Periyot) Her x için f (x + T )=f (x) olacak şekilde T pozitif
reel sayısı varsa, f ’ye periyodiktir, T ’ye de f ’nin periyodudur denir. Varsa,
periyotların en küçüǧüne f ’nin esas periyodu denir.
Örnek 2.23. f (x)=x − [|x|] fonksiyonunun periyodunu bulup grafiǧini çize-
lim. Her T ∈Z+ için
f (x + T ) = x + T − [|x + T |]
= x + T − [|x|] − T
= x − [|x|]
= f (x)
olduǧundan f ’nin periyodu herhangi bir pozitif tamsayıdır, ancak esas peri-
yodu 1’dir. [−2, +2] aralıǧındaki grafiǧini çizmek için tanım aralıǧını alt
aralıklara bölmeliyiz. Yani,

x ∈ [−2, −1) ⇒ [|x|] = −2 ⇒ x − [|x|] = x + 2

x ∈ [−1, 0) ⇒ [|x|] = −1 ⇒ x − [|x|] = x + 1


x ∈ [0, 1) ⇒ [|x|] = 0 ⇒ x − [|x|] = x
x ∈ [1, 2) ⇒ [|x|] = 1 ⇒ x − [|x|] = x − 1
x = 2 ⇒ [|x|] = 2 ⇒ x − [|x|] = 0
olur. Buna göre, grafik aşaǧıdaki gibidir.

Şekil 2.16 f (x)=x − [|x|] fonksiyonunun grafiǧi.

Şimdi, bazı özel fonksiyonların grafiklerini nasıl çizebileceǧimizi örnekler üze-


rinde görelim.
Örnek 2.24. f (x)=−x2 + 3x + 4 ise
2.4. FONKSİYONLAR 29

1. g(x)=|f (x)|
2. h(x)=f (|x|)
3. k(x)=|f (|x|)|
ile verilen g, h ve k’nın grafikleri aşaǧıdaki gibidir. g’yi çizmek için, f (x)’ler-
den x ekseni altında olanların x eksenine göre simetrikleri alınmıştır. h’yi
çizmek için, f (x)’lerden y ekseni solunda olanların y eksenine göre simetrik-
leri alınmıştır. k’yı çizmek için, f (|x|)’lerden x ekseni altında olanların
x eksenine göre simetrikleri alınmıştır. Benzer olarak, |y|=f (x)’i çizmek
için, y=f (x)’in grafiǧinde x ekseni altındaki bölüm atılır. Yine, |y|=f (|x|)’i
çizmek için, |y|=f (x)’in grafiǧi çizilir ve y ekseninin saǧında kalan bölümün
simetriǧi alınır.

Şekil 2.17 y=f (x) verildiǧinde çizilebilecek bazı grafikler.


Tanım 2.45. f : A ⊂ R 7→ R verildiǧinde,

 1, f (x) > 0
sgn[f (x)] := 0, f (x) = 0
−1, f (x) < 0

fonksiyonuna f ’nin işaret fonksiyonu denir.


Örnek 2.25. f (x)=−x2 + 3x + 4 ile verilen fonksiyonun işaret fonksiyonu
için tablo ve buna baǧlı olarak işaret fonksiyonunu oluşturalım:

x −∞ -1 4 +∞
f (x) −∞ – 0 + 0 – +∞
sgn[f (x)] -1 0 +1 0 -1

 −1, x < −1, x > 4
sgn[f (x)] = 0, x = −1, x = 4
1, −1 < x < 4

Bunun grafiǧi ise aşaǧıdaki gibidir.


30 CHAPTER 2. TEMEL BİLGİLER

Şekil 2.18 y=sgn[−x2 + 3x + 4]’ün grafiǧi.

Tanım 2.46. Bir fonksiyonun kuralı y = f (x) olarak verilmişse bu fonksiy-


ona açık fonksiyon denir. Eǧer x ile f (x) arasında F (x, f (x)) = 0 şeklinde
bir baǧıntı verilmişse, f (x) kapalı olarak verilmiştir. Bu durumda f ’ye kapalı
verilmiş fonksiyon denir.

ALIŞTIRMALAR (Fonksiyonlar)

1. |2 − x| ne zaman 2 − x’e, ne zaman x − 2’ye eşittir?

2. Fonksiyonların eşitliǧi tanımını kullanarak, aşaǧıdakilerden hangileri-


2
nin f (x)= x+2x
x
’e eşit olduǧunu belirleyiniz.
2 3
(a) g(x) = 1 + 2x (b) h(x) = x +2x
3 √ x2
(c) l(x) = (1+2x)(x+x
x(1+x2 )
)
(d) k(x) = 1 + 4x + 4x2


3. f (x)=|x|, g(x)= x2 ve h(x)=x.sgn(x) fonksiyonları eşit midir?

4. Aşaǧıdaki fonksiyonların en geniş tanım ve görüntü kümelerini belir-


leyiniz.
2
(a) f (x) = x2 (b) g(x) = x+2
x−2
x
(c) h(x) = 1+x 2
1 1√ x
p
(d) m(x) = 1+x2 (e) l(x) = 1+ x (f ) k(x) = 1+x

5. Aşaǧıdaki fonksiyonların grafiklerini [−2, +2] aralıǧında çiziniz.

(a) f (x)= [|x|]


x
, (x 6= 0)
(b) g(x)=[|x|] + [| − x|]
x
(c) h(x)= [|x|] , ([|x|] 6= 0)
2.4. FONKSİYONLAR 31

6. f (x)=|x| ve g(x)=f (x−2) fonksiyonlarının grafiǧini çizip karşılaştırınız.



7. |x| ≤ 1 için f (x)= 1 − x2 olduǧuna göre, aşaǧıdakilerin her birinin
tanım ve görüntü kümelerini bulup grafiklerini çiziniz.

(a) g(x) = 12 f (x) (b) h(x) = f (2x)


(c) k(x) = −f (−x) (d) l(x) = 2f (3x)

8. Tümevarım yöntemiyle |a1 +a2 +· · ·+an |≤|a1 |+|a2 |+· · ·+|an | olduǧunu
ve bundan yararlanarak |a1 +a2 +· · ·+an |≥|a1 |−|a2 |−· · ·−|an | olduǧunu
gösteriniz.

9. Aşaǧıdaki her f ifadesi için, y=f (x), y=|f (x)|, y=f (|x|), y=|f (|x|)| ve
son olarak y= f (x)+|f
2
(x)|
’nin grafiǧini çiziniz.

(a) f (x) = (x − 1)(x + 2) (b) f (x) = x2


(c) f (x) = −x2 (d) f (x) = 2 − x2

10. Tanım kümeleri aşaǧıdaki kümeler olan fonksiyonlar bulunuz.

(a) A = {x : x ≥ 0, x ∈ R} (b) R (c) C = {x : |x| > 4, x ∈ R}


(d) D = {x : |x| ≥ 4, x ∈ R} (e) ∅ (f ) (−1, 2]

11. f (x)=|x + 2| + 2 ile verilen fonksiyonun grafiǧini çiziniz.

12. (∗) Cebirsel sayıların sayılabilir olduǧunu nasıl gösterebileceǧinizi araş-


tırınız.

13. (∗) Reel sayıların sayılamaz olduǧunu nasıl gösterebileceǧinizi araştırı-


nız.

14. f : N 7→ N, f (n)=n!, f (0)=1 ile tanımlı fonksiyona faktöryel fonksi-


yonu diyelim. Bu fonksiyonun artan ve birebir olduǧunu ancak örten
olmadıǧını gösteriniz.

15. f ile g tek fonksiyonlar ise f +g’nin tek, f.g’nin çift olacaǧını gösteriniz.

16. Simetrik bir küme üzerinde tanımlanan her f fonksiyonunun biri tek
diǧeri çift iki fonksiyonun toplamı olarak yazılabileceǧini gösteriniz.
32 CHAPTER 2. TEMEL BİLGİLER

2.5 LOGARİTMA FONKSİYONU


Tanım 2.47. a ∈ R+ ve a 6= 1 olmak üzere,

f : R → R+ , f (x) = ax

şeklinde tanımlanan fonksiyona üstel fonksiyon denir.

Özellikleri:
(1) (a) a > 1 için monoton artandır. Grafiǧi aşaǧıdadır.

Şekil 2.19 a > 1 için y=ax ’in grafiǧi.

(b) 0 < a < 1 için monoton azalandır. Grafiǧi şöyledir:

Şekil 2.20 0 < a < 1 için y=ax ’in grafiǧi.

1
(2) f (0)=1, f (1)=a, f (x + t)=f (x) · f (t), f (−x)= f (x) ’tir.

Kolayca görülebilir ki f (x)=ax (a>0) fonksiyonu birebir ve örtendir. O halde


bir tersi vardır ve bu tersi aşaǧda tanımlıyoruz.

Tanım 2.48. a > 0 için, f : R → R+ , f (x) = ax ’in tersi f −1 : R+ → R


şeklinde olup fonksiyon kuralı özel olarak f −1 (x) = loga x ile gösterilir ve
logaritma a tabanına göre x diye okunur.
2.5. LOGARİTMA FONKSİYONU 33

Dolayısıyla x > 0 için y = loga x ⇔ x = ay denkliǧi yazılabilir.


Özellikleri:
(1) Aşaǧıdaki özellikler gösterilebilir.

(a) loga a = 1 (b) loga 1 = 0 (c) loga (x · y) = loga x + loga y


(d) loga x = n · loga x (e) loga x = log1
n
a
(f ) loga xy = loga x − loga y
x

(2) y=loga x’in grafiǧi y=ax ’in grafiǧinin y=x’e göre simetriǧi olacaǧın-
dan aşaǧıdaki grafikler elde edilir.
(a) a > 1 durumu:

Şekil 2.21 a > 1 için y=loga x’in grafiǧi.

(b) 0 < a < 1 durumu:

Şekil 2.22 0 < a < 1 için y=loga x’in grafiǧi.

Not 2.6. Pratikte en çok kullanılan logaritma doǧal logaritma denilen e


tabanına göre logaritmadır. Bu e sayısı limit konusunda göreceǧimiz ve
deǧeri e=2.718281828459045 · · · olan bir sayıdır. loge x yerine ln x yazarız.
Dolayısıyla aşaǧıdaki eşitlik doǧrudur.

y = ln x ⇐⇒ ey = x
34 CHAPTER 2. TEMEL BİLGİLER

ALIŞTIRMALAR (Logaritma)
1. Logaritmanın (a)-(f) özelliklerini gerçekleyiniz.
8
2. log 3 27
= x ise, x deǧerini hesaplayınız.
2

3. log10 x = 3 log10 a + 2 log10 b ise x deǧerini hesaplayınız.


4. log10 x + log10 y = 1 ve x4 + y 4 = 641 ise, x ile y nedir?
5. log10 2 = 0.3010 ve log10 3 = 0.4771 ise log10 6, log10 0.72 ve log10 1200
deǧerlerini hesaplayınız.
6. loga b = loga c · logc b olduǧunu gösteriniz.

2.6 TRİGONOMETRİK FONKSİYONLAR


Tanım 2.49. (Açı) Başlangıç noktaları aynı iki ışının birleşim kümesine açı
denir.

Şekil 2.23 AOB açısı ve düzlemde ayırdıǧı bölgeler.

Bir açı, bulunduǧu düzlemi iki bölgeye ayırır. Bunlara açısal bölgeler denir.
İsteriz ki iki açısal bölgeyi karşılaştırabilelim. Bunun için bir ölçme aracı ve
birimi geliştireceǧiz. Bu öçme aracı yardımıyla açısal bölgeleri ve dolayısıyla
da açıları ölçüp karşılaştırabileceǧiz.
Bir çemberin çevresini 360 eşit parçaya böldüǧümüzde, bu parçalardan her-
birini merkezden gören açının genişliǧine (ölçüsüne) 1 derecelik açı denir ve 1◦
ile gösterilir. O halde, bir çemberin çevresini gören açı 360◦ ölçüye sahiptir.
Birim çemberde, bir açıya karşılık gelen yayın uzunluǧuna o açının radyan
ölçüsü denir. Buna göre, birim çemberin çevresini gören açı ölçüsü 2π radyan-
dır. Dolayısıyla, her çemberin çevresini gören açı ölçüsü 2π radyandır. Aşa-
ǧıda sık kullanılan bazı açıların derece ve radyan ölçüleri verilmiştir. Bun-
lardan, örneǧin 30◦ ’lik açının radyan hesabı için, ”360◦ ’lik açı 2π radyan ise,
30◦ ’lik açı kaç radyandır?” şeklinde bir orantı kurulur.
2.6. TRİGONOMETRİK FONKSİYONLAR 35

Tablo 1.1 Temel açılar.


Der 0◦ 30◦ 45◦ 60◦ 90◦ 120◦ 135◦ 150◦ 180◦ 270◦ 360◦
π π π π 2π 3π 5π 3π
Rad 0 6 4 3 2 3 4 6
π 2

Şimdi, merkezi orijinde ve yarıçapı 1 br olan çemberi çizelim (Şekil 2.24).
A(1,0) noktasından başlayarak çember üzerinde |t| radyan ilerleyelim (t>0
için saat yönü tersine, t<0 için saat yönünde ilerlenecek). Bu durumda
çember üzerinde bir P (x, y) noktası elde edilecektir.

Şekil 2.24 Birim çember.

t deǧiştikçe bulunan bu P (x, y) noktasının apsis ve ordinatları da deǧişeceǧin-


den, P (x, y) noktasının apsisi x(t)=:cos t ve odinatı da y(t)=:sin t fonksiyon-
ları olarak adlandırılır. Böyelece her t reel sayısına birim çember üzerinde
bir P (x, y)=P (cos t, sin t) noktası karşılık getirilir. Buna göre

−1 ≤ sin t ≤ +1 ve − 1 ≤ cos t ≤ +1

olacaǧı açıktır. Şimdi, bir AOB açısı verildiǧinde, bu açının köşesini orijin
ve [OB] kenarını da x ekseni üzerinde alalım.

Şekil 2.25 Bir açının trigonometrik oranları.


36 CHAPTER 2. TEMEL BİLGİLER

Bu açının ölçüsü t radyan olsun. O halde, her AOB açısına bir t sayısı karşılık
getirilebilir. OP C ügeni ile OAB üçgeni benzer olduklarından, r=|OA| ol-
mak üzere,

(a) sin t = |P1N | = yr (b) cos t = |ON |


1
= xr (c) tan t = |P N |
|ON |
= xy
(d) cot t = |ON |
|P N |
= xy (e) sec t = r
x
= cos1 t (f ) csc t = r
y
= sin1 t

eşitlikleri vardır. Bunlar, OAB üçgenindeki O açısının trigonometrik oranları


olarak adlandırılır. Bu tanımlardan, 0, π2 , π, 3π 2
ve 2π radyanlık açıların
trigonometrik deǧerleri kolayca söylenebilir. Eşkenar ile ikizkenar dik üçgen-
lerden de diǧer temel açıların trigonometrik deǧerleri hesaplanarak aşaǧıdaki
tablo elde edilebilir.

Tablo 1.2 Temel açıların trigonometrik oranları.

Açı (t)→ 0◦

30


45


60


90

0 1 2 3 4
sin t 2 2 √2 √2 2
1 2 3
sin t 0 √2 √2 2
1
3 2 1
cos t 1 0
√2
3
2 √2
tan t 0 1 3 tanımsız
√3 √
3
cot t tanımsız 3 1 3
0

2.6.1 Trigonometrik oranların özellikleri


Aşaǧıdaki (1)-(6) özellikleri doǧrudan birim çemberden görülebilir. (7) özel-
liǧinden de diǧerleri elde edilebilir.

(1) sin2 t + cos2 t = 1 (2) sin(−t) = − sin t (3) cos(−t) = cos t


(4) tan(−t) = − tan t (5) cot(−t) = cot t (6) sin(t + 2π) = sin t

Şekilden görülüyor ki,

sin(t1 + t2 )= |QP |
|OP |
= |QS|+|SP
|OP |
| |QS|
= |OP + |SP |
| |OP |
|T R|
= |OP + |SP |
| |OP |
|T R| |OR|
= |OR| + |SP | |RP |
|OP | |RP | |OP |
=sin t1 cos t2 +cos t1 sin t2
2.6. TRİGONOMETRİK FONKSİYONLAR 37

ve böylece aşaǧıdaki özellik (7) elde edilir. Bundan yararlanarak da diǧer


özellikleri görmek kolaydır.

Şekil 2.26 İki açının toplamının sinüsü.

(7) sin(t1 + t2 ) = sin t1 cos t2 + sin t2 cos t1


(8) sin(t1 − t2 ) = sin t1 cos t2 − sin t2 cos t1
(9) sin( π2 − t) = cos t
(10) cos(t1 + t2 ) = cos t1 cos t2 − sin t1 sin t2
tant1 +tant2
(11) tan(t1 + t2 ) = 1−tant1 tant2

(12) cos t1 cos t2 = 12 [cos(t1 + t2 ) + cos(t1 − t2 )]


(13) sin t1 cos t2 = 12 [sin(t1 + t2 ) + sin(t1 − t2 )]
(14) sin t1 sin t2 = 12 [cos(t1 + t2 ) − cos(t1 − t2 )]

2.6.2 Trigonometrik fonksiyonlar


Madem ki sin t trigonometrik oranı her t reel sayısı için birim çember üzerinde
[−1, +1] aralıǧında bir deǧere karşılık gelmektedir, o halde R’den [−1, +1]’e
bir fonksiyon olarak tanımlanabilir. Bu durumda, f (x)=sin x, f : R 7→
[−1, +1] ile tanımlı fonksiyon örten olur ama birebir olmaz. Aşaǧıdaki şekil-
38 CHAPTER 2. TEMEL BİLGİLER

Şekil 2.27 f : R 7→ [−1, +1], f (x) = sin(x) fonksiyonu grafiǧi.

den de görüldüǧü gibi, f :[− π2 , π2 ] 7→ [−1, +1], f (x)=sin(x) olarak tanımlanır-


sa bu fonksiyon birebir ve örten olur. Bu durumda arksinüs (arcsin = sin−1 )
denen bir tersi vardır ve aşaǧıdaki gibi tanımlanır; grafiǧi aşaǧıdadır.
π π
arcsin : [−1, +1] 7→ [− , ], arcsin : x 7→ sin−1 (x) = arcsin(x)
2 2

Şekil 2.28 f : [− π2 , π2 ] 7→ [−1, +1], f (x) = sin(x) ve tersi.

Benzer olarak, uygun aralıklar üzerinde tanımlanan cos, tan ve cot fonksi-
yonları ile terslerinin grafikleri çizilebilir. Hemen belirtelim ki, bu fonksi-
yonların birebir ve örten olduǧu aralıklar [ π2 , 3π
2
], [ 3π
2
, 5π
2
] veya [− 5π
2
, − 3π
2
]
biçiminde de seçilebilir.

ALIŞTIRMALAR (Trigonometrik fonksiyonlar)

1. Trigonometrik fonksiyonların (1)-(6) özelliklerini birim çember üzerinde


açıklayınız.
2.6. TRİGONOMETRİK FONKSİYONLAR 39

2. Trigonometrik fonksiyonların (8)-(14) özelliklerini (7) özelliǧinden elde


ediniz.

3. Yarım açı formülleri denilen sin 2x, cos 2x, tan 2x ve cot 2x deǧerlerini
trigonometrik fonksiyonların özelliklerinden elde ediniz.

4. arcsin √213 + arcsin √313 = x ise x deǧerini hesaplayınız.

5. Bir dik üçgen üzerinde tan(arcsin x), cot(arccos x), cos(arctan x) deǧer-
lerini hesaplayınız.
a+b
6. arctan a + arctan b = arctan( 1−ab ) olduǧunu gösteriniz.

7. cos(2 arcsin x) deǧerini hesaplayınız.

8. tan fonksiyonunun grafiǧini çiziniz. Bu fonksiyonun birebir ve örten


olduǧu en geniş tanım aralıǧını bularak, bu aralıktaki tersinin (arctan)
grafiǧini çiziniz.
40 CHAPTER 2. TEMEL BİLGİLER
Chapter 3

TÜREV

Türev kavramı özel bir limit olarak tanımlandıǧından, öncelikle limit ve


süreklilik kavramları üzerinde duracaǧız.

3.1 LİMİT VE SÜREKLİLİK


Tanım 3.1. a sayısı y=f (x) fonksiyonunun tanım kümesinin bir yıǧılma
noktası olsun. Her ε > 0 için, |x − a| < δ olduǧunda |f (x) − L| < ε olacak
şekilde bir δ = δ(ε) pozitif sayısı bulunabiliyorsa, x a’ya yaklaşırken f(x)’in
limiti L’dir denir ve
lim f (x) = L
x→a
ile gösterilir.

Şekil 3.29 y = f (x)’in x → a iken limiti.


Not 3.1. Tanımda dikkat edeceǧimiz bir nokta, y=f (x) fonksiyonunun x=a
noktasında tanımlı olması gerekmediǧi, ancak, bu nokta civarında tanımlı
olması gerektiǧidir. Yani, y=f (x) fonksiyonu x=a noktasının bir delinmiş δ
komşuluǧunda tanımlı olmalıdır.

41
42 CHAPTER 3. TÜREV

Not 3.2. Burada, ε sayısı f (x)’i L’ye ne kadar yaklaştırmak istediǧimizin


ölçüsü iken, δ sayısı buna baǧlı olarak x’i a’ya ne kadar yaklaştırmamız
gerektiǧinin ölçüsüdür.

Örnek 3.1. limx→2 2x + 1 = 5 olduǧunu gösterelim. ε > 0 verilsin.


ε
|f (x) − 5| < ε ⇒ |(2x + 1) − 5| < ε ⇒ |2x − 4| < ε ⇒ |x − 2| <
2

bulunur. Buradan, yukarıdaki gerektirmeler tersine çalıştırılarak, her ε > 0


için, eǧer δ = 2ε seçilirse, |x − 2| < δ olduǧunda |f (x) − 5| < ε olur. O halde,
limx→2 2x + 1 = 5’tir.

Aşaǧıdaki örnek yukarıdaki notta sözettiǧimiz duruma ilişkindir: x = 2 için


fonksiyon tanımlı olmadıǧı halde, bu noktada limit vardır.
x2 −4
Örnek 3.2. limx→2 x−2
= 4 olduǧunu gösterelim. ε > 0 verilsin.

(x − 2)(x + 2)
|f (x) − 4| = | − 4| < ε ⇒ |(x + 2) − 4| < ε ⇒ |x − 2| < ε
x−2

bulunur. Burada da δ = ε seçilirse, |x − 2| < δ olduǧunda |f (x) − 4| < ε


2 −4
olur. O halde, limx→2 xx−2 = 4’tür.

x2 −4
Şekil 3.30 x 6= 2 için f (x) = x−2
= x + 2’nin x → 2 iken limiti.

Örnek 3.3. limx→0 (x2 + 3) = 3 olduǧunu gösterelim. ε > 0 verilsin.



|(x2 + 3) − 3| = |x2 | < ε ⇒ |x| < ε

bulunur. Şimdi, δ = ε seçilirse, |x| < δ olduǧunda |(x2 + 3) − 3| < ε olur.
O halde, limx→0 (x2 + 3) = 3’tür.
3.1. LİMİT VE SÜREKLİLİK 43

Tanım 3.2. (Saǧdan limit-soldan limit) x’in a’dan büyük deǧerlerle a’ya
yaklaşması halinde f (x) de l1 ’e yaklaşıyorsa, l1 ’e f (x)’in saǧdan limiti denir
ve
lim+ f (x) = l1
x→a

ile gösterilir. Benzer olarak

lim f (x) = l2
x→a−

soldan limiti tanımlanır. Saǧdan ve soldan limitler eşit ise, bulunan limit
deǧeri f (x)’in x→a için limitidir, aksi halde f (x)’in x→a için limiti yoktur
deriz.

Şekil 3.31 Saǧdan ve soldan limitler.


x
Örnek 3.4. f (x) = |x| fonksiyonu x = 0 için tanımlı olmadıǧı halde, x → 0+
saǧdan ve x → 0− soldan limitlerini sırasıyla bulalım. (Şekil:1.32)

Şekil 3.32 f (x) = x


|x|
’in x → 0+ ve x → 0− limitleri.
x x
x > 0 için: |x| = x olup, f (x) = |x|
= x
= 1 ve
x
limx→0+ f (x) = limx→0+ |x|
x
= limx→0+ x
= limx→0+ 1
=1
44 CHAPTER 3. TÜREV

bulunur.
x x
x < 0 için: |x| = −x olup, f (x) = |x|
= −x
= −1 ve
x
limx→0− f (x) = limx→0− |x|
x
= limx→0− −x
= limx→0− −1
= −1
x
bulunur. O halde, limx→0 |x|
yoktur.
x−1
Örnek 3.5. limx→1 1 − x + |x−1| limitini hesaplayalım. x deǧişkeni saǧdan
x−1
ve soldan 1’e yaklaşırken g(x) = |x−1| fonksiyonu farklı deǧerler aldıǧından
saǧdan ve soldan limitleri ayrı ayrı hesaplamak gerekir:
x−1 x−1
x − 1 > 0 için: |x − 1| = x − 1 olup, |x−1| = x−1 = 1 ve
x−1 x−1
limx→1+ 1 − x + |x−1|
= limx→1+ 1 − x + x−1
=1
bulunur.
x−1 x
x − 1 < 0 için: |x − 1| = −(x − 1) olup, |x−1|
= −(x−1)
= −1 ve
x−1 x−1
limx→1− 1 − x + |x−1|
= limx→1− 1 − x + 1−x
= −1
x−1
bulunur. O halde, limx→1 1 − x + |x−1|
yoktur.

Şimdi de x → ∞ için f (x)’in limitini tanımlayalım.

Tanım 3.3. (Sonsuzdaki limit) Her ε > 0 için istenildiǧi kadar büyük bir
P > 0 bulunabiliyorsa öyle ki |x| > P olduǧunda |f (x) − L| < ε oluyorsa x
∞’a yaklaşırken f(x)’in limiti L’dir denir ve

lim f (x) = L
x→∞

ile gösterilir. Benzer olarak

lim f (x) = L
x→−∞

limiti tanımlanır.
3.1. LİMİT VE SÜREKLİLİK 45

Örnek 3.6. limx→∞ x−1 x


= 1 olduǧunu gösterelim. Her ε > 0 için |x| > P
olduǧunda |f (x) − 1| < ε olacak şekilde P sayısı bulalım.

|f (x) − 1| < ε ⇐⇒ | x−1


x
− 1| = 1
|x|

1
⇐⇒ |x| > ε

olduǧundan, eǧer P = 1ε seçilirse, |x| > P için |f (x)−1| < ε olur. ε istenildiǧi
kadar küçük seçilirse P de istenildiǧi kadar büyük olur.

Şimdi de x → a (a ∈ R) için f (x)’in limitinin ∞ olması halini tanımlayalım.


Tanım 3.4. (Limitin sonsuz olması) x → a için f (x)’in limitinin ∞
olması demek, P gibi istenildiǧi kadar büyük pozitif bir sayı seçildikten sonra

|x − a| < δ

olduǧunda
|f (x)| > P
olacak şekilde bir δ sayısının bulunabilmesi demektir. Böyle bir limit

lim f (x) = ∞
x→a

ile gösterilir.
1
Örnek 3.7. limx→2 (x−2)2
= ∞ olduǧunu gösterelim. Limit doǧru ise, P > 0
için,
1 1
(x−2)2
>P ⇐⇒ |x − 2|2 < P
⇐⇒ |x − 2| < √1
P

ve δ = √1P olması gerekir. Böyelece, tersten giderek, P istenildiǧi kadar


büyük bir sayı olarak seçildiǧinde δ da istenildiǧi kadar küçük bir sayı olur.
O halde limit doǧrudur.
Tanım 3.5. (Sonsuzdaki limitin sonsuz olması) x → ∞ için f (x)’in
limitinin ∞ olması demek, P gibi istenildiǧi kadar büyük pozitif bir sayı
seçildikten sonra
|x| > N
olduǧunda
|f (x)| > P
46 CHAPTER 3. TÜREV

olacak şekilde bir N sayısının bulunabilmesi demektir. Böyle bir limit

lim f (x) = ∞
x→∞

ile gösterilir. Burada P istenildiǧi kadar büyük seçildiǧinde N de istenildiǧi


kadar büyük bir sayı olur.

Örnek 3.8. limx→∞ x2 = ∞ olduǧunu gösterelim. Limit doǧru ise, P > 0


için, √
|x2 | > P ⇐⇒ |x|2 > P ⇐⇒ |x| > P

ve N = P olması gerekir. Böyelece, tersten giderek, P istenildiǧi kadar
büyük bir sayı olarak seçildiǧinde N de istenildiǧi kadar büyük bir sayı olur.
O halde limit doǧrudur.

Not 3.3. Hemen belirtelim ki y=f (x) fonksiyonu x→a (a ∈ R veya a = ∞)


halinde sonlu veya sonsuz bir limite yaklaşmayabilir. Örneǧin,

lim cos x
x→∞

limiti yoktur.

Not 3.4. Şu limitler sıkça karşımıza çıkar:


1 1 1 1 1
lim = 0, lim 2 = 0, lim+ = ∞, lim− = −∞, lim 2 = ∞.
x→∞ x x→∞ x x→0 x x→0 x x→0 x

Sonuç 3.1. P1 (x) ve P2 (x) polinomları sırasıyla

P1 (x) = an xn + an−1 xn−1 + · · · + a0 ve P2 (x) = bm xm + bm−1 xm−1 + · · · + b0

olsun. Bu durumda,
an

,n=m
P1 (x)  bm
lim = 0, n < m
x→∞ P2 (x)
∞, n > m

olur. Örneǧin,
1
3x3 + x x3 (3 + x2
) 3
lim = lim 1 =
x→∞ 5x3 − 1 x→∞ x3 (5 − ) 5
x3
olur.
3.1. LİMİT VE SÜREKLİLİK 47

Limit ile ilgili tanımlar kullanılarak aşaǧıdaki teorem ispatlanabilir.


Teorem 3.1. İki fonksiyonun toplamının limiti limitleri toplamı ve çarpımı-
nın limiti limitleri çarpımıdır.
Problem 3.1. Birçok trigonometrik fonksiyonun limiti için temel olan
sin x
lim =1
x→0 x

eşitliǧini gösterelim.

sin x
Şekil 3.33 limx→0 x
=1
Yarıçapı 1 br olan yukarıdaki daireyi göz önüne alalım. 0 ≤ x < π2 olmak
üzere N OP açısına x diyelim. Şekilden görülmektedir ki, T OA üçgeninin
alanı N OA daire diliminin alanından büyük, N OA daire diliminin alanı da
N OA üçgeninin alanından büyüktür. Buna göre, |T A| = tan x olduǧundan,
1 1 1 sin x
· 1 · sin x ≤ x · ≤ · 1 · ⇒
2 2 2 cos x
sin x 1 1 cos x sin x
sin x ≤ x ≤ ⇒ ≥ ≥ ⇒1≥ ≥ cos x
cos x sin x x sin x x
olur. Buradan limite geçilirse,
sin x sin x
lim 1 ≥ lim ≥ lim cos x ⇒ 1 ≥ lim ≥1
x→0 x→0 x x→0 x→0 x
sin x
bulunur. O halde, limx→0 x
= 1’dir.
tan x
Örnek 3.9. limx→0 x
limitini bulalım.
tan x sin x 1
lim = lim · =1
x→0 x x→0 x cos x
bulunur.
48 CHAPTER 3. TÜREV

Tanım 3.6. (Süreklilik) A ⊂ R, f : A 7→ R ve a ∈ A olsun. Eǧer,

lim f (x) = f (a)


x→a

ise, f ’ye a noktasında süreklidir denir. Eǧer, f fonksiyonu A tanım kümesinin


her noktasında sürekli ise f ’ye süreklidir denir.

Örnek 3.10. R üzerinde



 −x2 , x < 0
f (x) = 0, x = 0
1, x > 0

şeklinde tanımlı fonksiyonun x = 0 noktasındaki sürekliliǧini inceleyelim.


f (0) = 0 olduǧu görülmektedir. Ancak, x = 0’daki limite baktıǧımızda,

lim f (x) = 1 ve lim− f (x) = 0


x→0+ x→0

olup limx→0 f (x) yoktur. O halde, f fonksiyonu x=0 noktasında süreksizdir.

ALIŞTIRMALAR (Limit ve Süreklilik)

1. f (x) = 2x − 5 olsun. Bir δ > 0 sayısı bulunuz ki |x − 2| < δ olduǧunda,


1
|f (x) + 1| < 100 olsun.

2. g(x) = 3x2 − 1 olsun. Bir δ > 0 sayısı bulunuz ki |x| < δ olduǧunda,
1
|g(x) − g(0)| < 10 olsun.

3. f (x) = 2x+3 ve ε > 0 olsun. Öyle bir δ > 0 sayısı bulunuz ki |x−2| < δ
olduǧunda, |f (x) − 7| < ε olsun.

4. f (x)=2x2 + 3 olsun. ε−δ tanımını kullanarak limx→0 f (x)=3 olduǧunu


gösteriniz.
sin mx m
5. limx→0 sin nx
=n olduǧunu gösteriniz.

6. Aşaǧıdaki limitleri hesaplayınız.


|3x| x3 −8
(a) limx→2 x22−4 − x−2 1

(b) limx→1/3 x
(c) limx→2 x−2
√ √ √ √ √
1− 1−x2 x− a
(d) limx→0 x2
(e) limx→a x−a (f ) limh→0 x+h− h
x
3.2. TÜREV KAVRAMI 49

(x+h)m −xm
7. m ∈ N+ için limh→0 h
= mxm−1 olduǧunu gösteriniz.

8. m, n, a belirli sabitler olmak üzere aşaǧıdaki trigonometrik limitleri


hesaplayınız.
sin x
(a) limx→∞ x
(b) limx→∞ x · sin x1 (c) limx→a sin x−sin a
x−a
cos h−1 sin(x2 −1) sin(tan x)
(d) limh→0 h (e) limx→1 x−1
(f ) limx→0 sin x

9. Aşaǧıdaki limitleri hesaplayınız.


√ 
(a) limx→∞ x2 − 3x − 4 − x
√ 
(b) limx→−∞ x2 − 3x − 4 + x
√ √
1+x− 1−x
(c) limx→0 x

10. f (x)=[|x|] fonksiyonunun x=2 noktasındaki sürekliliǧini inceleyiniz.

3.2 TÜREV KAVRAMI

∆y
Şekil 3.34 tan α = ∆x

y = f (x) eǧrisi üzerinde alınan A ve B noktalarını birleştiren doǧrunun eǧimi

∆y f (x + ∆x) − f (x)
tan α = =
∆x ∆x
olur. Burada eǧer ∆x deǧeri 0’a yaklaştırılırsa, B noktası da y = f (x)
eǧrisi üzerinden A’ya yaklaşır. Böylece AB doǧrusu da A noktasındaki teǧete
dönüşür. Bu yüzden,
50 CHAPTER 3. TÜREV

m = tan α
∆y
= lim∆x→0 ∆x
= lim∆x→0 ∆f∆x(x)
= lim∆x→0 f (x+∆x)−f
∆x
(x)

limiti y = f (x) eǧrisinin A(x, f (x)) noktasındaki teǧetinin eǧimi olur.


Örnek 3.11. y = x2 + 1 eǧrisinin A(1, 2) noktasındaki teǧetinin eǧimini
bulup, bu teǧeti belirleyelim. Buna göre,
f (1 + ∆x) − f (1)
m = tan α = lim
∆x→0 ∆x
ve ∆x’e h diyerek,
f (1+h)−f (1)
m = limh→0 h
(1+h)2 −1
= limh→0 h
h(h+2)
= limh→0 h
=2
bulunur. O halde, eǧimi m = 2 ve A(1, 2) noktasından geçen doǧru denklemi,
y − 2 = 2(x − 1) ⇒ y = 2x
bulunur.
Tanım 3.7. (Bir fonksiyonun bir noktasındaki türevi) f : A ⊂ R 7→ R
ve x0 ∈ A bir yıǧılma noktası olsun. Eǧer varsa,
f (x0 + h) − f (x0 )
lim
h→0 h
df (x0 )
limitine f fonksiyonunun x = x0 noktasındaki türevi denir ve f 0 (x0 ), dx
,
df (x)
] , dy ]
dx x=x0 dx x=x0
sembollerinden biri ile gösterilir.
Örnek 3.12. f (x)=|x| eǧrisinin x=0 noktasındaki türevini hesaplayalım.
|0 + h| − |0| |h|
lim = lim
h→0 h h→0 h

olur. Oysa, h > 0 için 1 ve h < 0 için -1 olduǧundan bu limit yoktur. O


halde, x=0 noktasında bu fonksiyonun türevi yoktur.
3.2. TÜREV KAVRAMI 51

Tanım 3.8. (Bir fonksiyonun türevi) Bir y = f (x) fonksiyonunun tanımlı


olduǧu her noktada türevi varsa, bu fonksiyon türevlenebilirdir denir. Bu
dy
fonksiyonun f 0 (x), y 0 veya dx ile gösterilen türevi

f (x+∆x)−f (x)
f 0 (x) = lim∆x→0 ∆x
f (x+h)−f (x)
= limh→0 h

limitlerinden biri olarak tanımlanır.

Örnek 3.13. f (x)=x2 + x fonksiyonunun türevini hesaplayalım.


f (x+h)−f (x) (x+h)2 +(x+h)−(x2 +x)
limh→0 h
= limh→0 h
= limh→0 h(2x+h+1)
h
= 2x + 1

olur.

Şimdi bazı temel fonksiyonların türevlerini hesaplayalım.

Problem 3.2. f (x) = c (c belirli bir sayı) fonksiyonunun türevini bulalım:


f (x+h)−f (x) c−c
limh→0 h
= limh→0 h
0
= limh→0 h
=0

olur. O halde, sabit fonksiyonun türevi 0’dır.

Problem 3.3. f (x) = x birim fonksiyonunun türevini bulalım:


f (x+h)−f (x) (x+h)−x
limh→0 h
= limh→0 h
h
= limh→0 h
=1

olur. O halde, birim fonksiyonun türevi 1’dir.

Şimdi de türev ile ilgili temel özellikleri verelim.


52 CHAPTER 3. TÜREV

Özellik 3.1. y = f (x) fonksiyonunun türevi f 0 (x) iken = c · f (x) (c belirli


bir sayı) fonksiyonunun türevini bulalım:
c·f (x+h)−c·f (x)
[c · f (x)]0 = limh→0 h
f (x+h)−f (x)
=c· limh→0 h
0
= c · f (x)

olur. O halde, bir fonksiyonun c katının türevi, türevinin c katıdır.

Özellik 3.2. f ile g fonksiyonlarının toplamının türevi bulalım:


[f (x+h)+g(x+h)]−[f (x)+g(x)]
[f (x) + g(x)]0 = limh→0 h
f (x+h)−f (x)
= limh→0 h
+ limh→0 g(x+h)−g(x)
h
0 0
= f (x) + g (x)

olur. O halde, iki fonksiyonun toplamının türevi, türevlerinin toplamıdır.

Yukarıdaki özelliǧin benzer olarak çarpım ve bölüm için de geçerli olduǧunu


ilk bakışta düşünebiliriz, ancak aşaǧıda göreceǧimiz gibi durum hiç de böyle
deǧildir.

Özellik 3.3. f ile g fonksiyonlarının çarpımının türevini hesaplayalım:


f (x+h)·g(x+h)−f (x)·g(x)
[f (x) · g(x)]0 = limh→0 h
f (x+h)·g(x+h)−f (x)·g(x)+f (x+h)·g(x)−f (x+h)·g(x)
= limh→0 h h i
= limh→0 f (x + h) g(x+h)−g(x)
h
+ g(x) f (x+h)−f (x)
h

= f (x) · g 0 (x) + g(x) · f 0 (x)

bulunur.

Örnek 3.14. f (x)=(x2 + x)(x − 1) için f 0 (x) türevini hesaplayalım. Buna


göre,
f 0 (x) = (x2 + x)0 (x − 1) + (x2 + x)(x − 1)0
= (2x + 1)(x − 1) + (x2 + x)(1 − 0)
= 3x2 − 1

bulunur.
3.2. TÜREV KAVRAMI 53

Özellik 3.4. f ile g fonksiyonlarının bölümünün türevini hesaplayalım:


f (x+h) f (x)
− g(x)
[ fg(x)
(x) 0
] = limh→0 h
g(x+h)

g(x)·f (x+h)−g(x+h)·f (x)


= limh→0 h·g(x)·g(x+h)
g(x)·f (x+h)−g(x+h)·f (x)−f (x)·g(x)+f (x)·g(x)
= limh→0 h·g(x)·g(x+h)
f (x+h)−f (x) g(x+h)−g(x)
g(x)· −f (x)·
= limh→0 h
g(x)·g(x+h)
h

g(x)·f 0 (x)−f (x)·g 0 (x)


= [g(x)]2

olur.
2
Örnek 3.15. f (x)= (x(x−1)
+x)
için f 0 (x) türevini hesaplayalım. Buna göre,

(x2 +x)0 (x−1)−(x2 +x)(x−1)0


f 0 (x) = (x−1)2
(2x+1)(x−1)−(x2 +x)
= (x−1)2
x2 −2x−1
= (x−1)2

bulunur.

Özellik 3.5. ( İki fonksiyonun bileşkesinin türevi) y = g(x) ve u = f (y)


ise, yani, u = f (g(x)) ise, du
dx
türevini arıyoruz. O halde,
du ∆u
dx
= lim∆x→0 ∆x
∆u ∆y
= lim∆x→0 ∆y
· ∆x
∆u ∆y
= lim∆y→0 ∆y · lim∆x→0 ∆x
du dy
= ·
dy dx


Örnek 3.16. f (x)= x2 + 1 fonksiyonunun türevini inceleyelim. y = x2 + 1

ve f (x) = y = u alınırsa
du dy
f 0 (x) = ·
dy dx
1
= √
2· y
· (2x)
x
= √
x2 +1

bulunur.
54 CHAPTER 3. TÜREV

ALIŞTIRMALAR (Türev Kavramı)


1. Türev tanımını kullanarak f (x) = x2 ise f 0 (x) türevini hesaplayınız.

2. Türev tanımını kullanarak y = x ise y 0 türevini hesaplayınız.

3. y = x2 eǧrisinin P(2,4) noktasındaki teǧetinin denklemini bulunuz.



4. y= x eǧrisinin x=4 apsisli noktadaki teǧetinin denklemini bulunuz.

5. y=ax2 eǧrisinin x=1 apsisli noktadaki teǧetinin Ox ekseniyle 45o ’lik


açı yapması için a ne olmalıdır?
q
6. y= 1+x 1−x
ise y 0 türevini hesaplayınız.

7. y = 5
x2 + 1 ise y 0 türevini hesaplayınız.

3.3 BAZI FONKSİYONLARIN TÜREVİ

Şimdi trigonometri ve logaritma fonksiyonlarının türevlerine bakalım.


Problem 3.4. f (x) = sin x fonksiyonunun türevini bulalım:
sin(x+h)−sin x
(sin x)0 = limh→0 h
sin x cos h+sin h cos x−sin x
= limh→0 h
cos h−1
= limh→0 h
· sin x + limh→0 sinh h · cos x
= cos x

bulunur. O halde, y = sin x ise y 0 = cos x olur.


Sonuç 3.2. Yukarıdakine benzer olarak, y = cos x ise y 0 = − sin x olduǧu
görülebilir. Buradan, iki fonksiyonun bölümünün türevinden de yararla-
narak, y = tan x ise y 0 = cos12 x ve y = cot x ise y 0 = sin−12 x elde edilir.

Problem 3.5. f (x) = ln x fonksiyonunun türevini bulalım. Doǧal logaritma


tabanı olan e = 2.718281828459045 · · · irrasyonel sayısı
1 1
e = lim (1 + x) x veya e = lim (1 + )x
x→0 x→∞ x
3.3. BAZI FONKSİYONLARIN TÜREVİ 55

limiti olarak tanımlanır. Buna göre, y = loge x = ln x ise


ln(x+h)−ln x
y0 = limh→0 h
ln x+h
= limh→0 hx
ln(1+ h )
= limh→0 h x
ln(1+ h )
= limh→0 h ·xx
x
x
ln(1+ h )h
= limh→0 x
x

h
ve burada x
= k alınırsa, h → 0 için k → 0 olacaǧından,
x
ln(1+ h )h
y0 = limh→0 x
x
1

limk→0 ln(1+k)
k
= x
ln e
= x
1
= x

bulunur. O halde, y = ln x ise y 0 = 1


x
olur.
Sonuç 3.3. y = loga x = loga e · loge x = loga e · ln x ise y 0 = loga e · 1
x
olur.
Sonuç 3.4. y = ax ise her iki tarafın a tabanında logaritması alınarak bulu-
nan eşitliǧin türevi hesaplanır ve y 0 türevi çekilir. Buna logaritmik türev veya
üstel fonksiyonun türevi denir. O halde, y = ax ise ln y = x ln a ⇒ y1 ·y 0 = ln a
ve y 0 = y · ln a = ax · ln a bulunur. (Burada y’nin fonksiyon olduǧuna,
dolayısıyla da ln y’nin bileşke fonksiyonu olduǧuna dikkat edelim.)
Örnek 3.17. y = 5x ise ln y = x ln 5 ⇒ y 0 = y · ln 5 = 5x · ln 5 bulunur.
Sonuç 3.5. y = xn (n ∈ R) ise, y 0 türevini bulalım. Bunun için logarit-
mik türeve başvurursak, ln y = ln xn = n ln x ve buradan iki tarafın türevi
0
alınırsa, yy = n · x1 ve y 0 = y · nx = n · xn−1 bulunur.
√ √ √
Örnek 3.18. y = x 5 ise y 0 = 5x 5−1 olur.
Örnek 3.19. y = ln(tan x) ise
y0 = 1
tan x
· (tan x)0
1
= · 1
tan x cos2 x
1
= sin x·cos x

bulunur.
56 CHAPTER 3. TÜREV

Özellik 3.6. (Bir fonksiyonun tersinin türevi) f : A ⊂ R 7→ B ⊂ R


fonksiyonu birebir ve örten olsun. f fonksiyonu x0 ∈ A noktasında türevlene-
bilir ve f 0 (x0 ) 6= 0 ise, f −1 : B 7→ A fonksiyonu da y0 = f (x0 )’da türevlene-
bilir ve
1
(f −1 )0 (y0 ) = 0
f (x0 )
olur. Gerçekten,
f −1 (y)−f −1 (y0 )
(f −1 )0 (y0 ) = limy→y0 y−y0
x−x0
= limy→y0 f (x)−f (x0 )
1
= limx→x0 f (x)−f (x0 )
x−x0
1
= f 0 (x0 )

bulunur.
dy
Sonuç 3.6. y = f (x)’in türevi dx
iken, x = f −1 (y)’nin türevi
dx 1
dy
= dy
dx
1
= f 0 (x)

olur.
Örnek 3.20. y = f (x) = 2x + 1’in tersinin türevi
dx 1
dy
= dy
dx
1
= f 0 (x)
1
= 2

olur.
Şimdi trigonometrik fonksiyonların terslerinin türevlerini hesaplayalım:
Problem 3.6. y=arcsin x için; y=arcsin x ⇒ x=sin y olup,
dy
y0 = dx
1
= dx
dy
1
= cos y
=√ 1
1−sin2 y
= √ 1
1−x2
3.3. BAZI FONKSİYONLARIN TÜREVİ 57

−1
olur. Benzer olarak, y=arccos x ⇒ y 0 = √1−x 0 1
2 ve y=arctan x ⇒ y = 1+x2
olduǧu kolaylıkla gösterilebilir.

Ödev 3.1. y=arcsec x’in türevinin y 0 = x√x12 −1 olduǧunu gösteriniz.

Tanım 3.9. (Kapalı fonksiyon) y=f (x) olmak üzere, g(x, y)=0 eşitliǧiyle
(baǧıntısıyla) verilen y fonksiyonuna kapalı olarak verilmiş fonksiyon deriz.

Örnek 3.21. y=f (x) olmak üzere, x2 + y 2 − 1 = 0 ve exy + x2 + y 3 + 1 = 0


eşitliklerinde y kapalı fonksiyondur.
dy
Örnek 3.22. x2 + y 2 − 1 = 0 için y 0 = dx türevini hesaplayalım. Bunun için,
y baǧımlı deǧişken ve x de baǧımsız deǧişken olduǧundan,

x2 + y 2 − 1 = 0 ⇒ 2x + 2yy 0 = 0
⇒ y 0 = − xy

olur. Dik koordinat sisteminde x2 + y 2 − 1 = 0 eşitliǧi bir çember belir-


tir. Bulduǧumuz y 0 deǧeri de, bu çemberin√ √
A(x, y) noktasındaki teǧetlerinin
eǧimleridir. Örneǧin, bu çemberin A( 2 , 22 ) noktasındaki teǧetin eǧimi
2

y 0 = −1 olup, teǧet denklemi


√ √
2 2 √
y− = −1(x − ) ⇒ x + y = 2’dir.
2 2

Şekil 3.35 x2 + y 2 − 1 = 0 çemberi ve bir teǧeti.

Tanım 3.10. (Yüksek basamaktan türevler) Bir f fonksiyonu, A tanım


kümesinde türevlenebiliyorsa, f 0 de bir fonksiyon belirtir. Eǧer bu f 0 fonksi-
yonu, bir x0 noktasında türevlenebiliyorsa, bu türeve f ’nin ikinci basamaktan
(mertebeden) türevi denir ve f 00 veya f (2) şekillerinden biri ile gösterilir.
58 CHAPTER 3. TÜREV

Benzer olarak, bir f fonksiyonunun bir x noktasındaki n-inci basamaktan


türevi
(n) f (n−1) (x + h) − f (n−1) (x)
f (x) = lim
h→0 h
limiti olarak tanımlanır.

Not 3.5. Bir y = f (x) fonksiyonunun yüksek basamaktan türevleri

y 0 , y 00 , y 000 , y (4) , y (5) , · · · , y (n) , · · ·

veya
y (1) , y (2) , y (3) , y (4) , y (5) , · · · , y (n) , · · ·

ile gösterilir. y (0) = y kabul edilir. Yani, bir fonksiyonun 0-ıncı basamaktan
türevi kendisi kabul edilir.

Örnek 3.23. y=x3 ise y 0 =3x2 , y 00 =6x, y 000 =6 ve y (4) =y (5) =· · · =0 yani, n≥4
için y (n) =0 olur.

Örnek 3.24. y= x1 = x−1 ise y (n) türevini bulalım.

y0 = (−1)x−2
y 00 = (−1)(−2)x−3
y (3) = (−1)(−2)(−3)x−4
..
.
n!
y (n) = (−1)(n) xn+1

olur.

Örnek 3.25. x2 − y 2 − 1 = 0 ise y (2) türevini bulalım. Bir kere türev alınırsa
2x − 2yy 0 = 0 ⇒ y 0 = xy ve bir kere daha türev alınırsa

0 0 00 1 − (y 0 )2
00
1 − ( xy )2 y 2 − x2
2 − 2[(y )(y ) + yy ] = 0 ⇒ y = = =
y y y3

bulunur.
3.4. DİFERENSİYEL KAVRAMI 59

ALIŞTIRMALAR (Bazı Fonksiyonların Türevi)

1. Ters fonksiyonun türevinden yararlanarak y = arctan x1 ise y 0 türevini


hesaplayınız.

2. y = arccsc x ise y 0 = √−1 olduǧunu gösteriniz.


x x2 −1
q
3. y = arctan x + ln 1+x
1−x
ise y 0 =? Doǧrudan hesaplayınız.

4. y = xx ise y 0 =? Logaritma yardımıyla hesaplayınız.


x 0
5. y = arcsin √1+x 2 ise y =? Doǧrudan hesaplayınız.

6. arctan xy = x + y ise y 00 ve y 000 türevlerini hesaplayınız.

3.4 DİFERENSİYEL KAVRAMI

Şimdi diferensiyel kavramı ve uygulamalarından söz edeceǧiz. Bir y=f (x)


fonksiyonunun türevinin
∆y
f 0 (x) = limx→0 ∆x
= limx→0 ∆f∆x(x)
= limx→0 f (x+∆x)−f
∆x
(x)

olduǧunu biliyoruz. Bundan anladıǧımız şudur: ∆x deǧeri 0’a çok yakın


olduǧunda ∆f (x)/∆x deǧeri de f 0 (x)’e çok yakın olur.

Tanım 3.11. (Diferensiyel) dy veya df (x) ile göstereceǧimiz y=f (x) fonk-
siyonunun diferensiyeli diye f 0 (x) · ∆x deǧerine denir. Yani, dy := f 0 (x) · ∆x
veya df (x) := f 0 (x) · ∆x şeklinde tanımlanır.

Bu tanımdan, f (x) = x seçilirse, ∆x → 0 için,

df (x) = dx
= f 0 (x) · ∆x
= 1 · ∆x
= ∆x
60 CHAPTER 3. TÜREV

ve böylece dx = ∆x olduǧu görülür. Dolayısıyla, y = f (x)’in diferensiyeli

dy = f 0 (x) · dx

olur.
dy
Sonuç 3.7. dy = f 0 (x) · dx ⇒ dx = f 0 (x) yazılabilir. O halde, bir fonksi-
yonun türevi, fonksiyonun diferensiyelinin baǧımsız deǧişkenin diferensiyeline
oranıdır.

Örnek 3.26. y = x2 için ∆y, dy ve ∆y − dy ifadelerini hesaplayıp yorum-


layalım.

∆y = f (x + ∆x) − f (x)
= 2x∆x + ∆x2
ve
dy = f 0 (x)∆x
= 2x∆x

olup, ∆y − dy=2x∆x + ∆x2 − 2x∆x=∆x2 bulunur.


Görülüyor ki, ∆x deǧeri 0’a yakın olduǧunda ∆y − dy deǧeri de 0’a yakın,
yani, ∆x → 0 için ∆y → dy olmaktadır. Örneǧin, y = f (x) = x2 için x = 1
ve ∆x = 0, 01 olduǧunda ∆y, dy ve ∆y − dy deǧerlerini hesaplayalım:

∆y = 2x∆x + ∆x2 , dy = 2x∆x ve ∆y − dy = ∆x2

olup ∆y = 0, 0201, dy = 0, 02 ve ∆y − dy = 0, 0001 bulunur.

Şu halde, ∆x ≈ 0 için ∆y ≈ dy olduǧunu gösterdik. (≈ işaretini yaklaşık


yerine kullanıyoruz.). Buradan, ∆x ≈ 0 için,

∆y = f (x + ∆x) − f (x)
≈ dy
= f 0 (x)∆x ⇒ f (x + ∆x) ≈ f 0 (x)∆x + f (x)

yazılabilir. Son yazdıǧımızın yaklaşık hesapta güzel uygulamaları vardır.


3.4. DİFERENSİYEL KAVRAMI 61

Örnek√ 3.27. 1, 01 deǧerini yaklaşık olarak hesaplayalım. Bu amaçla,
f (x)= x, ∆x=0,01 ve x=1 alınıp, yukarıdaki yaklaşık eşitlik kullanılabilir.
√ ∆x √
f (x + ∆x) ≈ f 0 (x)∆x + f (x) ⇒ x + ∆x ≈ √ + x
2 x
olduǧundan,
p 0, 01 √
1 + 0, 01 ≈ √ + 1 ≈ 0, 005 + 1 = 1, 005
2 1
bulunur.

Şimdi, bazı Diferensiyel denklemlerin çözümünde önemli bir araç olan toplam
diferensiyel kavramı üzerinde duracaǧız. Bunun için önce kısmi türev kavra-
mını verelim.
Tanım 3.12. (Kısmi türev) f (x, y, z) = 0 kapalı fonksiyonu verilsin. ∂f ∂x
ile gösterilen f ’nin x’e göre kısmi türevi diye, y ve z deǧişkenlerinin sabit
gibi düşünülerek bulunan x’e göre türevine denir.
Örnek 3.28. f (x, y, z) = x2 + y 2 + z 2 + xyz − exy + 5z + 1 = 0 için,
∂f ∂f ∂f
= 2x + yz − yexy , = 2y + xz − xexy , = 2z + xy + 5
∂x ∂y ∂z
olur.
Tanım 3.13. (Toplam diferensiyel) f (x, y, z) = 0 kapalı fonksiyonu ve-
rildiǧinde,
∂f ∂f ∂f
df = dx + dy + dz = 0
∂x ∂y ∂z
ifadesine f fonksiyonunun toplam diferensiyeli veya kısaca f ’nin diferensiyeli
denir.
Örnek 3.29. f (x, y) = x2 + y 2 + 2xy − 4 = 0 kapalı fonksiyonunun toplam
diferensiyelini hesaplayalım:
∂f ∂f
df = ∂x
dx + ∂y
dy
= (2x + 2y)dx + (2x + 2y)dy
= (2x + 2y)(dx + dy)
olur.
62 CHAPTER 3. TÜREV

Örnek 3.30. Bir dikdörtgenin x = 100cm uzunluǧundaki kenarı 2cm/s hzla


artarken, y = 80cm uzunluǧundaki diǧer kenarı 3cm/s hızla azalmaktadır.
Bu dikdörtgenin alanının deǧişim hızını hesaplayaım.
t anındaki uzunluǧu x = x(t) ve genişliǧi y = y(t) ise alanı A(x, y) = xy olur.
Buna göre, A’daki toplam deǧişim (toplam diferensiyel)
∂A ∂A
dA = dx + dy
∂x ∂y
ve deǧişim hızı
dA ∂A dx ∂A dy
= +
dt ∂x dt ∂y dt
olacaǧından, dikdörtgen alanının deǧişim hızı
dA dx dy
]x=100,y=80 = y + x ]x=100,y=80 = 80 · 2 + 100 · (−3) = −140
dt dt dt
olur. O halde, Dikdörtgenin alanı 140cm2 /s hızla azalmaktadır.

Şekil 3.36
Örnek 3.31. Dik koni (huni) şeklindeki dolu bir su tankı 5m derinliǧe ve 2m
1
yarıçapa sahiptir. Su seviyesi 4m olduǧu anda tank dakikada 12 m3 hızla su
boşaltmaktadır. Bu anda, tankın su seviyesi hangi hızla düşmektedir? Başka
bir deyişle, tanktaki suyun yüzeyinde bir şamandra (veya mantar) varsa, bu
şamandra hangi hızla aşaǧı iner? Hesaplayalım.
Su yüzeyinin yarıçapını r ile ve suyun yüksekliǧini de h ile gösterelim. Bu
durumda, t anındaki tanktaki su miktarı (hacmi) m3 cinsinden
1
V = πr2 h
3
3.4. DİFERENSİYEL KAVRAMI 63

olur. Şekilden, benzer üçgenlerden yararlanarak, r ile h arasında


r 2 2h 1 2h 4π 3
= ⇒r= ve V = π( )2 h = h
h 5 5 3 5 75
olur. t’ye göre diferensiyel alınırsa,
dV 4π 2 dh
= (h )
dt 25 dt
dV −1
bulunur. h = 4m iken dt
= 12
olduǧundan
−1 4π 2 dh dh 25
= (4 ) ⇒ =−
12 25 dt dt 768π
bulunur. Böylece, su tankındaki su seviyesinin düşüş hızı (seviye 4m iken)
dakikada yaklaşık 0, 01035m = 1, 035cm’dir.

ALIŞTIRMALAR (Diferensiyel Kavramı)


1. ln xyzt = 0 kapalı fonksiyonunun toplam diferensiyelini hesaplayınız.
2. 8m kenarı olan karenin her iki kenarı 2m/dak hızla artmaktadır. Bu
karenin alanı hangi hızda artar? Hesaplayınız.
3. Uzunluk (u), genişlik (g) ve yüksekliǧi (y) sırasıyla u = 6cm, g = 5cm
ve y = 4cm olan dikdörtgen prizma biçimindeki bir kutunun uzunluk
ve yüksekliǧi 1cm/s hızla artmakta ve genişliǧi de 2cm/s hızla azal-
maktadır. Kutunun hacminin deǧişim hızını bulunuz.
4. Su ile dolu iken 8mx20m boyutunda olan dikdörtgen biçimindeki bir
yüzme havuzunun derinliǧi, uzunlamasına bir uçtan (1m) ve diǧer uca
(3m) deǧişmektedir (havuz tabanı eǧimlidir). Havuzdan 1m3 /dak hızla
su çekilmektedir. Derin taraftaki su yüsekliǧi 2, 5m iken su yüzeyinin
küçülme hızı ne olur?
√ √ √ √
5. 3 10, 3 200, 3 126 ve 4 629 deǧerlerini yaklaşık olarak hesaplayınız.
6. Yarıçapı r olan bir kürenin hacmi 43 πr3 ’tür. Dünyanın ekvator çevresi
40.000km ise,
(a) Dünya yarıçapı ∆r = 1m arttırılırsa dünyanın hacmindeki artma
miktarı yaklaşık olarak ne kadar olur?
64 CHAPTER 3. TÜREV

(b) Ekvatoru çevreleyen bir ipin uzunluǧunu ne kadar arttıralım ki


yer yüzeyinden 2m yüksekten çevreleyebilsin?
√ √
7. |∆x| istenildiǧi kadar küçük olmak üzere x+∆x ≈ 2∆x
√ + x olduǧunu
x
√ √ √
gösteriniz. Bundan yararlanarak 5, 18 ve 24 deǧerlerini bulunuz.

8. Bir kürenin yarıçapı 10m’den 990cm’ye düşürülürse hacmindeki azalma


yaklaşık olarak ne kadar olur?

9. Bir kenarı 3,01m olan kübün hacminin yaklaşık deǧerini hesaplayınız.

3.5 TÜREVİN UYGULAMALARI

Bu kesimde, türevin limit konusundaki belirsiz şekiller, geometri ve fizikteki


bazı uygulamalarından söz edeceǧiz.

3.5.1 BELİRSİZ ŞEKİLLER

Bir fonksiyonun x = a noktasındaki limiti hesaplanırken şu belirsiz durumlar


karşımıza çıkabilir:

0 ∞
, , ∞ − ∞, 0 · ∞, 1∞ , 00 , ∞0 .
0 ∞
Bu durumlar genellikle türev yardımıyla kolaylıkla ortadan kaldırılabilir.
0
0
belirsizliǧi: Bunun için aşaǧıdaki teoremden yararlanacaǧız.

Teorem 3.2. (L’Hospital kuralı) f ve g fonksiyonları x = a noktasının


bir delinmiş komşuluǧunda türevli ve

lim f (x) = 0, lim g(x) = 0 ve lim g 0 (x) 6= 0 ise


x→a x→a x→a

f (x) f 0 (x)
lim = lim 0
x→a g(x) x→a g (x)

olur.
3.5. TÜREVİN UYGULAMALARI 65

İspat: f ve g fonksiyonları x = a için tanımlı ise f (a) = limx→a f (x) = 0


ve g(a) = limx→a g(x) = 0 olacaktır. Aksi durumda f (a) = g(a) = 0 olarak
tanımlayalım. Bu durumda,

f (x) f (x) − f (a)


=
g(x) g(x) − g(a)

yazılabilir. Burada, pay ile payda x − a ile bölünerek limit alınırsa,


f (x)−f (a)
f (x) x−a
limx→a g(x)
= limx→a g(x)−g(a)
x−a
f 0 (x)
= limx→a g 0 (x)
f 0 (a)
= g 0 (a)

f 0 (a) 0
bulunur. Burada, g 0 (a)
= 0
bulunuyorsa, yani,

f 0 (x) 0
lim 0
=
x→a g (x) 0

oluyorsa, L’Hospital kuralı tekrar uygulanır.


x2 −4 0
Örnek 3.32. limx→2 x−2
limitinde 0
belirsizliǧi olduǧundan L’Hospital ku-
ralı uygulanabilir:
x2 − 4 2x
lim = lim =4
x→2 x − 2 x→2 1

bulunur.
1−cos x 0
Örnek 3.33. limx→0 x2
limitinde 0
belirsizliǧi olduǧundan L’Hospital
kuralı uygulanabilir:

1 − cos x sin x cos x 1


lim 2
= lim = lim =
x→0 x x→0 2x x→0 2 2

bulunur.

∞ 0

belirsizliǧi: Bu ile 0
belirsizliǧi aynı olduǧundan, yine L’Hospital kuralı
uygulanır. (Neden?)
66 CHAPTER 3. TÜREV

ln x ∞
Örnek 3.34. limx→0+ cot x
= ∞
olduǧundan L’Hospital kuralı uygulanabilir:
1
ln x x − sin2 x 0
lim+ = lim+ −1 = lim+ = ⇒
x→0 cot x x→0 x→0 x 0
sin2 x

− sin2 x −2 sin x cos x


lim+ = lim+ =0
x→0 x x→0 1
bulunur.

∞ 0
0 · ∞ belirsizliǧi: Bu da kolaylıkla ∞
veya 0
belirsizliǧine dönüştürülür.

Örnek 3.35. limx→∞ x1 ex = 0 · ∞ olduǧundan

1 x ex ex
lim e = lim = lim =∞
x→∞ x x→∞ x x→∞ 1

bulunur.

∞ − ∞ belirsizliǧi: Bu belirsizlik şekli

1 1
g
− f
f −g = 1
f ·g

0
eşitliǧi yardımıyla 0
haline getirilebilir.

Örnek 3.36. limx→0 (csc x − x1 ) = ∞ − ∞ olduǧundan

1 1 1 x − sin x 1 − cos x
l = lim (csc x − ) = lim ( − ) = lim = lim ⇒
x→0 x x→0 sin x x x→0 x sin x x→0 sin x + x cos x

sin x
l = lim =0
x→0 cos x + cos −x sin x

bulunur.

∞0 , 00 , 1∞ belirsizlikleri: Logaritma yardımıyla 0 · ∞ belirsizliǧine dö-


nüştürülürler.
3.5. TÜREVİN UYGULAMALARI 67

Örnek 3.37. limx→0+ xx =? Bunun için, L=xx ise ln L=x ln x olacaǧından,


1
ln x x
lim+ ln L = lim+ x ln x = lim+ 1 = lim+ −1 = lim+ −x = 0 ⇒
x→0 x→0 x→0 x→0 x→0
x x2

lim L = lim+ xx = 1
x→0+ x→0

bulunur.

ALIŞTIRMALAR (Belirsiz şekiller)

1. Aşaǧıdaki limitleri hesaplayınız.


arctan x x−tan x sin x−x
(a) limx→0 sin x
(b) limx→0 x−sin x
(c) limx→0 x2
x3 −1 cos(sin x)−cos x
(d) limx→1 x−1 (e) limx→1 xln2 −1
x
(f ) limx→0 x2
sin 1 ln(sin x)
(g) limx→∞ arctanx 1 (h) limx→0+ ln(tan x)
(h) limx→0+ ln(sin 2x)
ln(sin 3x)
x

2. Aşaǧıdaki limitleri hesaplayınız.

(a) limx→0 ( x1 − 1
ex −1
) (b) limx→0 ( x1 − √1 )
x

(c) limx→∞ (x2 − x4 − x2 + 1) (d) limx→0 ( sin1x2 − 1
x2
)
(e) limx→1 ( ln1x − x
x−1
) (f ) limx→∞ x sin x3
1 1
1 1
(g) limx→0 ( 1−cos x
− sin2 x
) (h) limx→∞ x(e x − e x+1 )

3. Aşaǧıdaki limitleri hesaplayınız.


1
(a) limx→0 (1 + 5x) x (b) limx→0− (1 − 2x )sin x
1
(c) limx→∞ (e x + x1 )x (d) limx→ π4 (tan x)tan 2x
1
(1+x) x −e
(e) limx→0+ [ln(1 + x)]x (f ) limx→0 x


4. limx→∞ √2x−1 limitini hesaplayınız.
x+1

1
5. limx→0 (2x + 3x − 1) x =6 olduǧunu gösteriniz. Bunu genelleştirebilir
misiniz?
68 CHAPTER 3. TÜREV

3.5.2 DİǦER UYGULAMALAR

Bir y = f (x) eǧrisinin P (x0 , y0 ) noktasındaki y 0 = f 0 (x0 ) türevinin bu nok-


tadaki teǧetin eǧimi olduǧunu biliyoruz. Bu durumda teǧet ve normal denk-
lemleri sırasıyla
−1
y − y0 = f 0 (x0 )(x − x0 ) ve y − y0 = (x − x0 )
f 0 (x 0)

olacaktır.
Örnek 3.38. y = x3 + 2x2 − 4x − 4 eǧrisinin A(1, −4) noktasındaki teǧet
ve normalinin denklemini bulalım. yA0 = 3x2 + 4x − 4]A = 3 olup teǧet ve
normal sırasıyla
−1
y + 4 = 3(x − 1) ve y + 4 = (x − 1)
3
bulunur.
Şimdi, uygulamalarda karşımıza sık çıkan bir sonuç verelim.
Teorem 3.3. y = f (x) fonksiyonu x = c için türevli olsun. f 0 (c) > 0 ise f
fonksiyonu artan, f 0 (c) < 0 ise f fonksiyonu azalandır.

İspat: f 0 (c) deǧerinin işareti


f (x) − f (c)
f 0 (c) = lim
x→c x−c
limitine baǧlı olduǧundan, f 0 (c) > 0 olması pay ile paydanın aynı işaretli
olması anlamına gelir. Bu ise, x > c iken f (x) > f (c) olması veya x < c iken
f (x) < f (c) olması anlamına gelir. Bu da f ’nin artan olmasının tanımıdır.
Şu halde, f 0 (c) > 0 ise f (x) artandır. Benzer olarak f ’nin azalan olma koşulu
açıklanabilir.
Sonuç 3.8. Bir fonksiyonun türevinin işaretinin (+) olduǧu yerlerde fonk-
siyon artan, (-) olduǧu yerlerde fonksiyon azalandır.
Sonuç 3.9. Bir y = f (x) fonksiyonu verildiǧinde, eǧer x = c noktasında
yerel maksimum veya minimum (ekstremum) varsa ve f fonksiyonu x = c
noktasında türevlenebiliyorsa, f 0 (c) = 0 olur.
3.5. TÜREVİN UYGULAMALARI 69

Şekil 3.37 x = c1 ile x = c2 apsisli noktalarda yerel min/mak vardır ve


f 0 (c1 )=f 0 (c2 )=0’dır.

Not 3.6. Ancak, bu sonucun tersi doǧru deǧildir. Yani, f 0 (c)=0 ise x=c nok-
tasının bir yerel maksimum veya yerel minimum (ekstremum) olması gerek-
mez.

Örnek 3.39. y = x3 fonksiyonunun türevini 0 yapan noktalar,

f 0 (x) = 3x2 = 0 ⇒ x = 0

olur. Yani, f 0 (0) = 0’dır. Ancak, x = 0 noktası bir yerel maksimum deǧildir
çünkü, x > 0 ve x < 0 için f 0 (x) = 3x2 > 0’dır. Benzer olarak, bu noktada
yerel minimum da yoktur, çünkü x > 0 ve x < 0 için f 0 (x) = 3x2 > 0’dır.
(Yani, türevin işaretinde deǧişme yoktur; fonksiyon hep artandır.)

Not 3.7. Bir fonksiyonun herhangi bir noktada yerel ekstremum deǧerinin
olması o noktada türevlenebilir olmasını gerektirmez.

Şekil 3.38 f : R 7→ R, y = f (x) = x3 eǧrisinin yerel ekstremumu yoktur.


70 CHAPTER 3. TÜREV

Not 3.8. Bir fonksiyonun herhangi bir noktada yerel ekstremum deǧerinin
olması o noktada türevlenebilir olmasını gerektirmez.

Örnek 3.40. f (x) = |x| fonksiyonu x = 0’da bir yerel minimuma sahiptir,
ancak bu noktada türevli deǧildir.

Şekil 3.39 f : R 7→ R, y=f (x)=|x| eǧrisi.

Tanım 3.14. y=f (x) fonksiyonu verildiǧinde, f 0 (x)=0 denkleminin kökle-


rine f fonksiyonun kritik noktaları denir. Bunlar ekstremum adayı nokta-
lardır.

Örnek 3.41. f (x) = x2 − 4x + 3 fonksiyonunun kritik noktaları

f 0 (x) = 2x − 4 = 0 ⇒ x = 2

olur. f 0 fonksiyonunun işaretini aşaǧıdaki tabloda inceleyelim.

Tablo: f : R 7→ R, y=f (x)=x2 − 4x + 3 eǧrisi.

x −∞ 0 2 4 +∞
0
f (x) −∞ −4 0 +4 +∞
f (x) +∞ & 3 & -1 % 3 % +∞
(minimum)

f (2) = −1 deǧeri bir yerel minimumdur, çünkü eǧri x < 2 için azalan ve
x > 2 için artandır. Buna göre grafiǧi aşaǧıdaki gibidir.
3.5. TÜREVİN UYGULAMALARI 71

Şekil 3.40 f : R 7→ R, y=f (x)=x2 − 4x + 3 eǧrisi.

Madem ki kritik noktalar her zaman ekstremum deǧildir (y = x3 örneǧi),


bu noktaların maksimum veya minimum olmasına nasıl karar verebileceǧiz?
Aşaǧıdaki teorem ikinci türevin işareti ile ekstremumların varlıǧı arasındaki
ilişkiyi ortaya komaktadır.
Teorem 3.4. x = c noktası y = f (x) fonksiyonunun bir kritik noktası, yani,
f 0 (c) = 0 olsun.
(a) f 00 (c) > 0 ise x = c’de yerel minimum vardır.
(b) f 00 (c) < 0 ise x = c’de yerel maksimum vardır.

İspat: (a) f 0 (c) = 0 ve f 00 (c) > 0 demek,


f 0 (x)−f (c)
f 00 (c) = limx→c x−c
f 0 (x)
= limx→c x−c
>0
demektir. Bu ise, x−c > 0 için f 0 (x) > 0 (fonksiyon x = c noktasının saǧında
artan) ve x − c < 0 çin f 0 (x) < 0 (fonksiyon x = c noktasının solunda azalan)
demektir. Yani, x = c noktası bir minimum noktadır. (b) ’nin ispatı benzer
olarak yapılır.
Örnek 3.42. y = x(x − 1)2 eǧrisinin varsa yerel ekstremumlarını bulalım.
Kritik noktalar,
1
y 0 = (x − 1)(3x − 1) = 0 ⇒ x1 = ve x2 = 1
3
72 CHAPTER 3. TÜREV

olur. Bu noktalardaki ikinci türev deǧerleri,

y 00 = 6x − 4 ⇒ y 00 ]x= 1 = −2 < 0 ve y 00 ]x=1 = 2 > 0


3

olur. Buna göre, x= 13 için y= 27


4
maksimum ve x=1 için y=0 deǧeri mini-
mumdur.

Örnek 3.43. y = x eǧrisinin A(√92 , 0) noktasına en yakın olan noktasını
bulalım. Bu nokta P (x, y) ise, P (x, x) olup, |AP | = d uzaklıǧının minimum
olmasını istiyoruz. Kritik noktalar
r
9 √ 2x − 8
d = (x − )2 + ( x)2 ⇒ d0 = q =0⇒x=4
2 2 (x − 9 )2 + x
2

olur. y 00 ]x=4 = 2 > 0 olduǧundan, x = 4 minimum yapan deǧerdir. O halde,


Aranan P (x, y) noktası P (4, 2)’dir. Minimum uzaklık ise
r √
9 2 2
17
d = (4 − ) + (0 − 2) =
2 2
olur.

Örnek 3.44. 1000m uzunluǧundaki dikenli tel ile dikdörtgen şeklinde bir
bölge çevrelenecektir. En büyük alanlı bölgenin ölçülerini ve alanını bulalım.

Bölgenin eni x ve boyu y olsun. Alanı A = xy olup, çevresi 1000 = 2x + 2y


olur. Buna göre, A = x(500 − x) olup, kritik noktalar

A = x(500 − x) ⇒ A0 = 500 − 2x = 0 ⇒ x = 250 ⇒ y = 250

olur. O halde aranan bölge kare biçiminde ve alanı da A = 62 500m2 olur.

ALIŞTIRMALAR (Diǧer Uygulamalar)

1. x2 + y 2 = 1 çemberinin hangi noktası A(3, 4) noktasına en yakındır?

2. Toplamı 7 olan pozitif iki reel sayının çarpımı en çok ne olur?

3. Çarpımı 8 olan iki sayının toplamı en az kaç olur?


3.5. TÜREVİN UYGULAMALARI 73

4. Toplamları 60, negatif olmayan öyle iki sayı bulunuz ki birinin karesi
ile diǧerinin çarpımı maksimum olsun.

5. Alanları aynı olan dikdörtgenler içinde çevresi en küçük olanın kare


olduǧunu gösteriniz.

6. Aynı çevreli dikdörtgenler içinde alanı en büyük olanın kare olduǧunu


gösteriniz.

7. Negatif olmayan iki sayının toplamı t’dir. Bu sayıların kareleri topla-


mının minimum deǧeri nedir?

8. 1000m uzunluǧunda dikenli tel ile bir ahır yapmak istiyoruz. Ahırın
bir kenarı duvar, dikdörtgen şeklinde ve en büyük alanlı olmasını isti-
yorsak, boyutları ne olmalıdır?

Şekil 3.41

9. L birim uzunluǧunda olan bir tel iki parçaya bölünerek bir çember ile
bir kare yapılacaktır. Kare ile dairenin alanları toplamının en büyük
olabilmesi için çemberin yarıçapı ile karenin kenar uzunluǧu ne ol-
malıdır? Toplam alan ne olmalıdır?

10. Yarıçapı a olan bir küre içine yerleştirilebilecek dairesel silindirin hacmi
en fazla ne olur?

11. 1 litre hacminde, silindir biçimindeki bir konserve kutusunun en ekono-


mik (minimum alanlı-en az teneke harcanan-) boyutlarını hesaplayınız.

12. Bir hareketlinin hızı (v) zamana (t) baǧlı olarak v=t2 + 2t şeklinde
deǧişmektedir. Bu hareketlinin minimum hızını bulunuz.

13. Yüksek bir binanın kenarına paralel bir çit vardır. Çitin 27m yüksek-
likte ve binadan 64m uzakta olduǧunu düşünelim. Bir itfaiyeci bi-
naya çitin üzerinden dayadıǧı bir merdivenle ulaşmak istiyor. Binaya
ulaşabilecek en kısa merdiven ne kadar uzunluktadır?
74 CHAPTER 3. TÜREV

Şekil 3.42

14. f : R − {0} 7→ R, f (x)=x2 + xa veriliyor. a ne olmalıdır ki f fonksiyonu

(a) x = 2’de yerel minimuma


(b) x = −3’te yerel maksimuma
sahip olsun?

15. r yarıçaplı çember içine çizilen dikdörtgenin alanı en fazla ne olur?

16. Eşit kenarlarının uzunluǧu a olan bir ikizkenar üçgenin alanı en çok ne
olur?

17. Üst bölümü yarım daire ve buna baǧlı alt bölümü dikdörtgen olan 10m
çevreli bir pencere yapılacaktır (aşaǧı saǧdaki şekil). En çok güneş ala-
cak pencerenin boyutlarını (yarım dairenin yarıçapını ve dikdörtgenin
boyutlarını) belirleyiniz.

Şekil 3.43

18. 54m2 alan üzerinde birbirine yapışık iki dikdörtgen bölmeden oluşan
(yukarıda soldaki şekil) üstü açık bir ahır yapılmak isteniyor. En ekono-
mik boyutları belirleyiniz. Daha sonra, benzer bir problem oluşturun.
3.6. GRAFİK ÇİZİMİ 75

3.6 GRAFİK ÇİZİMİ

Biliyoruz ki her fonksiyon bir baǧıntıdır. Dolayısıyla, bir f fonksiyonu ikili-


lerden oluştuǧundan, bu ikililerin bir dik koordinat sisteminde gösterilmesi,
yani, grafiǧinin çizilmesi mümkündür.
A ⊂ R ve f : A 7→ R ile tanımlanan, f ’nin grafiǧini çizmek için çeşitli
özelliklerinin bilinmesi gerekir. Fonksiyonun artan veya azalan olduǧu ara-
lıklar, eksenleri kestiǧi noktalar, asimtotları (asimptotları), pik noktaları, ...
vb. belirlenmelidir. Özetleyecek olursak, bir fonksiyonun grafiǧini çizmek
için aşaǧıdaki bilgileri kullanırız:

1. f fonksiyonunun kendisi: Fonksiyonun x ile y eksenlerini kestiǧi nokta-


lar, fonksiyonun geçtiǧi belirli bazı noktalar, fonksiyonun asimtotları,
fonksiyonun tek veya çift olması gibi özellikleri belirlenir.

2. f ’nin türevinin işaret tablosu: Fonksiyonun ekstremum için aday nok-


taları, artan veya azalan olduǧu yerleri belirlenir.

3. f 00 ’nün işareti: Sıfır olduǧu yerler dönüm noktalarıdır. (+) olduǧu


yerlerde (ilk türev sıfır ise) minimum, (-) olduǧu yerlerde ise (yine ilk
türev sıfır ise) maksimum olacaǧından, bu tip yerler belirlenir.

Buraya kadar bize belki yabancı gelebilecek tek kavram asimtot kavramıdır.
Aşaǧıdaki şekilleri inceleyelim.

Şekil 1.44 Yatay, düşey ve eǧik asimtotlar.

İlk şekilde limx→∞ f (x) = a olduǧu görülüyor. Bu durumda y = a doǧrusu


bir yatay asimtottur deriz. Benzer olarak limx→−∞ f (x) = a ise, yine y = a
doǧrusu bir yatay asimtottur deriz. Bu durumdaki asimtotlara pay ile pay-
dası aynı dereceli polinomlar olan kesirli fonksiyonlarda ve aşaǧıda örneklerini
vereceǧimiz bazı başka fonksiyonlarda rastlanır.
76 CHAPTER 3. TÜREV

İkinci şekilde limx→b+ f (x) = ∞ olduǧu görülüyor. Bu durumda y = a


doǧrusu bir düşey asimtottur deriz. Bu durumdaki asimtotlara kesirli fonksi-
yonlarda paydanın köklerinin olması durumlarında rastlanır.
Üçüncü şekilde ise x deǧişkeni sınırsız olarak büyüdüǧünde y deǧişkeni de
büyüyorsa, bu durumda verilen eǧrinin düşey veya yatay asimtotları dışında
asimtotları bulunabilir. y = f (x) verildiǧinde,

lim [f (x) − g(x)] = 0 veya lim [f (x) − g(x)] = 0


x→∞ x→−∞

olacak şekilde bir y = g(x) eǧrisi varsa, bu eǧriye y = f (x) eǧrisinin eǧri
asimtotu denir. Eǧer bu eǧri özel olarak bir doǧru ise, buna da eǧik asimtot
denir. Bir eǧik asimtot y = g(x) = mx + n (x deǧişken, m ve n keyfi sabitler)
şeklindedir. Şimdi, asimtotun bu tipte olması halinde m ve n katsayılarının
nasıl bulunabileceǧini görelim:
f (x)
lim [f (x) − g(x)] = lim [f (x) − mx − n] = lim [ − m]x − n = 0
x→∞ x→∞ x→∞ x
olmalıdır. Bu limitin var olması için limx→∞ [ f (x)
x
− m] = 0 olmalıdır. Aksi
halde limit mevcut olmaz, çünkü ikinci çarpanın limiti sonsuzdur. Sonuç
olarak, eǧik asitotun m ile n deǧerleri
f (x)
m = lim , n = lim [f (x) − mx]
x→∞ x x→∞

eşitlikleri ile bulunur.


Aşaǧıdaki örnekleri inceleyelim.

Örnek 3.45. y = x2 + 1 eǧrisinin asimtotlarını bulalım. limx→∞ y = ∞
olduǧundan eǧik asimtot olabilir.

f (x) x2 + 1
m = lim = lim = 1 ve
x→∞ x x→∞ x
√ 6 x2 + 1− 6 x2
n = lim [f (x) − mx] = lim [ x2 + 1 − x] = lim √ =0
x→∞ x→∞ x→∞ x2 + 1 + x
bulunur. O halde, y = mx + n’den y = x bir eǧik asimtottur. Benzer olarak,
limx→−∞ y = ∞ olduǧundan bir asimtot daha bulunabilir. Gerçekten, aynı
adımlar izlenerek y = −x’in de bir eǧik asimtot olduǧu görülür. Dolayısıya
x>0 bölgesindeki asimtot y=x ve x<0 bölgesindeki asimtot da y=−x’tir.
3.6. GRAFİK ÇİZİMİ 77

1 1
Örnek 3.46. y = e x eǧrisinin varsa asimtotlarını bulalım. limx→∞ e x = 1
olduǧundan y = 1 bir yatay asimtottur.
2
Örnek 3.47. y = xx2+2x
−1
eǧrisinin yatay asimtotu y = 1 ve düşey asimtotları
x = 1 ile x = −1’dir.
x2 +2 2
Örnek 3.48. y = x
= x+ x
eǧrisinin eǧik asimtotu y = x ve düşey
asimtotu x = 0’dır.

Not 3.9. Kesirli bir fonksiyon için pay ile paydanın ortak çarpanları yokken
dereceleri eşit ise yatay asimtot, payın derecesi paydanınkinden 1 büyük ise
eǧik asimtot, payın derecesi paydanınkinden 2 veya daha büyük ise eǧri asim-
tot vardır ve yukarıdaki asimtotların tümü payın paydaya bölünmesiyle bu-
lunacak bölümden elde edilebilir.

Not 3.10. Tek bir fonksiyon orijine göre, çift bir fonksiyon ise y eksenine
göre simetriktir.

Şimdi bir y = f (x) fonksiyonunun grafiǧini çizerken yapmamız gerekenleri


tek tek sıralamaya hazırız. Bunları maddeleyelim:

1. f (x)’in tanım kümesi bulunur ve aşaǧıdaki özellikleri saptanır:

(a) Kesirli bir fonksiyon ise, düşey asimtotlar varsa saptanır. Bunun
için paydayı sıfır yapan x deǧerleri bulunur.
(b) lim±x→∞ f (x) limitine bakılarak yatay veya eǧik asimtotlar varsa
saptanır.
(c) f ’nin tek veya çift oluşu incelenerek herhangi bir simetri varsa
saptanır.
(d) Eksenleri kesen noktalar varsa belirlenir.

2. f 0 (x) ile f 00 (x) bulunur ve bunlar mümkünse çarpanlara ayırılır.

3. f 0 türevi hesaplanarak aşaǧıdakiler saptanır:

(a) Kökleri varsa kritik noktalar (maksimum veya minimum adayı


noktalar) belirlenir.
(b) f 0 ’nün işareti incelenerek f ’nin artan veya azalan olduǧu yerler
belirlenir.
78 CHAPTER 3. TÜREV

4. f 00 ve işareti incelenerek şunlar saptanır:

(a) Sıfır olduǧu yerler dönüm noktalarıdır.


(b) İşareti incelenerek içbükey veya dışbükey olan aralıklar belirlenir.
(c) (+) olduǧu yerlerde (ilk türev sıfır ise) minimum, (-) olduǧu yer-
lerde ise (yine ilk türev sıfır ise) maksimum olacaǧından, bu tip
yerler belirlenir.
2
Örnek 3.49. y = x +2x+4
2x
eǧrisinin grafiǧini çizelim. y’yi aşaǧıdaki biçimde
yazarak hem eǧik asimtotunu hem de türevlerini kolaylıkla belirleyebiliriz:

1 2 1 2 4
y = x + 1 + ⇒ y 0 = − 2 ⇒ y 00 = 3
2 x 2 x x

(A) y’den elde edebildiǧimiz bilgiler: x = 0 için tanımsız olup, diǧer her
yerde tanımlıdır. x = 0 düşey asimtot, y = x2 + 1 eǧik asimtottur. Fonksiyon
tek veya çift deǧildir.
x2 −4
(B) y 0 ’den elde edebildiǧimiz bilgiler: y 0 = 2x2
= 0 ⇒ x = ±2 kritik
noktalardır.

(C) y 00 ’den elde edebildiǧimiz bilgiler: Sıfır olduǧu yer yoktur. y ekseninin
saǧında pozitif, solunda ise negatiftir. Dolayısıyla, x = −2 için maksimum,
x = 2 için ise minimum vardır.

Bu bilgileri aşaǧıdaki tablo üzerine yerleştirebiliriz:

2 +2x+4
Tablo: y= x 2x
eǧrisine ilişkin bilgiler.

x −∞ −2 0 2 +∞
y0 + 0 − tanımsız − 0 +
y 00 − − − tanımsız + + +
y −∞ % −1 & −∞k + ∞ &3% +∞
MAKS Düşey Asm. MİN

Bu tablodaki bilgileri ise aşaǧıdaki grafik üzerinde gösterebiliriz.


3.6. GRAFİK ÇİZİMİ 79

2 +2x+4
Şekil 1.45 y= x 2x
eǧrisi.
x2 −1
Örnek 3.50. y = x2 −4
eǧrisinin grafiǧini çizelim.

3 0 −6x 00 6(3x2 + 4)
y =1+ ⇒ y = ⇒ y =
x2 − 4 (x2 − 4)2 (x2 − 4)3
(A) y’den elde edebildiǧimiz bilgiler: x = ±2 için tanımsız olup, diǧer her
yerde tanımlıdır. x = −2 ile x = 2 düşey asimtot, y = 1 yatay asimtottur.
Fonksiyon çift olup, y eksenine göre simetriktir. (0, 1/4), (−1, 0) ve (1, 0)
noktalarında aksenleri kesmektedir. Fonksiyon ayrıca, (3, 8/5) ve (−3, 8/5)
noktalarından geçmektedir.
(B) y 0 ’den elde edebildiǧimiz bilgiler: y 0 = (x−6x
2 −4)2 ⇒ x = 0 kritik noktadır.
00
(C) y ’den elde edebildiǧimiz bilgiler: Sıfır olduǧu yer yoktur. (−2, 2) aralıǧında
negatif, −2 ile 2 haricindeki yerlerde fonksiyon pozitiftir. Dolayısıyla, x = 0
için maksimum vardır. Bu bilgileri aşaǧıdaki tablo üzerine yerleştirebiliriz:
2
Tablo: y= xx2 −1
−4
eǧrisine ilişkin bilgiler.

x −∞ −2 0 2 +∞
y0 + tanımsız + 0 − tanımsız −
y 00 + tanımsız − − − tanımsız +
y 1 % +∞k − ∞ % % 41 & & −∞k + ∞ &1
Düşey Asm. MAKS
80 CHAPTER 3. TÜREV

Bu tablodaki bilgileri ise aşaǧıdaki grafik üzerinde gösterebiliriz.

x2 −1
Şekil 1.46 y = x2 −4
eǧrisi.

ALIŞTIRMALAR (Grafik Çizimleri)


(A) Aşaǧıdaki fonksiyonların grafiklerini çiziniz.
x3
(1)y = x(3 − x2 ) (2)y = x2 −1
−1
2−x
(3)y = 2 x2 −1 (4)y = x

(5)y = (x2 − 1)3 (6)y = x(x2 − 1)2


x−1 1
(7)y = x+1
(8)y = x2 +4
x3 x
(9)y = 1+x
(10)y = x2 −1
x2 x3
(11)y = x2 −1
(12)y = x2 −1
x3 x2
(13)y = x2 +1
(14)y = x2 +1
ln x
(15)y = x + sin x (16)y = x
, (x > 0)

sin x
(B) y= 1+x 2 eǧrisinin asimtotu nedir? Eǧri asimtotu hangi noktalarda keser?
Chapter 4

BELİRSİZ İNTEGRAL

Bu bölümde belirsiz integral ve integral alma yöntemleri üzerinde duracaǧız.

4.1 GİRİŞ
Elimizde bir fonksiyon olduǧunda bunun nasıl bir fonksiyonun türevi olaca-
ǧını bulmak isteyebiliriz. Aslında bu problem düzlemsel bölgelerin alanlarını
hesaplama probleminde karşımıza çıkmaktadır. Hangi fonksiyonun türevi
ex ’tir? Benzer olarak, sin x hangi fonksiyonun türevidir? Bu soruları çözme
işi ters türev alma veya integral alma olarak bilinir. Türev ile birlikte in-
tegral işlemi de fen ve mühendislikte çok güzel problemleri olan diferensiyel
denklemler için de çok önemli bir araçtır.
Tanım 4.1. (Belirsiz integral) Türevi f (x), başka bir deyişle diferensiyeli
f (x)dx olan F (x) ifadesine f (x)’in belirsiz integrali denir ve
Z
F (x) = f (x)dx

şeklinde gösterilir.
Sonuç 4.1. Bu tanımdan F 0 (x) = f (x) olduǧu, böylece, C keyfi bir sabit
olmak üzere
Z
0 0
(F (x) + C) = F (x) = f (x) ⇔ F (x) = f (x)dx

olduǧu anlaşılmaktadır. Buradaki C sabitine integral sabiti denir. Bu sabit,


türevi 0 olan herhangi bir sabit olabilir.

81
82 CHAPTER 4. BELİRSİZ İNTEGRAL
R R
Örnek 4.1. 2xdx = x2 + C veya 2xdx = x2 + ln C yazılabilir.
Yukarıdaki belirsiz integral tanımına dayanarak şu özellikleri verebiliriz.
Özellik 4.1. Her a ∈ R sabiti için
Z Z
af (x)dx = a f (x)dx

yazılabilir. Gerçekten,
Z Z
F (x) = f (x)dx ⇒ aF (x) = a f (x)dx (1)

ve
F 0 (x) = f (x) ⇒ aF 0 (x) = af (x) ⇒
Z
0
[aF (x)] = af (x) ⇒ aF (x) = af (x)dx (2)

olup, (1) ve (2)’den dolayı istenen özellik görülür.


Özellik 4.2.
R
F1 (x) + C1 = f1 (x)dx
R
F2 (x) + C2 = f2 (x)dx
..
.
R
Fn (x) + Cn = fn (x)dx

R R R
F1 (x)+F2 (x)+· · ·+Fn (x)+C = f1 (x)dx+ f2 (x)dx+· · ·+ fn (x)dx (1)

yazılabilir. Öte yandan,

[F1 (x) + C1 ]0 = f1 (x)


[F2 (x) + C2 ]0 = f2 (x)
..
.
[Fn (x) + Cn ]0 = fn (x)

[F1 (x)+F2 (x)+· · ·+Fn (x)+C]0 = f1 (x)+f2 (x)+· · ·+fn (x)


4.1. GİRİŞ 83

ve böylece
Z
F1 (x)+F2 (x)+· · ·+Fn (x)+C = [f1 (x)+f2 (x)+· · ·+fn (x)] dx (2)

yazılabilir. Şimdi, (1) ve (2) denklemlerinden


Z Z Z Z
[f1 (x)+f2 (x)+· · ·+fn (x)] dx = f1 (x)dx+ f2 (x)dx+· · ·+ fn (x)dx

bulunur. Bunun anlamı, fonksiyonların toplamlarının integrali, integralleri


toplamıdır.
Şimdi, integral alma tanımından yola çıkarak, türev alma konusunda buldu-
ǧumuz baǧıntılar yardımıyla şu integral formüllerini yazabiliriz:

1
R R
1. xm dx = m+1
xm+1 +C 6. cos xdx = sin x + C

dx dx
R R
2. x
= ln x + C 7. sin2 x
= − cot x + C

1 x dx
R R
3. ax dx = ln a
a +C 8. cos2 x
= tan x + C

eax dx = a1 eax + C dx
R R
4. 9. 1+x2
= arctan x + C

√ dx
R R
5. sin xdx = − cos x + C 10. 1−x2
= arcsin x + C

Örnek 4.2. Z
1 1
(x2 + x + 1)dx = x3 + x2 + x + C
3 2
Örnek 4.3.
Z √
3 4 3 5
(2 sin x − x2 + )dx = −2 cos x − x 3 + 4 ln x + C
x 5

Örnek 4.4.
x3 + x √ 
Z Z 
5 2 7 2 3
√ dx = x2 + x dx = x 2 + x 2 + C
x 7 3
84 CHAPTER 4. BELİRSİZ İNTEGRAL

Ödev 4.1. Hiperbolik fonksiyonlar diye aşaǧıda tanımladıǧımız fonksiyon-


lara denir. Bunların trigonometrik fonksiyonlardaki özelliklere benzer özellik-
lerini araştırınız. Bunlardan ilk ikisinin integralini bulunuz.

ex +e−x ex −e−x cosh x sinh x


cosh x := , sinh x := , coth x := , tanh x :=
2 2 sinh x cosh x

ALIŞTIRMALAR (Giriş)
Aşaǧıdaki integralleri hesaplayınız.
R
1. (a + x)2 dx =?
R √
2. (1 − x)3 xdx =?

x5 −2x4 +1
R
3. x2
dx =?

x3 −4x2 +1
R
4. 1+x2
dx =?

R 
5. 2 sinh x + √ 3 dx =?
1−x2

R
6. (1 + x3 )2 x2 dx =?
R
7. (2x + x2 )dx =?
R
8. (1 + x)4 dx =?

4.2 İNTEGRAL ALMA YÖNTEMLERİ

İntegrali alınacak fonksiyonun hangi fonksiyonun türevi oduǧunu belirle-


menin mümkün olmadıǧı, örneǧin, yukarıdaki 10 integral formülünün doǧru-
dan uygulanamadıǧı birçok durum karşımıza çıkar. Böyle durumlarda kulla-
nacaǧımız birkaç teknik vereceǧiz. Bunlar deǧişken deǧiştirme, kısmi integ-
rasyon, indirgeme formülleri, basit kesirlere ayırma ve binom integralleridir.
4.2. İNTEGRAL ALMA YÖNTEMLERİ 85

4.2.1 Deǧişken Deǧiştirme

Aşaǧıda integral almada deǧişken deǧiştirme şekillerinden sıkça karşılaşılan-


lar verilmiştir.
(A) f (u(x))u0 (x)dx integrali:
R

Bu integralde u(x) = t iken dt = u0 (x)dx deǧişken deǧişimi yapılarak


Z
f (t)dt

integrali bulunur.

Örnek 4.5. Z
2xdx
I=
1 + x2
integralinde 1 + x2 = t, dt = 2xdx deǧişken deǧişimi yapılırsa,
Z
dt
I= = ln t + C = ln(1 + x2 ) + C
t

bulunur.
R
Örnek 4.6. (1 + x)10 dx integralinde 1 + x = t, dt = dx deǧişken deǧişimi
uygulanıp
Z Z
10 1 11 1
(1 + x) dx = t10 dt = t + C = (1 + x)11 + C
11 11

bulunur.
R sin x R
Örnek 4.7. I = cos x
dx = tan xdx integrali için, cos x = t, dt = − sin xdx
dönüşümü uygulanırsa,

−dt
Z
I= = − ln t + C = − ln(cos x) + C
t

bulunur.
R
Uygulamalarda karşımıza sık çıkan bir başka örnek de sec xdx integrelidir.
86 CHAPTER 4. BELİRSİZ İNTEGRAL
R
Örnek 4.8. I = sec xdx integrali için,
Z Z
sec x sec x(sec x + tan x)
I= dx = dx
1 sec x + tan x
yazar ve
t = sec x + tan x, dt = (sec x tan x + sec2 x)dx
dönüşümünü uygularız. Bu durumda,
Z Z
dt
I = sec xdx = = ln t + C = ln(sec x + tan x) + C
t
bulunur.

ALIŞTIRMALAR (Deǧişken deǧiştirme-A)

1. Aşaǧıdaki integralleri uygun bir deǧişken deǧiştirme yardımıyla hesap-


layınız.
R√
1 − 3xdx (c) 2xxdx
R R
(a) (1 + 2x)7 dx (b) 2 +1
R 2x+2 R dx
R dt
(d) x2 +4x+4 dx (e) √x√1−x (f ) t ln t

2. Aşaǧıdaki integralleri hesaplayınız.


R sin xdx
(b) arctan xdx
R R
(a) 1+cos 2x 1+x2
(c) cos3 x sin xdx
R R R dx
(d) cos3 xdx (e) cos2 xdx (f ) 1+cos x

3. Aşaǧıdaki integralleri hesaplayınız.


R √x
(a) ecos x sin xdx (b) e−3x dx (c) e √xdx
R R
R x
(d) ee2xdx
R dx R
+1
(e) 1+e x (f ) (2 + 3ex )2006 ex dx

Aşaǧıdaki integrallerde yaptıǧımız dönüşümlerin amacı, ifadeleri kök içinden


kurtarmaktır. Kök içinde ikinci dereceden her ifade bu integrallere dönüştü-
rülebildiǧinden, bunların iyi bilinmesi önemlidir. Şimdi bunları inceleyelim.


(B) a2 − x2 ’li integraller: (x = a sin t dönüşümü yapılır.)
4.2. İNTEGRAL ALMA YÖNTEMLERİ 87
R dx
Örnek 4.9. I = √4−x 2 integralinde x = 2 sin t, dx = 2 cos tdt dönüşümü
yapılırsa,
Z Z
2 cos tdt x
I= p = dt = t + C = arcsin + C
2
4(1 − sin t) 2

bulunur.
√ dx
R
Örnek 4.10. I = 4−3x2
integralini hesaplayalım. Bu integral,
Z
1 dx
I=√ q
3 4
− x2
3

şeklinde yazılabilir. Burada x = √2 sin t dönüşümü yapılırsa


3

√2

1
Z
3
cos tdt 1 1 3
I=√ q = √ t + C = √ arcsin x+C
3 4
− 4
sin 2
t 3 3 2
3 3

bulunur.


(C) x2 − a2 ’li integraller: (x = a sec t dönüşümü yapılır.)
dx
R
Örnek 4.11. I = √x2 +2x−3 integralini hesaplayalım. Bunun için kök içini
tam kare yapıp, Z
dx
I= p
(x + 1)2 − 4
buluruz. x + 1 = 2 sec t, dx = 2 sec t tan tdt dönüşümünü yaparak da,
Z Z
2 sec t tan tdt
I= √ = sec tdt = ln(sec t + tan t) + C
4 sec2 t − 4
buluruz. Burada tan2 t = sec2 t − 1 olduǧunu göz önüne alalım.√ Şimdi,
2
x + 1 = 2 sec t ise x+12
= sec t olup, dik üçgen yardımıyla tan t = x +2x−3
2
deǧeri yerine yazılırsa,

x+1 x2 + 2x − 3
I = ln( + )+C
2 2
bulunur.
88 CHAPTER 4. BELİRSİZ İNTEGRAL


(D) a2 + x2 ’li integraller: (x = a tan t dönüşümü yapılır.)

√dx
R
Örnek 4.12. I = x2 x2 +4
integralinde x = 2 tan t dönüşümü yapılırsa
Z 2dt Z
cos2 t 1 cos tdt
I= √ =
4 tan2 t 4 tan2 t + 4 4 sin2 t

bulunur. Burada da sin t = u dönüşümü yapılırsa


Z
1 du 1
I= 2
=− +C
4 u 4 sin t
x √ x
bulunur. Dik üçgen yardımıyla 2
= tan t ise sin t = x2 +4
olup,

x2 + 4
I=− +C
4x
bulunur.

ALIŞTIRMALAR (Deǧişken deǧiştirme-B/C/D)

1. Aşaǧıdaki integralleri hesaplayınız.


R dx R x dx
(b) √21−4 (c) (5−xdx2 )3/2
R
(a) √4x−x 2 x
R dx R√ R √
(d) √2x−x 2 (e) 5 − 3x2 dx (f ) x 9 − x2 dx

2. Aşaǧıdaki integralleri hesaplayınız.


(x−1)dx
√ xdx dx
R R R
(a) x2 −2x+5
(b) √
x2 −4x+3
(c) √
x2 −2x−8

3. Aşaǧıdaki integralleri hesaplayınız.


dx
(b) x2 √dx xdx
R R R
(a) (x2 +1) 3/2 2
x +4
(c) 4x2 +12x+13


(E) ni
ax+b’li integraller: (ni ’lerin ekok’u n ise, ax+b = tn yazılır.)
4.2. İNTEGRAL ALMA YÖNTEMLERİ 89

Örnek 4.13. Z √
x+1+1
I= √
3
dx
x+1
integralini hesaplayalım. Kök kuvvetlerinin okek’i okek(2,3)=6 olduǧundan,
x + 1 = t6 ve dx = 6t5 dt dönüşümü yapılırsa,

t6 + 1 5
Z
6 6
I= √ 3 6
6t dt = 6 (t6 + t3 )dt = t7 + t4 + C
t 7 4

ve t = 6 x + 1 deǧeri yerine konursa,
6 7 6 4
I = (x + 1) 6 + (x + 1) 6 + C
7 4
bulunur.

(F) t = tan x2 dönüşümü: Polinom fonksiyonlar ile trigonometrik fonk-


siyonlar arasındaki deǧişken deǧiştirmeyi yapmak için bu dönüşüm yapılır.
Buna göre,
2dt
x = 2 arctan t ⇒ dx =
1 + t2
ve dik üçgen yardımıyla

x x 2t 1 − t2
sin x = 2 sin cos = ; cos x =
2 2 1 + t2 1 + t2
deǧerleri bulunur.
R dx
Örnek 4.14. I = 1+sin x
integralini hesaplayalım. Bunun için bir çözüm
x
yolu t = tan 2 dönüşümünü yapmaktır. Bu durumda,
2dt
−2
Z Z
1+t2 dt
I= 2t =2 2
= +C
1 + 1+t 2 (1 + t) 1+t

ve t deǧeri yerine konarak,


−2
I= +C
1 + tan x2

bulunur.
90 CHAPTER 4. BELİRSİZ İNTEGRAL

ALIŞTIRMALAR (Deǧişken deǧiştirme-E/F)

1. Aşaǧıdaki integralleri hesaplayınız.


√ √
3 x+ √
4x
dx√ xdx
R R R
(a) √
x+1+ 4 x+1
(b) √ √
x− 3 x
(c) √ 6 x dx
4 x+ √

2. Aşaǧıdaki integralleri hesaplayınız.


R dx
(b) cos 1+sin x 1+sin x
R R
(a) 1+cos x x(1+cos x)
dx (c) 1+cos x
dx

4.2.2 Kısmi integrasyon

İki fonksiyonun çarpımının türevinden hareketle kullanılan bir tekniktir. Bil-


diǧimiz gibi, iki fonksiyonun çarpımının diferensiyeli d(u · v) = u · dv + v · du
ve buradan
u · dv = d(u · v) − v · du
yazılabilir. Şimdi, iki tarafın integrali alınırsa,
Z Z
u · dv = u · v − v · du (∗)

yazılabilir. Kısmi integrasyondan anladıǧımız, bu baǧıntının kullanıldıǧı in-


tegral almadır.
R
Örnek 4.15. ln xdx integralini hesaplayalım.
 
1
ln x = u, dx = dv ⇒ du = dx, x = v
x

olduǧundan, bu deǧerler (∗)’da yerine yazılırsa,


Z Z
dx
ln xdx = x ln x − x · = x ln x − x + C
x

bulunur.
4.2. İNTEGRAL ALMA YÖNTEMLERİ 91
R
Örnek 4.16. arctan xdx integralini hesaplayalım. Yine,
 
dx
arctan x = u, dx = dv ⇒ du = , x=v
1 + x2

olup (∗)’dan yararlanarak


Z Z
xdx 1
arctan xdx = x arctan x − 2
= x arctan x − ln(1 + x2 ) + C
1+x 2

bulunur.
R
Örnek 4.17. xex dx integralini hesaplayalım. Burada,

[x = u, ex dx = dv ⇒ du = dx, v = ex ]

olup (∗) eşitliǧinden yararlanarak,


Z Z
I = xe dx = xe − ex dx = xex − ex + C = ex (x − 1) + C
x x

bulunur.
R
Örnek 4.18. x2 sin xdx integralini hesaplayalım.
 2 
x = u, sin xdx = dv ⇒ du = 2xdx, v = − cos x

dönüşümü yapılırsa,
Z Z
2 2
I = x sin xdx = −x cos x + 2 x cos xdx

ve
[x = u, cos xdx = dv ⇒ du = dx, v = sin x]
dönüşümü yapılarak,
Z
2
I = −x cos x+2[x sin x− sin xdx+C] = −x2 cos x+2x sin x+2 cos x+C

bulunur.
92 CHAPTER 4. BELİRSİZ İNTEGRAL

Bazen de kısmi integrasyonun uygulanması doǧrudan bir sonuç vermeyebilir.


Bunlara örnek,
Z Z Z Z
ax ax
e sin bxdx, e cos bxdx, cos ax sin bxdx, cos ax cos bxdx

şeklindeki ve indirgemeler konusunda göreceǧimiz integrallerdir. Bu du-


rumda, kısmi integrasyon uygulanarak, aranan integralin aynısı eşitliǧin saǧ
tarafında bulunarak integral alınabilir.
R
Örnek 4.19. I = eax sin bxdx (a, b belirli sabitler) integralini bulalım:
 
ax ax 1
e = u, sin bxdx = dv ⇒ du = ae dx, v = − cos bx
b
ve Z
1 ax a
I = − e cos bx + eax cos bxdx
b b
bulunur. İkinci kez kısmi integrasyon uygulanırsa,
 
ax ax 1
e = u, cos bxdx = dv ⇒ du = ae dx, v = sin bx
b
olup,  Z 
1 a 1 ax a
I = − eax cos bx + e sin bx − ax
e sin bxdx
b b b b
ve  
1 ax a 1 ax a
I = − e cos bx + e sin bx − I ⇒
b b b b
ax
e
I= 2 (a sin bx − b cos bx) + C
a + b2
bulunur.
R
Örnek
R 4.20. Son olarak, I = sec3 xdx integralini hesaplayalım. Burada,
I = sec x sec2 xdx eşitliǧi ve aşaǧıdaki dönüşüm kullanılırsa,
sec x = u, sec2 xdx = dv ⇒ du = sec x tan xdx, v = tan x
 
Z Z
I = sec x tan x − tan x sec xdx = sec x tan x − (sec2 x − 1) sec xdx ⇒
2

Z
1 1
I = sec x tan x−I + sec xdx ⇒ I = sec x tan x+ ln(sec x+tan x) + C
2 2
bulunur.
4.2. İNTEGRAL ALMA YÖNTEMLERİ 93

ALIŞTIRMALAR (Kısmi integrasyon)


1. Aşaǧıdaki integralleri hesaplayınız.
R R√ R
(a) (ln x)2 dx (b) x ln xdx (c) x2 ln xdx
(d) ln(ln x)
R R R
x
dx (e) x(ln x)3 dx (f ) sin(ln x)dx
R R √ R
(g) ln(4 + x2 )dx (h) x ln x + 2dx (i) x ln(x3 + x)dx

2. Aşaǧıdaki integralleri hesaplayınız.


R R R
(a) arccot xdx (b) arcsin xdx (c) arccos xdx
R R R
(d) arcsec xdx (e) arccsc xdx (f ) x sec2 xdx
R R R
(g) xarcsec xdx (h) sin 3x cos 2xdx (i) x2 sinh xdx

3. Aşaǧıdaki integralleri hesaplayınız.


(b) (x2 − 2x + 5)e−x dx
R R R
(a) x2 e3x dx (c) x sin 2xdx
R R R
(d) x arcsin xdx (e) (x2 + 5x + 6) cos 2xdx (f ) x arctan xdx
R √
(g) x1 ln(ln x)dx (h) sin 2x sin 3xdx
R R
(i) e x dx

4.2.3 İndirgemeler
Bu kesimde, kısmi integrasyonun kullanıldıǧı indirgeme baǧıntıları üzerinde
duracaǧız. Özellikle bazı fonksiyonların yüksek kuvvetlerinin integrallerini
bulmak için bu yöntem kaçınılmazdır. Şimdi bunlardan bazılarını hesapla-
mak için kısmi integrasyon yönteminin nasıl uygulandıǧını görelim.
(A) In = sinn xdx (n ∈ N+ ) integrali:
R

In = sinn xdx
R

= sinn−1 x sin xdx


R

olup,

u = sinn−1 x, dv = sin xdx ⇒ du = (n − 1) sinn−2 x cos xdx, v = − cos x


 
94 CHAPTER 4. BELİRSİZ İNTEGRAL

dönüşümü yapılırsa,

In = − cos x sinn−1 x + (n − 1) cos x sinn−2 x cos xdx


R

= − cos x sinn−1 x + (n − 1) sinn−2 x(1 − sin2 x)dx


R

= − cos x sinn−1 x + (n − 1) sinn−2 xdx − (n − 1)In


R

ve buradan
n−1
Z
1
In = − cos x sinn−1 x + sinn−2 xdx
n n
bulunur.

Örnek 4.21. Yukarıdaki indirgeme formülünden yararlanarak I = sin6 xdx


R

integralini hesaplayalım:

I = − 16 cos x sin5 x + 56 sin4 xdx


R

= − 61 cos x sin5 x + 56 [− 14 cos x sin3 x + 34 sin2 xdx]


R

= − 61 cos x sin5 x − 245


cos x sin3 x + 15 [− 21 cos x sin x + 12 sin0 xdx]
R
24

= − 61 cos x sin5 x − 5
24
cos x sin3 x − 15
48
cos x sin x + 15
48
x +C

bulunur.

sin7 xdx integrali yukarıdaki indirgeme formülü ile he-


R
Örnek 4.22. I =
saplanıp
1 6 8 16
I = − cos x sin6 x − cos x sin4 x − cos x sin2 x − cos x + C
7 35 35 35
bulunabilir. Ancak, ikinci bir yol olarak,

I = sin6 x sin xdx


R

= (sin2 x)3 sin xdx


R
R
= (1 − cos2 x)3 sin xdx

ve
[u = cos x ⇒ −du = sin xdx]
4.2. İNTEGRAL ALMA YÖNTEMLERİ 95

dönüşümü yapılarak
R
I = (1 − u2 )3 (−du)
R
= (−1 + 3u2 − 3u4 + u6 )du
= −u + u3 − 53 u5 + 71 u7 + C
= − cos x + cos3 x − 53 cos5 x + 17 cos7 x + C

bulunur. (n tek iken bu durumu genelleştirebilirsiniz).


R
Ödev 4.2. Benzer olarak In = cosn xdx (n ∈ N+ ) integrali için indirgeme
baǧıntısının
1 n−1
In = sin x cosn−1 x + In−2
n n
olacaǧını gösteriniz. (n tek iken bu integrali hesaplamak için bir yöntem
öneriniz.)

dx
R
(B) In = cosn x
(n ∈ N+ , n 6= 1) integrali:

In = cosdxn x
R

1 dx
R
= cosn−2 x cos2 x
R
= cos2−n x sec2 xdx

olup,
u = cos2−n x, dv = sec2 xdx ⇒ du = (2 − n) cos1−n x(− sin x)dx, v = tan x
 

dönüşümü yapılırsa,
R
In = cos2−n x tan x − (n − 2) tan x cos1−n x sin xdx
R (1−cos2 x)dx
= cossin x
n−1 x − (n − 2) cosn x
sin x
= cosn−1 x
− (n − 2)In + (n − 2)In−2

ve buradan
1 sin x n−2
In = n−1
+ In−2
n − 1 cos x n−1
bulunur.
96 CHAPTER 4. BELİRSİZ İNTEGRAL

dx
R
Örnek 4.23. I3 = cos3 x
integralini hesaplayalım. Yukarıdaki indirgeme
formülü uygulanırsa,
1 sin x
I3 = 2 cos2 x
+ 12 I1
1 sin x
+ 12 dx
R
= 2 cos2 x cos x
1 sin x
= 2 cos2 x
+ 12 ln(sec x + tan x) + C

bulunur.
dx
R
Ödev 4.3. Benzer olarak In = sinn x
(n ∈ N+ , n 6= 1) integrali için in-
dirgeme baǧıntısının
− cos x n−2
In = n−1 + In−2
(n − 1) sin x n−1
olacaǧını gösteriniz. n çift iken bu integrallerin hesaplanabilmesi için başka
bir yöntem önerebilir misiniz?
R
(C) In = tann xdx (n ∈ N+ , n 6= 1) integrali:
R
In = tann xdx
R
= tann−2 x(tan2 x + 1 − 1)dx
R
= tann−2 x(1 + tan2 x)dx − In−2
1
= n−1
tann−1 x − In−2

bulunur.
R
Örnek 4.24. I4 = tan4 xdx integralini hesaplayalım. Yukarıda buldu-
ǧumuz indirgeme formülü kullanılırsa,
I4 = 31 tan3 x − I2
= 13 tan3 x − tan x + I0
= 31 tan3 x − tan x + dx
R

= 13 tan3 x − tan x + x + C

bulunur.
4.2. İNTEGRAL ALMA YÖNTEMLERİ 97
R
Ödev 4.4. Benzer olarak In = cotn xdx (n ∈ N+ , n 6= 1) integrali için
indirgeme baǧıntısının
1
In = − cotn−1 x − In−2
n−1
olacaǧını gösteriniz. Bundan yararlanarak I5 integralini hesaplayınız. Ayrıca
I1 integralini hesaplayınız.

(D) In,m = sinn x cosm xdx (n, m ∈ N+ ) integrali:


R

In,m = sinn x cosm xdx


R

= sinn x cosm−1 x cos xdx


R

olup,
[u = cosm−1 x, dv = sinn x cos xdx ⇒
1
du = (m − 1) cosm−2 x(− sin x)dx, v = sinn+1 x]
n+1
dönüşümü yapılırsa,
1
sinn+1 x cosm−1 x + m−1 sinn+1 x sin x cosm−2 xdx
R
In,m = n+1 n+1
1
sinn+1 x cosm−1 x + m−1 sinn x(1 − cos2 x) cosm−2 xdx
R
= n+1 n+1

= 1
n+1
sinn+1 x cosm−1 x + m−1
I
n+1 n,m−2
− m−1
I
n+1 n,m

ve buradan In,m çekilirse,


1 m−1
In,m = sinn+1 x cosm−1 x + In,m−2
n+m n+m
bulunur. Burada, indirgeme formülünü
R ardışık uyguladıǧımızda, m çift iken
n n
R
sin xdx integraline ve m tek iken sin x cos xdx integraline ulaşırız. Bun-
ların ilkini (A)’da nasıl hesaplayacaǧımızı gördük. İkincisini ise deǧişken
deǧiştirme ile kolaylıkla hesaplayabiliriz.
Örnek 4.25. I3,3 = sin3 x cos3 xdx integralini hesaplayalım:
R

I3,3 = 16 sin4 x cos2 x + 62 sin3 x cos xdx


R

= 61 sin4 x cos2 x + 62 ( 14 sin4 x) + C


= 61 sin4 x cos2 x + 1
12
sin4 x + C
98 CHAPTER 4. BELİRSİZ İNTEGRAL

bulunur.
Not 4.1. Burada olduǧu gibi, In,m integralinde n veya m’den en az birinin
tek olması durumunda ikinci bir yol olarak, basit bir deǧişken dönüşümü çok
daha kolaydır. Bunu yukarıdaki örnek üzerinde görelim:
Z
I3,3 = sin3 x(1 − sin2 x) cos xdx

olup, [u = sin x, du = cos xdx] dönüşümü yapılırsa, kolayca


I3,3 = sin3 x(1 − sin2 x) cos xdx
R
R
= u3 (1 − u2 )du
R
= (u3 − u5 )du
= 41 u4 − 16 u6 + C
= 14 sin4 x − 61 sin6 x + C

bulunur.
Ödev 4.5. Benzer olarak sinn x cosm xdx (n ∈ N+ ) integrali için sin fonksi-
R

yonunu indirgeyecek baǧıntının


1 n−1
In,m = − sinn−1 x cosm+1 x + In−2,m
n+m n+m
olacaǧını gösteriniz.
dx
R
(E) In = (a2 +x 2 )n (n ∈ N+ , n 6= 1) integrali:
Bunun için In−1 = (a2 +xdx2 )n−1 integralinden başalayacaǧız. Burada,
R

u = (a2 + x2 )1−n , dv = dx ⇒ du = (1 − n)(a2 + x2 )−n (2xdx), v = x


yazılırsa,
In−1 = (a2 + x2 )1−n x + 2(n − 1) (a2 + x2 )−n (x2 )dx
R

= (a2 + x2 )1−n x + 2(n − 1) (a2 + x2 )−n (a2 + x2 − a2 )dx


R

= (a2 +xx2 )n−1 + 2(n − 1) (a2 +xdx2 )n−1 − 2(n − 1)a2 (a2 +x
dx
R R
2 )n

x
= (a2 +x2 )n−1
+ 2(n − 1)In−1 − 2a2 (n − 1)In
4.2. İNTEGRAL ALMA YÖNTEMLERİ 99

ve In çekilerek,

1 x 2n − 3
In = + 2 In−1
2a2 (n − 1) (a2 2
+x ) n−1 2a (n − 1)

bulunur.

dx
R
Örnek 4.26. Yukarıdaki indirgeme formülünü kullanarak I3 = (1+x2 )3
in-
tegralini hesaplayalım.

1 x 2·3−3
I3 = 2(3−1) (1+x2 )3−1
+ I
2(3−1) 2
h i
1 x 3 1 x 2·2−3
= 4 (1+x2 )2
+ 4 2(2−1) (1+x2 )2−1
+ I
2(2−1) 2−1
h i
1 x 3 1 x 1 dx
R
= 4 (1+x2 )2
+ 4 2 (1+x2 )
+ 2 1+x2

x 3x
= 4(1+x2 )2
+ 8(1+x2 )
+ 38 arctan x + C

bulunur.

Ödev 4.6. Yukarıdaki (E) indirgeme formülünü, x = a tan t dönüşümü


yardımıyla, bildiǧiniz bir trigonometrik indirgemeye dönüştürünüz.

R
(F) In,m = xn (ln x)m dx (n, m ∈ N+ ) integrali:

xn+1
 
m n m m−1
u = (ln x) , dv = x ⇒ du = (ln x) dx, v =
x n+1

dönüşümü yapılırsa,

1 m
R
In,m = n+1
xn+1 (ln x)m − n+1
xn (ln x)m−1 dx
= 1
n+1
xn+1 lnm x − m
I
n+1 n,m−1

bulunur.
100 CHAPTER 4. BELİRSİZ İNTEGRAL
R
Örnek 4.27. I3,2 = x3 (ln x)2 dx integralini, yukarıdaki indirgeme formülü
yardımıyla hesaplayalım.
I3,2 = 14 x4 ln2 x − 42 I3,1
= 14 x4 ln2 x − 21 [ 14 x4 ln x − 14 I3,0 ]
= 14 x4 ln2 x − 81 x4 ln x + 18 x3 dx
R

= 41 x4 ln2 x − 81 x4 ln x + 1 4
32
x +C

bulunur.

ALIŞTIRMALAR (İndirgemeler)
1. Aşaǧıdaki indirgeme formüllerinin doǧruluǧunu gösteriniz.
R R
(a) xn sin xdx = −xn cos x + n xn−1 cos xdx (n ∈ N)
R R
(b) (ln x)n dx = x(ln x)n − n (ln x)n−1 dx (n ∈ N)
1 2na2
R R 2
(c) (a2 − x2 )n dx = 2n+1 x(a2 − x2 )n + 2n+1 (a − x2 )n−1 dx (n ∈ N)
R m −1 cosm+1 x
R cosm xdx
(d) cossinnxdxx
= n−1 sin n−1 x
− m−n+2
n−1 sinn−2 x
(n ≥ 2, m ∈ Z, m 6= n)
R cosn xdx 1 n−2
cosn−1 x + cos sin xxdx (n ≥ 2)
R
(e) sin x
= n−1
(f) xn e−x dx = −xn e−x + n xn−1 e−x dx (n ∈ N)
R R

2. In = sin
R nx
sin x
dx ve n > 1 için
2 sin(n − 1)x
In = + In−2
n−1

olduǧunu gösteriniz.
R xn dx
3. In = 1+x 2 ve n > 1 için

xn − 1
In = − In−2
n−1

olduǧunu gösteriniz. Bundan yararlanarak I7 integralini hesaplayınız.


Aynı integral için ikinci bir yol öneriniz. (İpucu: Payı paydaya bölüp
integral alınız.)
4.2. İNTEGRAL ALMA YÖNTEMLERİ 101

4.2.4 Basit Kesirlere Ayırma


1 1
x
− x+1 = x21+x eşitliǧi integral almada bize yeni bir fikir verir. Eşitliǧin saǧ
tarafının integralini almak yerine sol tarafının integralini alırız:
R dx
= ( x1 − x+1
1
R
x2 +x
)dx
R dx R dx
= x − x+1
= ln x − ln(x + 1) + C
x
= ln x+1 +C

olur.
R 3 dx
Örnek 4.28. I = xx2 −1 integralinde olduǧu gibi payın derecesi paydanın
derecesinden büyükse payı paydaya böleriz:

x3 x
2
=x+ 2
x −1 x −1
Şimdi, eşitliǧin saǧındaki kesir daha basit kesirlerin toplamı olarak yazılır:

x A B 1 1
= + ⇒A= , B=
(x − 1)(x + 1) x−1 x+1 2 2

ve buradan,
x3 dx 1/2 1/2
R R
x2 −1
= (x + x−1 + x+1 )dx
= xdx + 21 x−1 dx
+ 12 x+1
R R R dx

= 12 x2 + 12 ln(x − 1) + 21 ln(x + 1) + C

olur.

Genel olarak, f ile g polinom fonksiyonlar olmak üzere fg(x)


(x)
rasyonel fonksi-
yonunu basit kesirlerin bir toplamı olarak yazmak isteyelim. Bunu yapa-
bilmek şu durumlara baǧlıdır:

1. Payın derecesi paydanın derecesinden daha küçük olmalıdır. (Yukarı-


daki örnekteki gibi, durum böyle deǧilse pay paydaya bölünür.)

2. Paydanın, yani g(x)’in çarpanları bilinmelidir.


102 CHAPTER 4. BELİRSİZ İNTEGRAL

(a) (x−r) ifadesi g(x)’in bir çarpanı olsun. Eǧer (x−r)m de (x−r)’nin
g(x)’i bölen en yüksek dereceli çarpanı ise,
A1 A2 Am
+ 2
+ ··· +
x − r (x − r) (x − r)m
f (x)
toplamı g(x)
’in basit kesirleridir.
(b) (x2 + px + q) ifadesi g(x)’in bir çarpanı olsun. (x2 + px + q)n de
(x2 + px + q)’nun g(x)’i bölen en yüksek dereceli çarpanı ise,
B1 x + C1 B2 x + C2 Bn x + Cn
2
+ 2 2
+ ··· + 2
x + px + q (x + px + q) (x + px + q)n
f (x)
toplamı g(x)
’in basit kesirleridir.

2(a) ve 2(b)’ye ilişkin olarak hemen belirtelim ki, kuramsal olarak herhangi
bir g(x) polinomu her zaman 1. ve/veya 2. dereceden polinomların çarpımları
biçiminde ifade edilebilir (çünkü, gerçek katsayılı polinomların kökleri her
zaman gerçek ve/veya karmaşıktır). Ancak pratikte çarpanlara ayırma işi
zor olabilir.
R 3 +2
Örnek 4.29. I = xx3 −x dx integralini hesaplayalım. Payın derecesi pay-
danınkinden daha büyük olduǧundan pay paydaya bölünür ve
x3 + 2 x+2
= 1 +
x3 − x x3 − x
bulunur. Şimdi, saǧ taraftaki kesri basit kesirlerine ayıralım:
x+2 A B C
= + + (∗)
x(x − 1)(x + 1) x x−1 x+1
3
bulunur. Burada paydalar eşitlenip polinomların eşitliǧinden A = −2, B = 2
ve C = 12 bulunur. O halde,
x+2
R
I = [1 + x(x−1)(x+1) ]dx
= x − 2 x + 2 x−1 + 12 x+1
R dx 3 R dx R dx

= x − 2 ln x + 32 ln(x − 1) + 12 ln(x + 1) + C

bulunur.
4.2. İNTEGRAL ALMA YÖNTEMLERİ 103

Not 4.2. Buradaki A, B ve C katsayılarını bulurken alternatif pratik bir yol


da şudur: Madem ki (∗) eşitliǧi her x için geçerlidir, o halde eşitliǧin her iki
yanını x ile çarpıp x yerine 0 (sıfır) yazarak A’nın deǧerini bulabiliriz. Yani,

(x + 2) 6 x Bx Cx 0+2 B.0 C.0


= A+ + ⇒ = A+ +
6 x(x − 1)(x + 1) x−1 x+1 (0 − 1)(0 + 1) 0−1 0+1

ve A = −2 bulunur. Benzer olarak, eşitliǧin her iki yanını x − 1 ile çarpıp x


yerine 1 (bir) yazarak B’nin deǧerini buluruz:

(x + 2) 6 (x − 1) C(x − 1) 1+2 C.0


= A(x − 1) + B + ⇒ = A.0 + B +
x 6 (x − 1)(x + 1) x+1 1(1 + 1) 1+1
3
ve B = 2
bulunur. Yine benzer olarak C = 12 bulunur.
R 2 +3x+2
Örnek 4.30. I = xx(x 2 +1) dx integralini hesaplayalım. Payın derecesi pay-

danınkinden küçük olduǧundan, integrali alınan fonksiyon hemen basit ke-


sirlerine ayrılabilir:

x2 + 3x + 2 A Bx + C
2
= + 2 ⇒ A = 2, B = −1, C = 3
x(x + 1) x x +1

ve aranan integralin deǧeri,


−x+3
I = [ x2 +
R
x2 +1
]dx
R −xdx R 3dx
= 2 ln x + x2 +1 + x2 +1
= 2 ln x − 12 ln(x2 + 1) + 3 arctan x +C

bulunur.

ALIŞTIRMALAR (Basit Kesirlere Ayırma)

1. Aşaǧıdaki eşitliklerin doǧruluǧunu gösteriniz.

(a) x2dx−4 = 14 ln x−2


R
x+2
+C
3/10 (x+3)−2/15
(b) x3 +x2 −6x = ln (x−2) x1/6
R (x+1)dx
+C
R (3x+5)dx 4
(c) x3 −x2 −x+1 = − x−1 + 12 ln x−1
x+1
+C
dx
= 16 ln x+1
R
(d) x2 +7x+6 x+6
+C
104 CHAPTER 4. BELİRSİZ İNTEGRAL

2. Aşaǧıdaki integralleri hesaplayınız:


dx
R
(a) (x−3)(x+2)
dx
R
(b) (x+2)(x+1) 2

xdx
R
(c) (x+1)(x+2)(x+3)
(d) xx+2
R
2 +x dx

(e) (x+1)xdx
R
2 (x+2)2
R dx
(f) (x2 +1)2
R 2
(g) xx(x+x+1
2 +1) dx

R x4
(h) x4 −1 dx
(i) x4dx+1
R
R 2
(j) xx3 +2x+7
+x2 −2
dx
R x3 dx
(k) (x+1)3
R √x+1
(l) x+1
dx


4.2.5 ax2 + bx + c’yi İçeren İntegraller

Bu ifadeyi bulunduran integraller çok karmaşık biçimlerde olabilmektedir.


Bunları basitten karmaşıǧa doǧru inceleyelim.
R√
(A) ax2 + bx + cdx integrali:
Daha önce de incelediǧimiz bu integralde kök içindeki ifade tam kareye
çevirilip uygun bir trigonometrik dönüşüm yardımıyla integral hesaplanabilir.

Örnek 4.31.
R√ Rp
−x2 + 2x + 1dx = −(x2 − 2x − 1)dx
Rp
= −[(x − 1)2 − 2]dx
Rp
= 2 − (x − 1)2 dx
4.2. İNTEGRAL ALMA YÖNTEMLERİ 105
√ √
olur. Burada, x − 1 = 2 sin t ⇒ dx = 2 cos tdt dönüşümü yapılırsa,
Rp Rp
2 − (x − 1)2 dx = 2 − 2 sin2 t cos tdt
√ R
= 2 cos2 tdt
√ R
= 2 1+cos 2
2t
dt
√ √
2 2
= t + cos 2t + C
√2 4 √
= 2
arcsin √ + 2 (cos2 t − sin2 t) +
x−1
C
2 2 4
√ √
2
= 2
arcsin 2 + 42 (−x2 + 2x) + C
x−1

bulunur.

dx
R
(B) √
ax2 +bx+c
integrali:
Anlaşıldıǧı gibi, yukarıdakine benzer olarak bu integralde de karekök içi tam
kare bir ifadeye dönüştürülerek integral hesaplanabilir.
Örnek 4.32. I = √2x2dx
R
−3x+1
integralini hesaplayalım.
dx
R
I = √
2x2 −3x+1
√ √
dx
R
=
2 x2 − 32 x+ 12
√1 √ 3dx2 9 1
R
= 2 (x− 4 ) − 16 + 2
1 √ dx3 2 1
R
= √
2 (x− 4 ) − 16

3
bulunur. Burada x − 4
= 41 sec t ⇒ dx = 14 sec t tan tdt dönüşümü yapılırsa,
1
√1 √4 sec t tan tdt
R
I = 2 1 2 1
16
sec t− 16
√1
R
= 2
sec tdt
= √1 ln(sec t + tan t) + C
2
p
= √1 ln(4x − 3 + (4x − 3)2 − 1) + C
2

bulunur.
R (mx+n)dx
(C) √
ax2 +bx+c
integrali:
Aşaǧıdaki örneǧe benzer olarak hesaplanabilir.
106 CHAPTER 4. BELİRSİZ İNTEGRAL
R (x+3)dx
Örnek 4.33. I = √
x2 +2x+2
integralini hesaplayalım.
1
R (2x+6)dx
I = 2

x2 +2x+2
1 (2x+2)dx 1 √ 4dx
R R
= 2

x2 +2x+2
+ 2 x2 +2x+2
= I1 + I2
2
bulunur. Buradaki √ I1 integralinde x + 2x + 2 = u ⇒ du = 2x + 2 dönüşümü
yapılarak I1 = x2 + 2x + 2 + C1 bulunur. I2 integrali için ise karekök içi
(x + 1)2 + 1 şeklinde tam kare yapılarak x + 1 = tan t ⇒ dx = sec2 tdt
dönüşümü yardımıyla,
2 tdt
I2 = 2 √sec
R
tan2 t+1
R
= 2 sec tdt
= 2 ln(sec t + tan t) + C2

= 2 ln( x2 + 2x + 2 + x + 1) + C2
√ √
ve böylece I = x2 + 2x + 2 + 2 ln( x2 + 2x + 2 + x + 1) + C bulunur.

(D) (mx+n)√dxax2 +bx+c integrali:


R

1
mx+n
= t dönüşümüyle (C) tipi bir integrale dönüştürülerek hesaplanabilir.
Örnek 4.34. I = (x+1)dx 1
= t ⇒ dx = −dt
R

x2 +1
integrali için x+1 t2
dönüşümü
yapılırsa,
I = (x+1)dx
R

x2 +1
tdt
√ 1−t2 2
R
=
( t −1) +1
−1 √ dt1 2
R
=√ 2 (t− 2 ) + 14

bulunur. Burada ise, t − 12 = 21 tan u ⇒ dt = 12 sec2 udu dönüşümü yapılarak,


−1
R
I =√ 2
sec udu
−1
= ln(sec u + tan u) + C

2

−1 1−x+ 2(x2 +1)
= 2 ln(

x+1
)+C

bulunur.
4.2. İNTEGRAL ALMA YÖNTEMLERİ 107
R Pn (x)dx
(E) √
ax2 +bx+c
integrali:
Burada Pn (x) n-inci dereceden bir polinomdur. Bu integrali hesaplamak için,
Qn−1 (x) polinomu n − 1-inci dereceden, katsayıları belirlenecek bir polinom
ve λ belirlenecek bir sayı olmak üzere,
Z
Pn (x)dx √ Z
dx
√ = Qn−1 (x). ax + bx + c + λ √
2
2
ax + bx + c 2
ax + bx + c
eşitliǧi yazılabilir. Burada, her iki tarafın türevi alınıp bulunacak eşitlikten
Qn−1 (x) ile λ sayısı hesaplanır. Daha sonra da (B) tipindeki, λ’nın önündeki
integral hesaplanarak aranan sonuca ulaşılabilir.
R 3 +2x2 +1)dx
Örnek 4.35. I = (x √ x2 +2x
integralini hesaplayalım. Bunun için
Z
(x3 + 2x2 + 1)dx 2
√ Z
dx
√ 2
= (Ax + Bx + C) x + 2x + λ p (∗)
2
x + 2x (x + 1)2 − 1
eşitliǧinde her iki tarafın türevi alınarak,
x3 + 2x2 + 1 √ x+1 λ
√ = (2Ax + B) x2 + 2x + (Ax2 + Bx + C) √ +√
2
x + 2x 2
x + 2x 2
x + 2x
bulunur. Burada ise, polinomların eşitliǧinden A = 31 , B = − 16 , C = 21 ve
λ = − 21 bulunur. (∗) eşitliǧinde bulunacak son şey saǧ taraftaki integraldir.
Burada da x − 1 = sec t ⇒ dx = sec t tan tdt dönüşümü yapılarak
R dx R sec t tan tdt

2
x +2x
= tan t

= ln(sec t + tan t) + K

= ln(x − 1 + x2 + 2x) + K

ve aradıǧımız integralin deǧeri


1 1 1 √ 1 √
I = ( x2 − x + ) x2 + 2x − ln(x − 1 + x2 + 2x) + K
3 6 2 2
bulunur.

√dx
R
(F) (x+p)n ax2 +bx+c
integrali:
1
Burada da x+p
= t dönüşümü yapılarak (E) tipi bir integral bulunur.
108 CHAPTER 4. BELİRSİZ İNTEGRAL

Örnek 4.36. (x+1)3dx 1


= t ⇒ dx = − dt
R

x2 +2x
integralini hesaplayalım. x+1 t2
dönüşümü yapılırsa,
t2 dt
Z Z
dx
p =− √
(x + 1)3 (x + 1)2 − 1 1 − t2

bulunur. Bu ise (E) tipinde bir integraldir ve yukarıdaki yol izlenerek ko-
laylıkla bulunabilir. (Ancak, t = sin u ⇒ dt = cos udu dönüşümü yapılarak
da hesaplanabilir):
Z
t2 dt √ Z
dt
− √ = (At + B) 1 − t2 + λ √ ⇒
1 − t2 1 − t2
t2 √ −2t λ
−√ = A 1 − t2 + (At + B) √ +√
1−t 2 2 1−t 2 1 − t2
eşitliǧinden A = 21 , B = 0 ve λ = − 12 bulunur ve yerine yazılırsa, aranan
integralin deǧeri
√ R dt
I = (At + B) 1 − t2 + λ √1−t 2

= 12 t 1 − t2 − 12 arcsin t + C
q
1 1 1 1
= 2x+2 1 − (x+1) 2 − 2 arcsin x+1 + C

bulunur.

ALIŞTIRMALAR ( ax2 + bx + c’yi İçeren İntegraller)
1. Aşaǧıdaki integralleri hesaplayınız.
R√
(a) x2 + 2x + 10dx
R√
(b) x − x2 dx
R √
(c) x2 − 6xdx

2. Aşaǧıdaki integralleri hesaplayınız.


dx
R
(a) √x2 +4x−8
(b) √4x2dx
R
−4x+5
dx
R
(c) √15−4x−4x 2
4.2. İNTEGRAL ALMA YÖNTEMLERİ 109

3. Aşaǧıdaki integralleri hesaplayınız.


R (x+3)dx
(a) √ 2x−x2

(b) √x2xdx
R
−2x+5
R (1−2x)dx
(c) √8+2x−x2

4. Aşaǧıdaki integralleri hesaplayınız.

(a) (x−1)√dx
R
x2 −2x+2
R dx
(b) x√1−x2
(c) (x+2)√dx
R
x2 +4x+5

5. Aşaǧıdaki integralleri hesaplayınız.


3 dx
(a) √xx2 −x+1
R
R 7 dx
(b) √x1−x 2

R (x3 +3)dx
(c) √x2 −4x+1

6. Aşaǧıdaki integralleri hesaplayınız.

(a) x5 √dx
R
x2 −1

(b) (x+1)3 √dxx2 −x+1


R

R 3
(c) (xx√+2x+2)dx
x2 −x+2

4.2.6 Binom İntegralleri


R
xr (a + bxp )q dx (p, q, r ∈ Q; a, b ∈ R) şeklindeki integrallerdir. p, q, r’nin
durumlarına göre yapılacak dönüşümlerle bu integraller hesaplanır.
(A) q tamsayı ise, r ile p’nin paydalarının OKEK’i k olmak üzere, x = tk
dönüşümü yardımıyla rasyonel bir fonksiyona dönüştürülür.
dx√
R
Örnek 4.37. I = √x(1+ 3 x)2 integralini hesaplayalım. Bu integral I =
R −1 1
x 2 (1 + x 3 )−2 dx şeklinde yazıldıǧında, görülüyor ki r = − 12 , p = 13 ve
110 CHAPTER 4. BELİRSİZ İNTEGRAL

q = −2 (tamsayı) olup, (A) tipindedir. OKEK(2,3)=6 olduǧundan, x = t6


ve dx = 6t5 dt dönüşümüyle

t2 dt
Z
I=6
(1 + t2 )2

bulunur. Burada da t = tan u dönüşümü yardımıyla


R tan2 u(1+tan2 u)du
I =6 (1+tan2 u)2
tan2 udu
R
=6 sec2 u
2
R
= 6 sin udu
= 6 (1−cos2 2u)du
R

= 3(u − 12 sin 2u) + C


= 3(u − sin u cos u) + C
t
= 3(arctan t − 1+t2) + C

√ √
36x
= 3 arctan 6 x − 1+ 3x + C

bulunur.

(B) q tamsayı deǧil ise, binom integralinde yapılacak xp = y dönüşümüyle


Z Z
r 1 p q
r+1
−1
x (a + bx ) dx = (a + by)q y p dy
p
r+1
yazılabilir. Burada ise p
’nin tamsayı olup olmamasına göre karşımıza iki
durum çıkmaktadır.
(B-1) q tamsayı deǧil ancak r+1 p
tamsayı ise, q’nun paydası n olmak üzere,
a + bxp = tn dönüşümü yapılır.

R √
3 √
1+ 4 x R −1 1 1
Örnek 4.38. I = √
x
dx = x 2 (1 + x 4 ) 3 dx integralini hesaplayalım.
Burada, r = − 21 , p = 1
4
ve q = 13 olup, r+1
p
= 2 bir tamsayıdır. O halde, (B-1)
1
tipinde integral olup 1 + x 4 = t3 ⇒ x = (t3 − 1)4 ⇒ dx = 4(t3 − 1)3 3t2 dt
dönüşümü yapılaırsa,
4.2. İNTEGRAL ALMA YÖNTEMLERİ 111

R
I = 12 (t6 − t3 )dt
12 7
= 7
t− 3t4 + C
√ 7 √ 4
= 12
7
(1 + 4
x) 3 − 3(1 + 4 x) 3 + C

bulunur.

(B-2) q tamsayı deǧil ancak r+1 p


+ q tamsayı ise, q’nun paydası n olmak
−p n
üzere, ax + b = t dönüşümü yapılır.
Rq R 1 1 11
Örnek 4.39. I = x

(1+ 3 x)11
dx = x 2 (1 + x 3 )− 2 dx integralinde r = 21 ,
1
p = 13 ve q = − 11
2
olup (B-2) tipindedir. x− 3 + 1 = t2 ⇒ x = (t2 − 1)−3 ⇒
2 −4
dx = −6t(t − 1) dt dönüşümü yapılırsa

I = −6 t−10 dt
R

= 23 t−9 + C

3 9
x
= 23 ( 1+ √
3 x)2 + C

bulunur.
R
Not 4.3. P.L.Chebyshev (1821-1894) göstermiştir ki, xp (a + bxr )q dx integ-
ralinin var olması için q, p+1
r
veya p+1
r
+ q rasyonel sayılarından en az biri
tamsayı olmalıdır.

Belirsiz integralle ilgili söyleyeceklerimiz bu kadardır. Şimdi karşımıza şu


soru çıkmaktadır: Acaba her fonksiyon integrallenebilir midir? Hangi tür
fonksiyonlar integrallenebilirdir? 1835 yılında Liouville aşaǧıdaki sonuçları
elde etmiştir:
R
f (x) ile rasyonel fonksiyonlar olmak üzere, I = f (x)eg(x) dx integrali
R g(x) g(x)
varsa f (x)e dx = R(x)eg(x) şeklinde olmalıdır. Burada, R(x) keyfi sabit
de içeren bir rasyonel fonksiyondur. Bu eşitliǧin her iki tarafının türevi
alınarak
R0 (x) + R(x)g(x) = f (x) (∗)
eşitliǧi bulunur. Bunun tersi de doǧrudur. Yani, (∗) eşitliǧini saǧlayan ras-
yonel bir R(x) fonksiyonu varsa, I integrali I = R(x)eg(x) şeklinde mevcuttur.
2
Böyle integrallere elementer (temel) integral denir. Örneǧin, I = e−x dx
R
112 CHAPTER 4. BELİRSİZ İNTEGRAL
R √sin x
dx, dx , sinx x dx in-
R R R
temel (basit) deǧildir. Benzer olarak, xx dx, x x
tegralleri de temel deǧildir. Bu tip integrallerin deǧerleri nümerik (sayısal)
yollarla hesaplanır.
ALIŞTIRMALAR (Binom İntegralleri)

1. Aşaǧıdaki integralleri hesaplayınız.


R q 1+x2
(a) x2
dx
R x3 dx
(b) √1+x2
R √x4 +4dx
(c) x3
R √3 2
x√dx
(d) 1+ x
dx
R
(e) 2 4
3
x (1+x ) 4

2. Aşaǧıdaki integralleri hesaplayınız.


3
(a) x3 (1 + 2x2 )− 2 dx
R
R dx
(b) √
4
1+x4
R dx
(c) x √ 3
1+x5

(d) √ √ dx
R
3 √
4
x3 1+ x3

3
√ xdx
R
(e) √
1+ 3 x
5
√ x dx
R
(f) 3
(x −1)2
2
Chapter 5

BELİRLİ İNTEGRAL

Bu bölümde belirli integral ve uygulamaları üzerinde duracaǧız. Bunun için


ilk önce bir eǧrinin altındaki alanı yaklaşık olarak nasıl hesaplayabileceǧimizi,
daha sonra da Newton ile Leibniz’in birbirinden baǧımsız olarak geliştirdikleri
bir yöntem olan diferensiyel ve integral yardımıyla nasıl kesin olarak hesapla-
yabileceǧimizi göreceǧiz.

5.1 Eǧri Altındaki Alan


Geometri bize belirli çokgenlerin alanlarını nasıl bulacaǧımız konusunda yar-
dımcı olur. Herhangi bir çokgenin alanını üçgenlere bölerek hesaplayabiliriz.
Dairenin alanını da bilindik πr2 formülünden kolayca hesaplarız. Ancak,
göreceǧiz ki bu basit formülün ardındaki düşünce çok da basit deǧildir.

Problem 5.1. Yarıçapı r olan bir çemberin çevrelediǧi alanı çember içine
çizeceǧimiz düzgün çokgenlerin alanından yararlanarak yaklaşık olarak he-
saplamak istiyoruz.

Şekil 5.1 Daire içine çizilen düzgün çokgenler.

113
114 CHAPTER 5. BELİRLİ İNTEGRAL

İlk şekilde bir eşkenar üçgen (düzgün üçgen) vardır ve alanı

A3 = 3 · 21 · r · r · sin 2π
3

3 3 2
= 4
r
olur. İkinci şekilde bir kare (düzgün dörtgen) vardır ve alanı

A4 = 4 · 21 · r · r · sin 2π
4

= 2r2
olur. Böyle devam ederek, genel olarak bir düzgün n-gen çizilir ve bunun da
alanı

An = n · 12 · r · r · sin 2π
n

= n2 r2 sin 2π
n

olur. Şimdi de, r yarıçaplı dairenin alanını, daire içine çizilen düzgün n-
genin kenar sayısını sonsuza götürürken bulunan limn→∞ An limiti olarak
tanımlayalım. Bu durumda, aradıǧımız dairenin alanı

limn→∞ An = limn→∞ n2 r2 sin 2π


n

sin
= limn→∞ r2 · 2π
n
·π
n

= πr2
bulunur. Aşaǧıda göreceǧimiz gibi, eǧri altındaki alanları hesaplarken de
benzer bir yol izleyeceǧiz.

Problem 5.2. y = f (x) fonksiyonu [a, b] kapalı aralıǧında sürekli bir fonk-
siyon olsun. Problemi basitleştirmek için, f ’nin bu aralıkta negatif olmadıǧını
kabul edelim. Böylece, aşaǧıdaki şekildeki gibi, verilen y = f (x) eǧrisi ile
yanlardan x = a, x = b dikey doǧruları ve aşaǧıdan da x ekseni ile sınırlı
olan alanın hesaplanması problemini ele alalım.

Şekil 5.2 Bir eǧrinin altındaki alan.


5.1. EǦRI ALTINDAKI ALAN 115

Bu alanı, x = a ve x = b sınır deǧerleri ile x1 , x2 , ..., xn−1 şeklindeki ara


noktalardan geçen x eksenine dik doǧrularla ∆x = (b − a)/n genişliǧindeki
sabit şeritlere bölelim.

Şekil 5.3 Bir eǧrinin altındaki alanın şeritlere bölünmesi.


Her şeridin alanını bulmak için içine çizilen dikdörtgenin alanını kullanalım.
Birinci kapalı aralıkta a ile x1 arasında öyle bir c1 sayısı vardır ki ilk dikdört-
genin alanı f (c1 )∆x; ikinci kapalı aralıkta öyle bir c2 sayısı vardır ki ikinci
dikdörtgenin alanı f (c2 )∆x; ... vb olur. c1 ’in yeri a ile x1 ’in arasında f ’nin
minimum olduǧu yerdir. Benzer biçimde, ci ’nin yeri xi−1 ile xi arasında f ’nin
minimum olduǧu yerdir. Böylece, şekildeki dikdörtgenlerin alanları toplamı,
∆x = b−an
, a = x0 ve b = xn olmak üzere,

An = f (c1 )∆x + f (c2 )∆x + · · · + f (cn )∆x


= ni=1 f (ci )∆x
P

olacaktır. Eǧri altındaki alana dönersek, bu alanı şeritlerin sayısı sınırsız ar-
tarken şeritlerin içine çizilen dikdörtgenlerin alanları toplamının limiti olarak
tanımlıyoruz. Simgesel olarak gösterirsek,
n
X
A = lim f (ci )∆x (∗)
n→∞
i=1

olur.
Gözlem 5.1. f sürekli ise (∗) ile gösterilen limit vardır. Yani, n’nin giderek
büyük deǧerlerini alarak ve her n için şeritlerin içine çizilen dikdörtgenlerin
alanları toplamını hesaplayarak, n artarken birbirinden (ve eǧrinin altındaki
alandan) rasgele küçük ölçülerde farklı sonuçlar elde edebiliriz.
Gözlem 5.2. Şeritlerin içi yerine dışına çizilecek dikdörtgenler de kulla-
nılabilir. Bu durumda, (∗)’daki ci sayıları yerine ei sayıları kullanılır. Bu ei
sayıları da sözkonusu aralıklarda eǧrinin minimum yerine maksimum olduǧu
yerler olur. Bu durumda ise, bulunan alan gerçek alandan biraz büyük ol-
makla birlikte, limit alındıǧında her iki yöntemle de aynı sonuca ulaşılır.
116 CHAPTER 5. BELİRLİ İNTEGRAL

Örnek 5.1. f (x) = x2 + 1; a = 0, b = 1 olsun. Eǧri altındaki alanı n = 4


şerit alarak şeritler ıçine çizilen dikdörtgenler yardımıyla hesaplayalım. Bu
durumda ∆x = 14 olup, [0, 1] aralıǧı 4 eşit aralıǧa bölünmüştür.

Şekil 5.4 f (x) = 1 + x2 altındaki alanın 4 şeride bölünmesi.

Şeritlerin içine çizilen dikdörtgenlerin alanları toplamı,


1 2 3 16 17 20 25
f (0)∆x + f ( )∆x + f ( )∆x + f ( )∆x = + + + = 1.21875
4 4 4 64 64 64 64
olur. Eǧrinin altındaki alan şeritlerin içine çizilen alanlardan biraz daha
büyük olduǧundan, eǧri altındaki alanın 1.22’den biraz daha büyük olacaǧı
söylenebilir. Gerçekten, göreceǧiz ki bu alan tam olarak 34 ’tür.
Eǧrinin altındaki gerçek alanı bulmak için, [0, 1] aralıǧını n eşit parçaya bölüp
∆x = n1 alırsak,

An = f (0)∆x + f ( n1 )∆x + · · · + f ( n−1


n
)∆x
12 1 22 1 (n−1)2 1
= (1 + 0) n1 + (1 + )
n2 n
+ (1 + )
n2 n
+ · · · + (1 + n2
)n
1
= [n + n2
(12 + 2 2
+ 3 + · · · + (n − 1)2 )] n1
2

(n−1)n(2n−1)
=1+ 6n3

olur. Eǧri altındaki alan, n → ∞ için An ’nin limiti olarak tanımlandıǧından,


1
A = lim An = 1 +
n→∞ 3
bulunur. Aynı alanı şeritler dışına çizilen dikdörtgenler yardımıyla da hesap-
layabiliriz. Bu durumda,
5.1. EǦRI ALTINDAKI ALAN 117

Un = f ( n1 )∆x + f ( n2 )∆x + · · · + f (1)∆x


12 1 2 2 n2 1
= (1 + n2 n
) + (1 + n2 2 ) n1 + (1 + n3 2 ) n1 + · · · + (1 + )
n2 n

= [n + n12 (12 + 22 + 32 + · · · + n2 )] n1
= 1 + n(n+1)(2n+1)
6n3

olur. Eǧri altındaki alan, n → ∞ için Un ’nin limiti olarak tanımlandıǧından,

1
A = lim Un = 1 +
n→∞ 3
bulunur. Dikkat edilirse, bu dǧer daha önce bulduǧumuzla aynıdır.

Örnek 5.2.

ALIŞTIRMALAR (Eǧri Altındaki Alan)

1. f (x) = x1 için, x = 1 ile x = 5 arasında, x ekseni üstünde ve f (x)’in


altında kalan alanı hesaplamak istiyoruz. Yukarıdaki probleme benzer
olarak

(a) n = 5 alarak alanı yaklaşık olarak hesaplayınız.


(b) Sözkonusu alanı şeritler içine çizilen dikdörtgenlerin alanları top-
lamının limiti olarak hesaplayınız.
(c) Aynı alanı şeritler dışına çizilen dikdörtgenlerin alanları topla-
mının limiti olarak hesaplayınız.

2. f (x) = x2 için, x = 0 ile x = 4 arasında, x ekseni üstünde ve f (x)’in


altında kalan alanı hesaplamak istiyoruz.

(a) n = 6 alarak alanı yaklaşık olarak hesaplayınız.


(b) Sözkonusu alanı şeritler içine çizilen dikdörtgenlerin alanları top-
lamının limiti olarak hesaplayınız.
(c) Aynı alanı şeritler dışına çizilen dikdörtgenlerin alanları topla-
mının limiti olarak hesaplayınız.
118 CHAPTER 5. BELİRLİ İNTEGRAL

5.2 Diferensiyel ve İntegralle Alan Hesabı


Bundan önceki derste limitler hesaplayarak bazı alanları bulmuştuk. Şimdi
ise, Leibniz ve Newton tarafından geliştirilen bir yol izleyerek, diferensiyel ve
integral hesap yardımıyla alanların kesin olarak nasıl kolaylıkla bulunduǧunu
göreceǧiz.

Şekil 5.5 y = f (x) eǧrisi altındaki alan.

[a, b] tanım aralıǧında şekildeki gibi negatif olmayan sürekli bir f fonksi-
yonunu ele alalım. Aba işareti, f eǧrisi ile x ekseni arasında kalan a’dan b’ye
kadar olan alanı göstersin. Eǧer a ≤ c ≤ b ise,

Aba = Aca + Abc

olur. Burada, c = a alınarak Aaa = 0 olduǧu kolaylıkla görülür. Alanın işareti


konusunda; x ekseni üzerinde, alanın soldan saǧa gidildiǧinde pozitif, saǧdan
sola gidildiǧinde ise negatif olduǧunu kabul edeceǧiz. Bu durumda,

Aba = −Aab

olacaktır. Aşaǧıdaki teorem, amacımız olan ters türev (integral) ile alan
arasındaki ilişkiyi kurabilmek için işimize yarayacak.

Teorem 5.1. (Alanlar için ortalama deǧer teoremi) f fonksiyonu [a, b] aralı-
ǧında negatif olmayan sürekli bir fonksiyon olsun. Aba de yukarıda tanımladı-
ǧımız gibi bu aralıkta f ’nin grafiǧinin altında kalan alanı göstersin. Bu du-
rumda, a ile b arasında
Aba = f (c) · (b − a)
olacak biçimde en az bir c sayısı vardır.
5.2. DIFERENSIYEL VE İNTEGRALLE ALAN HESABI 119

İspat: m ve M sayıları f fonksiyonunun [a, b]’de sırasıyla minimum ve mak-


simumlarını göstersin. Bu durumda,

Aba
m(b − a) ≤ Aba ≤ M (b − a) ⇒ m ≤ ≤M
b−a
Aab
olur. f sürekli olduǧundan ve b−a sayısı [a, b] aralıǧında f ’nin maksimum ve
minimum deǧerleri arasında bulunduǧundan, a ile b arasında öyle bir c sayısı
vardır ki
Aba
f (c) =
b−a
olur. Buradan da istenen sonuca ulaşılır. (Aşaǧıdaki şekli inceleyiniz.)

Şekil 5.6 y = f (x) eǧrisi altındaki alana bir dikdörtgenin alanı ile yaklaşım.

Not 5.1. Bu teorem bize şunu söyler: Eǧri altında [a, b] aralıǧındaki alan
P Qba dikdörtgeninin alanına, yani, f (c)(b − a)’ya eşittir (Şekil:2.6).

Şimdi, Aba alanını hesaplamak için aşaǧıdaki şekildeki gibi ∆x 6= 0 artışı


alarak, a ile b arasında bir x apsisi, Axa ve Ax+∆x
x alanlarını gözönüne alalım.

Şekil 5.7 Axa alan fonksiyonunda ∆x artımı.


120 CHAPTER 5. BELİRLİ İNTEGRAL

Görülüyor ki,
Axa + Ax+∆x
x = Aax+∆x ⇒ Aax+∆x − Axa = Axx+∆x
| {z }
ve işaretlenen kısım ∆(Axa ) olduǧundan,
∆(Axa ) = Axx+∆x
bulunur. Oysa, yukarıdaki teoremden biliyoruz ki,

Ax+∆x
x = f (c)(x + ∆x − x)
= f (c)∆x
yazılabilir. Burada, x < c < x + ∆x olup, ∆x → 0 için c → x olur. (Yani,
∆x deǧeri 0’a yaklaşırken c de x’e yaklaşır.) O halde,
∆(Axa )
f (c) =
∆x
ve dolayısıyla,
∆(Axa )
lim = lim f (c)
∆x→0 ∆x c→x
olur. Buradan, türev tanımı hatırlanırsa,
(Axa )0 = f (x)
bulunur. Şimdi, eǧer türevi f (x) olan fonksiyona F (x) dersek ve
Z
F (x) + C = f (x)dx

ile gösterirsek, (C keyfi sabiti türevi 0 olan bir sabittir),


Z
x
Aa = f (x)dx = F (x) + C

yazılabilir. Şimdi, madem ki Aaa = 0’dır, buradan C = −F (a) olur. Bu deǧer


son eşitlikte yerine yazılırsa,
Axa = F (x) − F (a) =: F (x)]xa
olur. Burada, F (x)]xa deǧerinin anlamı F (x) − F (a)’dır. Esas amacımız Aba
alanını bulmak olduǧundan, bu son eşitlikte x yerine b yazarak
Aba = F (b) − F (a)
eşitliǧini buluruz.
5.2. DIFERENSIYEL VE İNTEGRALLE ALAN HESABI 121

Sonuç 5.1. Üstten y = f (x) ve alttan x ekseni ile sınırlı, [a, b] aralıǧındaki
alan olan Aba deǧerini bulmak için, f (x) fonksiyonunun integralini alır ve
böylece F (x) + C fonksiyonunu buluruz. Daha sonra, bunun x = a için
deǧerini bulup x = b için bulacaǧımız deǧerinden çıkarırız. Yani, aradıǧımız
alanı
Aba = F (x)]ba
eşitliǧi ile hesaplarız. Bunu da kısaca,
Z b
b
Aa = f (x)dx
a
ile gösteririz. Buna, [a, b] aralıǧında f (x)’in belirli integrali denir.
Örnek 5.3. Üstten f (x) = 1+x2 , alttan x ekseni ile sınırlı, [0, 1] aralıǧındaki
alanı hesaplayalım. Yukarıdaki bilgilerimizi kullanırsak, istenen alanı
R1
A10 = 0 (1 + x2 )dx
= x + 13 x3 + C]10
= (1 + 13 + C) − (0 + C)
= 43
şeklinde hesaplarız. Dikkat edilirse, bu deǧer daha önce bulduǧumuzla aynıdır.
Şunu da belirtmeden geçmeyelim: Görülüyor ki belirli integralde C integral
sabitinin bir önemi kalmamaktadır.
Örnek 5.4. y = 4 − x2 ile x ekseni arasında kalan alanı hesaplayalım.

Şekil 5.8 y = 4 − x2 ile x ekseni arasında kalan bölge.


Şekilden görülüyor ki,
R +2
A+2
−2 = −2
(4 − x2 )dx
= 4x − 13 x3 ]+2
−2
−8
= (8 − 83 ) − (−8 − 3
)
32
= 3
122 CHAPTER 5. BELİRLİ İNTEGRAL

bulunur.

Bu örnekte görülüyor ki alanını aradıǧımız bölge y eksenine göre simetriktir.


O halde y ekseninin sadece bir tarafında kalan alan hesaplanarak işlem ka-
labalıǧından kaçınılabilir. Aşaǧıdaki teorem tek ve çift fonksiyonların belirli
integralini daha kolay hesaplamak içindir.

Teorem 5.2. f : [a, b] → R sürekli olsun.


Ra Ra
1. f çift ise −a f (x)dx = 2 0 f (x)dx,
Ra
2. f tek ise −a f (x)dx = 0 olur.

İspat:
Ra
I = −a
f (x)dx
R0 Ra
= −a
f (x)dx + 0
f (x)dx
= I1 + I2
R0
olsun. I1 = −a
f (x)dx integralinde x = −t ⇒ dx = −dt yazılırsa,
R −t=0
I1 = −t=−a
f (−t)(−dt)
R t=0
= − t=a f (−t)dt
Ra
= 0
f (−t)dt
Ra
= 0
f (−x)dx

olur. Bu deǧer I integralinde yerine yazılırsa,


Ra
I = −a f (x)dx
Ra Ra
= 0 f (−x)dx + 0 f (x)dx
Ra
= 0 [f (−x) + f (x)]dx
bulunur. Şimdi,
Ra
1. f çift ise f (−x) = f (x) olacaǧından, I = 2 0 f (x)dx,
Ra
2. f tek ise −a f (x)dx = 0 olur ve ispat biter. 2
5.2. DIFERENSIYEL VE İNTEGRALLE ALAN HESABI 123

Örnek 5.5. R2 R2
−2
|x|dx = 2 0
|x|dx
R2
=2 0
xdx
2 2
= x ]0
=4
olur.

Örnek 5.6. −π | sin5 x|dx integralini hesaplayalım. Görülüyor ki mutlak
deǧerli bir fonksiyondur ve dolayısıyla [−π, π] aralıǧında çifttir. Yukarıdaki
bilgilerimizi kullanırsak,
Rπ 5

−π
| sin x|dx = 2 0
| sin5 x|dx

= 2 0 sin5 xdx

= 2 0 (1 − cos2 x)2 sin xdx
= −2[cos x − 32 cos3 x + 51 cos5 ]π0
32
= 15
bulunur.

ALIŞTIRMALAR (Diferensiyel ve İntegralle Alan Hesabı)


1. Aşaǧıdaki alıştırmalarda, y = f (x) eǧrisi, x ekseni ve verilen doǧrularla
sınırlı alanları hesaplayınız.

(a) y = 1 + x2 ; x = 0, x = 3
(b) y = 2x + 3; x = 0, x = 1

(c) y = 2x + 1; x = 0, x = 4
1
(d) y = (2x+1)2
; x = 1, x = 2

2. Koordinat eksenleri ile x + y = 1 doǧrusu tarafından sınırlanan alanı


integral yardımıyla bulunuz.

3. r yarıçaplı çemberin çevrelediǧi alanı integral yardımıyla hesaplayınız.


R2 √ √
4. 0 [|x2 |]dx = 5 − 2 − 3 olduǧunu gösteriniz.
R9 √
5. 0 [| x|]dx = 13 olduǧunu gösteriniz.
124 CHAPTER 5. BELİRLİ İNTEGRAL

6. Aşaǧıdaki fonksiyonların [−1, 2] aralıǧındaki integrallerini hesaplayınız.

(a) f (x) = [|x|]


(b) f (x) = [| − x|]
(c) f (x) = [|2x|]
(d) f (x) = [|x + 12 |]
R2
7. 0
|x3 − 3x2 + 2x|dx integralini hesaplayınız.

8. −π
| cos3 x|dx integralini hesaplayınız.

5.3 Uygulamalar
Bu kesimde bir eǧrinin yay uzunluǧu ile bir eǧrinin x ekseni etrafında döndü-
rülmesi ile oluşan cismin hacim hesaplarının integral yardımıyla nasıl yapıla-
bildiǧini göreceǧiz.

5.3.1 Yay Uzunluǧu Hesabı


y = f (x) eşitliǧi ile verilen türevli f fonksiyonunu düşünelim. Bu eǧrinin
x1 = a, x2 = b apsisli noktaları arasında kalan parçasının uzunluǧunu bulmak
istiyoruz.

Şekil 5.9 y = f (x) eǧrisinin [a, b] aralıǧındaki yay uzunluǧu.

∆x çok küçük alındıǧında AB


d yayının ∆l uzunluǧu ile [AB] doǧru parçasının
uzunluǧu yaklaşık eşittir. Buna göre,
p
∆l ≈ (∆x)2 + (∆y)2
q
∆y 2
≈ 1 + ( ∆x ) ∆x
5.3. UYGULAMALAR 125

yazılabilir. Buradan, her iki taraf ∆x ile bölünür ve ∆x → 0 limiti alınırsa


r
∆l ∆y 2 p
lim = lim 1+( ) ⇒ l0 = 1 + (y 0 )2
∆x→0 ∆x ∆x→0 ∆x
bulunur. O halde, [a, b] aralıǧındaki l yay uzunluǧu
Z bp
l= 1 + (y 0 )2 dx
a

bulunur.

Örnek 5.7. r yarıçaplı merkezil çemberin denklemi x2 + y 2 = r2 olup bunun


çevresini hesaplamak istiyoruz. y 0 = − xy olduǧundan, aradıǧımız uzunluk
Rr p
l = −r
1 + (y 0 )2 dx
Rr q 2
= −r 1 + xy2 dx
Rrq 2
= 4 0 1 + r2x−x2 dx
Rrq 1
= 4r 0 r2 −x 2 dx

= 4r arcsin xr ]r0
= 2πr
q
2
bulunur. Yukarıdaki eşitliǧin ikinci adımında 1 + xy2 iki deǧişkenli fonksi-
yonunun hem x hem de y’ye göre çift olduǧunu, yani, grafiǧin sırasıyla hem
y ekseni ve hem de x eksenine göre simetrik olduǧunu belirtelim. Bu yüzden
hesaplamada 4 çarpanı gelmiştir.

Örnek 5.8. y = x2 parabolünün [0, 1] aralıǧındaki yay uzunluǧunu hesapla-


yalım. y 0 = 2x olup, aradıǧımız uzunluk
R1p
l = 0 1 + (y 0 )2 dx
R1√
= 0 1 + 4x2 dx
√ √
5 ln( 5+2)
= 2
+ 4

bulunur.
126 CHAPTER 5. BELİRLİ İNTEGRAL

5.3.2 Hacim Hesabı


(1) Kesit Yöntemi:
Aşaǧıdaki şekildeki gibi bir B cismi ile bir l doǧrusu verilmiş olsun. Doǧru
üzerinde bir O başlangıç noktası alalım.

Şekil 5.10 B cismi ile l doǧrusuna dik düzlemler.


l doǧrusunu t noktasında dik kesen düzlemi Dt ve B cismi ile Dt düzleminin
arakesiti olan bölgenin alanını A(t) ile gösterelim.

A : t → A(t)

fonksiyonu [a, b] aralıǧında integrallenebilir olsun. B’nin Da ile Dt (a < t)


düzlemi arasında kalan parçasının hacmini V (t) ile gösterirsek, V (a) = 0 olup
V (b) = V istenen bütün hacimdir. V (t + ∆t) − V (t) farkı yaklaşık olarak
yüksekliǧi ∆t ve taban alanı A(t) olan dik silindirin hacmini verir. Şu halde,

V (t + ∆t) − V (t) = ∆tA(t) ⇒


V (t + ∆t) − V (t)
A(t) =
∆t
olur. ∆t → 0 için
Z x
0
V (t) = A(t) ⇒ V (x) = A(t)dt
a
Rb
olup V (b) = V olduǧundan, V = a
A(t)dt bulunur.

Not 5.2. Bu incelemeden şu prensip çıkarılabilir: İki cisim bir doǧruya dik
düzlemlerle kesildiǧinde elde edilen kesitleri eşit alana sahip ise, bu iki cisim
eşit hacimlidir. Buna Cavalieri Prensibi denir.
5.3. UYGULAMALAR 127

Örnek 5.9. Tepe noktasının taban düzlemine olan uzaklıǧı H birim olan bir
dairesel koninin taban yarıçapı r olduǧuna göre bu koninin hacmini bulalım.

Şekil 5.11 Bir eǧik koninin hacmi.

Tabandan t kadar yüksekteki kesit yine bir daire olup, yarıçapına ρ denirse,
ρ H −t t
= ⇒ ρ = r(1 − )
r H H
olur. Böylece kesitin alanı πρ2 = πr2 (1− Ht )2 olur. Böylece aradıǧımız hacim,
RH t 2
V = 0
πr2 (1 −H
) dt
1
= 3
πr2 H( Ht − 1)]H
0
1 2
= 3
πr H

bulunur. (Yani, eǧik veya dik bir koninin hacmi, taban alanı ile yüksekliǧinin
çarpımının 3’te 1’ine eşittir.)

Örnek 5.10. r yarıçaplı kürenin hacmini hesaplayalım.

Şekil 5.12 r yarıçaplı kürenin hacmi için kesit.


128 CHAPTER 5. BELİRLİ İNTEGRAL

Görüldüǧü gibi kesit alanı πy 2 dir. Öte yandan, x2 + y 2 = r2 olduǧundan,


Rr
V = 2 0 πy 2 dx
Rr
= 2π 0 (r2 − x2 )dx
= 34 πr3

bulunur.

(1) Disk Yöntemi:


Bu kesimde dönel cisimlerin hacimlerinin nasıl bulunabileceǧini göreceǧiz.

Şekil 5.13 Dönel cismin hacmi.


n tane diskin hacimleri toplamı yaklaşık olarak dönel cismimizin hacmini
verecektir. k inci diskin hacmi π[f (xk )]2 ∆x olacaǧından, aradıǧımız hacim
bunların tümünün n → ∞ için toplamıdır:

V = limn→∞ nk=1 π[f (xk )]2 ∆x


P
Rb
= π a f 2 (x)dx

olur. Kesit yöntemini kullanarak da aynı sonucu bulabilirdik: Bu durumda


kesit alanı (ilk şekilden) πf 2 (x) olup, aradıǧımız hacim
Z b
V = πf 2 (x)dx
a

olur.
5.3. UYGULAMALAR 129

Örnek 5.11. Alttan x ekseni, üstten y = 13 x2 + 1 parabolü, ile yanlardan


x = 0 ve x = 3 doǧruları tarafından sınırlanan düzlemsel bölgenin x ekseni
etrafında döndürülmesi ile oluşan cismin hacmini bulalım.
R3
V =π 0
y 2 dx
R3 1 2
=π ( x + 1)2 dx
0 3
R3 1 4 2 2
=π ( x + 3x +
0 9
1)dx
5
= π( 19 x5 + 29 x3 + x)]30
72π
= 5

olur.

Not 5.3. x = g(y) eǧrisi, y = c ve y = d doruları ile y ekseni tarafından


sınırlanan bölgenin y ekseni etrafında döndürülmesiyle oluşan dönel cismin
hacmi,
Z d
V = x2 dy
c

olur.

Not 5.4. y=f (x) eǧrisi, x=a, x=b ve y=k doǧruları tarafından sınırlanan
düzlemsel bölge, y=k doǧrusunun bir tarafında kalsın. Bu bölge, y = k
etrafında döndürülürse, meydana gelen dönel cismin hacmi y = f (x) − k
eǧrisinin x ekseni etrafında döndürülmesi ile oluşan cismin hacmine eşit
olacaǧından
Z b
V =π |f (x) − k|2 dx
a

olur.

Not 5.5. [a, b] aralıǧında 0 ≤ g(x) ≤ f (x) olsun. y = g(x) ile y = f (x)
eǧrileri arasında kalan şeridin x = a ve x = b arasında kalan kısmının x
ekseni etrafında döndürülmesiyle oluşan cismin hacmi
Z b
V =π [f 2 (x) − g 2 (x)]dx
a

olur. Buna güzel bir örnek aşaǧıdadır.


130 CHAPTER 5. BELİRLİ İNTEGRAL

Örnek 5.12. (Simit hacmi) 0 < a < b olsun. Merkezi (0, b) ve yarıçapı
a olan dairenin x ekseni etrafında döndürülmesi ile oluşan simidin hacmini
bulalım.

Şekil 5.14 Dairenin x ekseni etrafında döndürülmesiyle oluşan Simidin


(tor’un) hacmi.

Çember denklemi x2 + (y − b)2 = a2 olup y = b ± a2 − x2 olur. Buna göre
aradıǧımız hacim,
Ra √ √
V = π −a [(b + a2 − x2 )2 − (b − a2 − x2 )2 ]dx
Ra √
= 4bπ −a a2 − x2 dx
Ra√
= 8bπ 0 a2 − x2 dx
= 2π 2 a2 b2

olur.
5.3. UYGULAMALAR 131

ALIŞTIRMALAR (Uygulamalar)
1. Aşaǧıda denklemleri verilen eǧrilerin karşılarında apsisleri verilen nok-
talar arasında kalan parçalarının uzunluklarını hesaplayınız.
3
(a) y = 13 (x2 + 2) 2 , (x = 0, x = 3)
3
(b) y = x 2 , (x = 0, x = 5)
3 1
(c) y = 32 x 2 − 12 x 2 , (x = 1, x = 4)
2. y = e−2x , y = 1 + x ve x = 1 eǧrileri ile sınırlı bölgenin x ekseni
etrafında döndürülmesi ile oluşan cismin hacmini hesaplayınız.
3. Birinci bölgede y = x3 ve y = 2x − x2 eǧrileri ile sınırlı alan A olsun.
Aşaǧıdakileri hesaplayınız.
(a) A’nın alanı.
(b) A’nın x ekseni çevresinde döndürülmesiyle oluşan cismin hacmi.
(c) A’nın y ekseni çevresinde döndürülmesiyle oluşan cismin hacmi.
4. y = x ve y = x2 eǧrileriyle sınırlı bölgenin aşaǧıdaki doǧrular etrafında
döndürülmesiyle elde edilen cisimlerin hacimlerini hesaplayınız.

(a) x ekseni.
(b) y ekseni.
(c) y = 2 doǧrusu.

5. Bir cismin tabanı, yarıçapı 3 olan bir disktir. Tabana dik olan pa-
ralel kesitler, hipotenüsü taban üzerinde olan ikizkenar dik üçgenlerse,
cismin hacmini bulunuz.
6. Bir cismin tabanı, y = x2 ve y = 2 − x2 parabolleriyle sınırlı bölgedir.
x eksenine dik olan kesitler, bir kenarı taban üzerinde olan karelerse,
cismin hacmini bulunuz.
7. (a) Bir cismin tabanı, köşeleri (1,0), (0,1), (-1,0) ve (0,-1) nokta-
larında olan bir karedir. x eksenine dik olan kesitlerin her biri
bir yarıdairedir. Cismin hacmini bulunuz.
(b) Yukarıdaki cismi keserek, bir koni oluşturacak biçimde tekrar bir-
leştirebileceǧimizi gösteriniz. Böylelikle, hacmini daha basit bir
biçimde bulunuz.

You might also like